SlideShare a Scribd company logo
DANAMATH
www.toanhocdanang.com
www.facebook.com/ToanHocPhoThongDaNang
Tổng Hợp
5 Đề và
đáp án
GV:Phan Nhật Nam
ÔN THI QUỐC GIA 2015
ĐỀ SỐ 1
Câu 1(2,0 điểm). Cho hàm số
2 1
1
x
y
x
+
=
+
có đồ thị (C).
1. Khảo sát sự biến thiên và vẽ đồ thị (C) của hàm số đã cho.
2. Viết phương trình tiếp tuyến của đồ thị (C) biết tiếp tuyến đó cách đều hai điểm A(2; 4) và B(-4; -
2).
Câu 2(1,0 điểm):
1. Giải phương trình sau: 2cos6 2cos4 3cos2 sin 2 3x x x x+ − = +
2. Giải phương trình : 3 3 33log 2 log log2 3
.2 12x x
x x x+ = +
Câu 3 (1,0điểm).
1. Tính tích phân: 4
20
sin
5sinx.cos x + 2cosx
x
I dx
π
= ∫
2. Cho hình phẳng (H) được giới hạn bởi đồ thị của các hàm số sin3 , 0 , 0y x y x= = = và
6
x
π
= .
Tính thể tích khối tròn xoay sinh bởi (H) khi quay quanh trục Ox.
Câu 4 (1,0 điểm).
1. Tìm tập hợp các điểm biểu diển của số phức z thỏa mãn đẳng thức sau : 2 1 1z i iz i− + = + − .
trong các số phức thỏa điều kiện trên , hãy tìm số phức z có môđun nhỏ nhất.
2. Chọn ngẫu nhiên hai số hạng trong khai triển nhị thức Newton ( )
100
6
3 5− . Tính xác suất để hai
số được chọn có một số là số hữu tỷ và một số là số vô tỷ.
Câu 5(1,0 điểm).Cho hình chóp SABCD có đáy ABCD là hình vuông, tam giác SAB vuông góc tại S và
nằm trong mặt phẳng vuông góc với đáy (ABCD). Biết khoảng cách giữa hai đường thẳng SC
và AB bằng
2 57
19
a
; góc tạo bới SB và mặt phẳng (ABCD) bằng 300
. Tính thể tích của khối
chóp SABCD và diện tích mặt cầu ngoại tiếp tứ diện SBCD.
Câu 6(1,0điểm).Trong không gian với hệ trục tọa độ Oxyz, cho điểm M(0; 2; 0) và hai đường
thẳng 1
1 2 1
:
2 2 1
x y z
d
− − +
= =
−
và 2
3 1
:
2 2 1
x y z
d
− +
= =
−
. Viết phương trình mặt phẳng (P) đi qua
M song song vớitrục Ox, sao cho (P) cắt hai đường thẳng d1, d2 lần lượt tại A, B thỏa mãn AB = 1.
Câu 7(1,0 điểm).Trong mặt phẳng với hệ trục tọa độ Oxy, cho hình thoi ABCD có góc  0
60BAD = .
Trên cạnh AB, BC lấy các điểm M, N sao cho MB + NB = AB. Biết ( )3 ;1P thuộc đường thẳng
DN và đường phân giác trong của góc MDN có phương trình 3 6 0x y− + = . Tìm tọa độ đỉnh
D của hình thoi đã cho.
Câu 8(1,0 điểm).Giải hệ phương trình:
2 2
2
1
3 2
2
3 6 2 2 3 10 2 2 1 4 6 4 2
x
x x y x y
x y x y x y x y
+
+ − = + +

 − − + + = − + + − +
Câu 9 (1,0 điểm).Cho x, y là hai số thực dương thỏa mãn điều kiện : ( )2 2 2 2
2 x y x y+ = . Tìm giá trị nhỏ nhất
của biểu thức :
2 2
1
1 1 1
x y
P
y x x y
= + +
+ + + +
--- Hết ---
GIẢI ĐỀ SỐ 1
Câu 1(2,0 điểm). Cho hàm số
2 1
1
x
y
x
+
=
+
có đồ thị (C).
2.Viết phương trình tiếp tuyến của đồ thị (C) biết tiếp tuyến đó cách đều hai điểm A(2; 4) và B(-4; -
2).
Phương pháp chung :
Viết phương trình tiếp tuyến với (C): ( )y f x= thỏa mãn tính chất P.
• Gọi M(xo; 0( )f x ) , 0 fx D∀ ∈ là tiếp điểm của tiếp tuyến cần tìm ⇒ hệ số góc của tiếp tuyến là
0'( )k f x=
• Phương trình tiếp tuyến cần tìm có dạng : 0 0 0: '( )( ) ( )d y f x x x f x= − + {tới đây ta chỉ cần tìm 0x }
• Phân Tích tính chất P.
o Nếu tính chất P liên quan đến hệ số góc {song song đường thẳng cho trước hoặc
vuông góc đường thẳng cho trước hoặc tạo với đường thẳng cho trước một góc cụ
thể hoặc tạo với hệ trục Oxy thành tam giác có tính chất về tỷ số …} Thì ta sử dụng
các công thức ở trên để tìm hệ số góc tiếp tuyến sau đó giải phương trình 0'( )k f x= để
tìm 0x
o Nếu tính chất P là tiếp tuyến kẻ từ hoặc đi qua ( )1 1;A x y thì ta có
( )1 0 1 0 0'( ) ( )A d y f x x x f x∈ ⇔= − + {với 1 1,x y cho trước} từ đó suy ra 0x
o Nếu tính chất P liên quan đến khoảng cách , góc, diện tích, … thì ta sử dụng công thức
tương ứng để lập phương trình theo biến 0x sau đó giải phương trình để tìm 0x
Các chú ý về hệ số góc của đường thẳng :
d: y = ax + b ⇒ hệ số góc của d là a = tan (d,chiều dươngOx)
ĐB :d : x = a ⇒ d không có hệ số góc
Cho d1 : y = a1x + b1 và d2 : y = a2x + b2 khi đó ta có
• { }1 2 1 2 1 2/ /d d a a b b⇔ = ≠
• 1 2 1 2. 1d d a a⊥ ⇔ =−
• ( ) 1 2
1 2
1 2
tan ,
1
a a
d d
a a
−
=
+
• d1 tạo với trục Ox một góc α ⇒ αtan1 ±=a
Nếu
{ }
{ }
d Ox A
d Oy B
∩ =

∩ =
Thì d
OB
K
OA
= { dK là hệ số góc của đường thẳng d}
Cụ thể : Nếu . 0A Bx y < thì d
OB
K
OA
= Nếu . 0A Bx y > thì d
OB
K
OA
= −
Bài giải :
Gọi 0
0
0
2 1
;
1
x
M x
x
 +
 
+ 
(với 0 1x ≠ − ) là tiếp điểm của tiếp tuyến d cần tìm
2 20
0 0 0 02
0 0
2 11
: ( ) : ( 1) 2 2 1 0
( 1) 1
x
d y x x d x x y x x
x x
+
= − + ⇔ − + + + +=
+ +
d cách đều hai điểm A, B
( )
2 2 2 2
0 0 0 0 0 0
4 4
0 0
2 ( 1) 4 2 2 1 4 ( 1) ( 2) 2 2 1
, ( , )
1 ( 1) 1 ( 1)
x x x x x x
d A d d B d
x x
− + + + + − − + − + + +
= = =
+ + + +
2
0 0 0 02 2
0 0 0 0 2
0 0 0
2 0 0, 2
2 6 1 4 6 1
1 0 1, 1( )
x x x x
x x x x
x x x l
 + =⇔ = =−
⇔ − − − = + − ⇔ 
− = ⇔ = =−
0 0 : 1x d y x=⇒ =+ 0 2 : 5x d y x=− ⇒ = + 0
1 5
1 :
4 4
x d y x=⇒ = +
Vậy có ba tiếp tuyến thỏa yêu cầu bài toán: : 1d y x= + , : 5d y x= + và
1 5
:
4 4
d y x= +
Câu 2(1,0 điểm):
1. Giải phương trình sau: 2cos6 2cos4 3cos2 sin 2 3x x x x+ − = +
Dấu hiệu cần nhớ :
Nếu phương trình lượng giác có chứa 3 thì thông thường ta sẽ xét hai trường hợp:
• TH1: Nếu phương trình có chứa bật cao và dạng tích thì ta sử dụng công thức biến đổi
tích thành tổng và công thức hạ bật để quy về dạng phương trình bật một theo sin, cos.
• TH2: Nếu phương trình không có bật cao và dạng tích thì ta sử dụng công thức biến đổi
tổng thành tích và công thức nhân đôi để biến đổi phương trình về dạng phương trình
tích.
Nếu phương trình lượng giác có chứa hai số hạng đồng thới thỏa mãn điều kiện: cùng hệ số,
cùng hàm(sin hoặc cos), cùng tính chẳn hoặc lẻ của cung thì ta sử dụng công thức biến đổi
tổng thành tích để quy phương trình về dạng phương trình tích
Bài giải :
( ) ( )2
4cos5 cos 3 2cos 1 2sin cos 3 2cos 2cos5 3 cos sin 0pt x x x x x x x x x⇔ − −= + ⇔ − −=
cos 0
2
3 cos sin 2cos5 (1)
x x k
x x x
π
π

= ⇔ = +⇔

+ =
3 1
(1) cos sin cos5
2 2
x x x⇔ + = (chia hai vế cho
2
2 2 2
3 1 2a b+ = + = )
5 2
6 24 2
cos cos sin sin cos5 cos5 cos
6 6 6
5 2
6 36 3
x x k x k
x x x x x
x x k x k
π π π
π
π π π
π π π
π

=− + ⇔ =− + 
⇔ + = ⇔ = − ⇔  
   =− + + ⇔ = +

Vậy phương trình có ba nghiệm:
2
x k
π
π= + ,
24 2
x k
π π
=− + và
36 3
x k
π π
= + với k Z∈
2. Giải phương trình : 3 3 33log 2 log log2 3
.2 12x x
x x x+ = +
Dấu hiệu cần nhớ: Nếu phương trình lôga chứa số hạng có dạng: logb x
a thì ta đặt
log t
bt x x b= ⇔= để chuyển phương trình về dạng phương trình mũ cơ bản
Bài giải :
ĐK: 0x > . Đặt : 3log 3t
t x x= ⇔=
( )
3
3
3 3 3
log 23log 2 log 8 log 8
3 3 3 8
t
tt t
x= = = = .
( ) ( )
2 3
8 3 .2 12 3 8 18 12 27t t t t t t t t t
pt ⇔ + = + ⇔ + = +
18 12 27
1
8 8 8
t t t
     
⇔ + = +     
     
3 2 2
3 3 3 3 3 3
1 0 1 1 0 1 0 1
2 2 2 2 2 2
t t t t t t
t x
              
⇔ − + − = ⇔ − + = ⇔ = ⇔ = ⇔ =              
                 
Vậy phương trình có nghiệm duy nhất x = 1.
Câu 3 (1,0điểm).
1. Tính tích phân: 4
20
sin
5sinx.cos x + 2cosx
x
I dx
π
= ∫
Dấu hiệu cần nhớ: Nếu tích phân luong giác dạng phân số không căn và tất cả các số hạng cùng
loại bậc chẳn hoặc cùng bật lẻ thì ta chia tử và mẫu cho cosn
x để quy về dạng :
2
1
(tan ).
cos
f x dx
x∫ sau đó đặt tant x= hoặc chiatử và mẫu cho sinn
x để quy về dạng :
2
1
(cot ).
sin
f x dx
x∫ sau đó đặt cott x= (với n là bậc cao nhất)
Bài giải :
3
4 4
2 2 20 0
3
sin
tan 1cos
5sinx.cos x + 2cosx 2tan 5tan 2 cos
cos
x
xxI dx dx
x x x
x
π π
=
+ +∫ ∫
Đặt : 2
1
tan
cos
t x dt dx
x
= ⇒ = Đổi cận:
1
4
0 0
x t
x t
π
= ⇒ =

 = ⇒ =
1 1
20 0
1 2(2 1) ( 2)
2 5 2 3 (2 1)( 2)
t t t
I dt dt
t t t t
+ − +
= =
+ + + +∫ ∫
1
0
11 2 1 2 1 1 2
ln 2 ln 2 1 ln3 ln 2
03 2 2 1 3 6 2 3
dt t t
t t
 
= − = + − + = − 
+ +  
∫
2. Cho hình phẳng (H) được giới hạn bởi đồ thị của các hàm số sin3 , 0 , 0y x y x= = = và
6
x
π
= .
Tính thể tích khối tròn xoay sinh bởi (H) khi quay quanh trục Ox.
Bài giải :
( ) ( )
6 6
2
0 0
1
sin3 1 cos6 sin 6 ( )6
2 2 6 12
0
V x dx x dx x x dvtt
π π
π
π π π
π
 
= = − =− = 
 
∫ ∫
Câu 4 (1,0 điểm).
1. Tìm tập hợp các điểm biểu diển của số phức z thỏa mãn đẳng thức sau : 2 1 1z i iz i− + = + − .
trong các số phức thỏa điều kiện trên , hãy tìm số phức z có môđun nhỏ nhất.
Bài giải :
Gọi z x yi= + với ,x y R∈ , z có điểm biểu diển là M(x ; y).
( ) ( ) ( ) ( ) ( ) ( ) ( ) ( )
2 2 2 2
1 2 1 1 1 2 1 1 2 1 0gt x y i y x i x y y x y⇔ + + − = − − + + ⇔ + + − = − − + + ⇔ − =
Vậy tập hợp điểm biểu diển của z là đường thẳng có phương trình là 2 1 0y − =
21 1 1 1
0
2 4 2 2
z x i z x Min z khi x= + ⇒ = + ≥ ⇒ = =
Số phức có môđun nhỏ nhất thỏa yêu cầu bài toán là
1
2
z i=
2. Chọn ngẫu nhiên hai số hạng trong khai triển nhị thức Newton ( )
100
6
3 5− . Tính xác suất để hai
số được chọn có một số là số hữu tỷ và một số là số vô tỷ.
Bài giải :
Ta có khai triển: ( ) ( ) ( )
100100 100
6 6
100
0
3 5 ( 1) 3 5
k k
k k
k
C
−
=
− = −∑
Số hạng tổng quát trong khai triển là:
100
62
1 100( 1) 3 5
kk
k k
kT C
−
+ = −
Vì 3 và 5 là các số nguyên tố nên 1kT + là số hữu tỷ thì
0 100
6
k
k
≤ ≤
⇒
 
có 17 số k thỏa điều kiện
Vậy trong khai triển ta có 17 số hữu tỷ và 84 số vô tỷ.
Gọi Ω là không gian mẫu của phép thử chọn ngẫu nhiên 2 số hạng trong 101 số hạng của khai
triển:
⇒số phần tử củaΩ là 2
101 5050CΩ= =
Gọi A là biến cố hai số hạng chọn ra có một số vô tỷ và một số hữu tỷ:
• Chọn một số hữu tỷ từ 17 số hữu tỷ trong khai triển⇒ có 1
17 17C = cách chọn
• Chọn một số vô tỷ từ 84 số vô tỷ trong khai triển⇒ có 1
84 84C = cách chọn
Theo quy tắc nhân ta có số phần tử của không gian biến A là 17 84 1428AΩ = × =
Vậy xác suất của biến cố A là
1428
( ) ...
5050
A
P A
Ω
= = =
Ω
Câu 5(1,0 điểm).Cho hình chóp SABCD có đáy ABCD là hình vuông, tam giác SAB vuông góc tại S và
nằm trong mặt phẳng vuông góc với đáy (ABCD). Biết khoảng cách giữa hai đường thẳng SC
và AB bằng
2 57
19
a
; góc tạo bới SB và mặt phẳng (ABCD) bằng 300
. Tính thể tích của khối
chóp SABCD và diện tích mặt cầu ngoại tiếp tứ diện SBCD.
Bài giải :
Gọi H là hình chiếu của S lên AB mặt khác ta có
(SAB) và (ABCD) vuông góc nhau theo giao tuyến AB ( )SH ABCD⇒ ⊥
S
A D
K
Ta có BH là hình chiếu của BS lên mặt phẳng (ABCD)
( ) ( ) ,( ) ,SB ABCD BS BH SBH= = ,  0
30gt SBH⇒ =
Ta có : ( )/ / / / ( , ) ( ,( ))AB CD AB SCD d AB SC d H SCD⇒ ⇒ =
Dựng HM CD⊥ (với M CD∈ )
( )CD SHM⇒ ⊥ ( vì SH CD⊥ )
⇒ (SCD), (SHM) vuông góc nhau theo giao tuyến SM
Dựng HK SM⊥ (với K SM∈ ) ( )HK SCD⇒ ⊥
⇒ K là hình chiếu của H lên (SCD) ( , )d H SCD HK⇒ =,
2 57
19
a
gt HK⇒ =
Đặt: cạnh của hình vuông là b khi đó ta có HM = b.
Xét ABS∆ ta có:  0 3
cos cos30
2
SB b
SBA SB
AB
= = ⇒ =
Xét SHB∆ ta có:  0 3
sin sin30
4
SH b
SBH SH
SB
= = ⇒ =
Xét SHM∆ ta có: 2 2 2 2 2 2 2
1 1 1 19 1 16 19
2
12 3 3
b a
HK HS HM a b b b
= + ⇔ = + = ⇔ =
3
2
SH a
AB a
 =
⇒ 
=
3
21 1 4 3
. .
3 3 3
SABCD ABCD
a
V SH S SH AB= = = (đvtt)
Gọi I là tâm của ABCD, khi đó ta có: IB = IC = ID (1)
Ta có :
BC AB
BC SA
BC SH
⊥
⇒ ⊥
⊥
mà SA SB⊥ ( )SA SBC⇒ ⊥ SA SC SAC⇒ ⊥ ⇒ ∆ vuông tại S (2)
IS IC=
Từ (1) và (2) ta có I là tâm mật (S) cầu ngoại tiếp SBCD⇒bán kính mặt cầu (S) là
1
2
2
R IA AC a= = =
Vậy diện tích mặt cầu ngoại tiếp hình chóp SBCD là: 2 2
( ) 4 8SS R aπ π= = (đvdt)
Câu 6(1,0điểm).Trong không gian với hệ trục tọa độ Oxyz, cho điểm M(0; 2; 0) và hai đường
thẳng 1
1 2 1
:
2 2 1
x y z
d
− − +
= =
−
và 2
3 1
:
2 2 1
x y z
d
− +
= =
−
. Viết phương trình mặt phẳng (P) đi qua
M song song vớitrục Ox, sao cho (P) cắt hai đường thẳng d1, d2 lần lượt tại A, B thỏa mãn AB = 1.
Bài giải :
Ta có: 1
1 2
: 2 2
1
x t
d y t
z t
= +

= −
 =− +
và 2
3 2
: 1 2
x m
d y m
z m
= +

=− −
 =
( )1( ) 1 2 ; 2 2 ; 1 ( )A P d A t t t P= ∩ ⇒ + − − + ∈ , ( )2( ) 3 2 ; 1 2 ; ( )B P d B m m m P= ∩ ⇒ + − − ∈
( )2 2 2; 2 2 3; 1AB m t m t m t= − + − + − − +

, ( )1 2 ; 2 ; 1AM t t t= − − − +

( ) ( ) ( ) ( ) ( )
2 2 2 2
1
1 2 2 2 2 2 3 1 1 9 22 13 0 13
9
m t
AB m t m t m t m t m t
m t
− =−
= ⇔ − + + − + − + − + = ⇔ − + − + = ⇔
 − =−

Với 1m t− =− 1m t⇒ = − ( )0; 1; 0AB⇒ = −

( ), 1; 0; 2 1pn AM AB t t = = − + + 
  
là VTPT của (P).
( )/ /( ) . 0 1 0 1 0; 0; 3Ox p Ox p pOx P u n u n t t n⇒ ⊥ ⇔ = ⇔ − + = ⇔ = ⇒ =
    
(với Oxu

là VTCP của Ox)
(P) qua M(0; 2; 0) và có VTPT là ( )0; 0; 3pn =

( ) : 0P y⇒ =(loại vì (P) chứa Ox)
Với
13
9
m t− =−
8 1 4
; ;
9 9 9
AB
 
⇒ =− − − 
 

9 1 12 18 1
, ; ;
9 9 9
p
t t
n AM AB
− + +  = = −    
  
là VTPT của (P).
1 12 3
/ /( ) . 0 9 1 0 0; ;
9 9 9
Ox p Ox p pOx P u n u n t t n
 
⇒ ⊥ ⇔ = ⇔ − + = ⇔ = ⇒ = − 
 
    
(với Oxu

là VTCP của
Ox)
(P) qua M(0; 2; 0) và có VTPT là
12 3
0; ;
9 9
pn
 
= − 
 

( ) : 4 8 0P y z⇒ − − =.
Vậy mặt phẳng cần tìm là: ( ) : 4 8 0P y z− − =
Câu 7(1,0 điểm).Trong mặt phẳng với hệ trục tọa độ Oxy, cho hình thoi ABCD có góc  0
60BAD = .
Trên cạnh AB, BC lấy các điểm M, N sao cho MB + NB = AB. Biết ( )3 ;1P thuộc đường thẳng
DN và đường phân giác trong của góc MDN có phương trình 3 6 0x y− + = . Tìm tọa độ đỉnh
D của hình thoi đã cho. B : 3 6 0d x y− + =
H
P’ N
M
Bài giải :
( )3 6 ;D d D m m∈ ⇒ −
MB + NB = AB
NC MB CDN CDM
NB MA NDB MDA
= ∆ =∆ 
⇒ ⇒ 
= ∆ =∆ 
 
 
 0
60
NDC MDB
MDN
NDB MDB
 =
⇒ ⇒ =
=
( Vì  0
120ADC = )
Gọi P’ là điểm đối xứng với P qua d 'P DM⇒ ∈
'DPP⇒ ∆ là tam giác đều có d là đường phân giác 2 ( , )DP d P d⇒ =
( ) ( )
2 2
2
2
3 3 6 1
3 6 3 1 2
3 3 11 3
m
m m
m
− + =
⇔ − − + −= ⇔ 
= ++
Vậy có hai điểm cần tìm ( )3 6 ;1D − và ( )3 3 ; 3 3 1D + +
Câu 8(1,0 điểm).Giải hệ phương trình:
2 2
2
1
3 2 (1)
2
3 6 2 2 3 10 2 2 1 4 6 4 2 (2)
x
x x y x y
x y x y x y x y
+
+ − = + +

 − − + + = − + + − +
Bình luận:
Vì phương trình (2) quá phức tạp và không có dấu hiệu để biến đổi nên ta sẽ tập trung vào (1).
Ta sẽ biến đổi pt(1) về dạng tích bằng phương pháp nhân liên hợp,trước tiên ta phải đoán được nhóm
chung cần tìm là gì:
2 2
(1) 2 3 2 2 1x x y x y x⇔ + − = + + + . Ta sẽ tìm nhóm chung bằng máy tính vớikỹ thuật sau:
Xét 1x = ta có: 2
2 4 2 2 1 2 0y y y− = + + ⇒ =
Xét x = 2 ta có: 2 1
2 10 2 2 2 3
2
y y y− = + + ⇒ =
Nếu nhóm chung bật một ax by c+ + thì ta có :
0
2 11
22 0
2
a c
c a
x y
b aa b c
+ =
= −
⇔ ⇒ − − 
= −+ + = 
là nhóm chung.
Đến đây ta sẽ đại lượng cần thêm vào các nhóm phần tử:
( ) ( ) ( )
2 22 2 2 2
3 2 2 1 3 2 2 1 1 1x x y A x y A x x y x y x A x+ − − = − − ⇔ = + − − − − = + ⇒ = +
Vậy đại lượng cần thêm vào 2
3 2x x y+ − chình là 1x +
Bài giải :
ĐK:
( )
( )
2 2
2 2 2
2 2 2
2 1 4 2
(1) 3 2 1 3 2 2 0 0
3 2 1 3 2 2
x y x y x y
x x y x x x y x y
x x y x x x y x y
− − − − −
⇔ + − − + + + − − + = ⇔ + =
+ − + + + − + +
( )
( )2 2 2
1 2
2 1 0 2 1 0
3 2 1 3 2 2
x y
x y x y
x x y x x x y x y
 +
 ⇔ − − + = ⇔ − − =
 + − + + + − + + 
(vì
2 0x y+ ≥ )
Thay 2 1y x= − vào phương trình (2) ta có: ( )2
3 6 1 8 5 2 1 4 3 1 3x x x x x− − + −= − + +
Bình luận :
Đến đây ta dễ dàng thấy được 2
6 1 2 1. 3 1x x x x− −= − + Điều này khiến ta nghĩ đến đến hai phép
đặt ẩn phụ cơ bản: 2 1 4 3 1t x x= − + + hoặc
2 1
3 1
u x
v x
= −

= +
để quy về pt bậc 2 theo 2 biến. Nhưng ta lại
thấy phép đặt 2 1 4 3 1t x x= − + + không phù hợp với bài này vì không thể khử hết biến x trong
phương trình
Đặt :
2 1 0
3 1 0
u x
v x
= − ≥

= + ≥
. 2
6 1x x uv− − = , 2 2
8 5 2 6x u v− = + −
( )2 2 2 2
(3) 3 2 6 4 3 1 2 4 6 0uv u v u v u v u v v⇔ + + − = + ⇔ + − + − − =
( ) ( ) ( )
2 22
3 1 4 2 4 6 5u v v v v∆ = − − − − = +
( ) ( )
( ) ( )
3 1 5
2 2 ( 0, 0)
2
(3)
3 1 5
3
2
v v
u v loai vi u v
v v
u v
− − − +
= =− − ≥ ≥
⇔
− − + +
= =− +

2
9 5 0
3 2 1 3 1 3 2 (2 1)(3 1) 9 5 1
86 85 0
x
u v x x x x x x
x x
− ≥
⇔ + = ⇔ − + + = ⇔ − + = − ⇔ ⇔ =
− + =
Vậy ( ; ) (1;0)x y = là nghiệm duy nhất của phương trình.
Bài tập tương tự:
1.
( )
( )
2
3
2 2
2 3 2 1 11
y
x x y
x y
x y x

− + = −
 + − − =
2.
( ) ( )
2
1 2 1
2 3 6 1 2 2 4 5 3
y x y x x y y
y x y x y x y
 − − + = + − −

− + += − − − −
3.
2
16( ) 2
13 ( ) 3 5
x y x y xy
x y y
 + − + =

− + − − =
Câu 9 (1,0 điểm).Cho x, y là hai số thực dương thỏa mãn điều kiện : ( )2 2 2 2
2 x y x y+ = . Tìm giá trị nhỏ nhất
của biểu thức :
2 2
1
1 1 1
x y
P
y x x y
= + +
+ + + +
Bài giải :
Ta có: ( ) ( )
22 2 2 2
2x y x y x y xy x y= + ≥ + ⇒ ≥ +
Do đó: ( ) ( ) ( ) ( )
2 2 22 2
1 2 1 2 1 1x y x y xy x y x y x y+ + = + − + ≤ + − + + = + −
2 2
2 2
1 1
1 1
11
x y x y
x yx y
⇒ + + ≤ + − ⇒ ≥
+ −+ +
( ) ( )
1 1 4
1 1 2 1 2 1 2
1 1 1 1 1 1 2
x y x y
x y x y
y x y x x y x y
 
+ = + + + − = + + + − ≥ + + − 
+ + + + + + + + 
Vậy ( )
4 1
1 2
2 1
P x y
x y x y
≥ + + + −
+ + + −
Đặt: .t x y= + ta có : 2
( ) 4 4( ) 4 4a b xy x y x y t+ ≥ ≥ + ⇒ + ≥ ⇒ ≥
Từ đó ta được : [ )
4 4 1
2 ( ) , 4 ,
2 1
t
P f t t
t t
+
≥ + −= ∀ ∈ + ∞
+ −
Ta có:
( )
( )
( ) ( )
[ )2 2 22
3 44 1
'( ) 0 , 4 ,
( 1)2 2 1
t t
f t t
tt t t
−
= − = ≥ ∀ ∈ + ∞
−+ + −
Do đó ( )f t là hàm liên tục và đơn điệu tăng trên [ )4 , + ∞
[ )
[ ]4 ,
5 5
( ) (4) ( )
3 3
f t f Min f x
+∞
⇒ ≥ ≥ ⇒ =khi 4t =
Vậy ( )
5
3
Min P = khi 2x y= =
ĐỀ SỐ 2
Câu 1(2,0điểm).Cho hàm số
2 1
2
x
y
x
+
=
+
có đồ thị (C).
1. Khảo sát sự biến thiên và vẽ đồ thị (C) của hàm số trên.
2. Chứng minh rằng đường thẳng d: y x m=− + luôn cắt đồ thị (C) tại hai điểm phân biệt A và B.
Tìm m để đoạn AB có độ dài nhỏ nhất.
Câu 3 (1,0 điểm).
1. Giải phương trình:2sin 2 2 sin 2 5sin 3cos 3
4
x x x x
π 
+ + + − = 
 
2. Giải phương trình: ( )2
32
4 8log ( 1) 2 log 4 log 4x x x+ += − + +
Câu 3 (1,0 điểm).
1. Tính tích phân:
( ) ( )
2
1
30
1 12 16 4
1 3 1
x x
I dx
x x
− + +
=
+ +
∫
2. Tính diện tích hình phẳng được giới hạn bởi các đường sau :
2
2
4 ,
3
x
y x y= − =
Câu 4 (1,0 điểm).
1. Cho hai số phức 1z và 2z thỏa mãn điều kiện 1 22, 5z z= = và 1 2 4z z− =. Hãy tính mô đun của
số phức 1 2z z z= + .
2. Tổ một có 3 học sinh nam và 4 học sinh nữ. Tổ hai có 5 học sinh nam và 2 học sinh nữ. Chọn ngẫu
nhiên mỗi tổ một học sinh đi làm nhiệm vụ. Tính xác suất để chọn được hai học sinh gồm một nam
và một nữ.
Câu 5 (1,0 điểm).Cho hình chóp SABC, có cạnh SA vuông góc với đáy và AB = a, AC = 2a,
 0
120BAC = . Mặt phẳng (SBC) tạo với đáy một góc 600
. Tính thể tích của khối chóp SABC và
khoảng cách giữa hai đường thẳng SB và AC theo a .
Câu 6 (1,0 điểm).Trong không gian với hệ trục tọa độ Oxy z, cho hai mặt phẳng (P): 2x + z – 3 = 0 và
(Q): x + 2y + 3z – 4 = 0. Viết phương trình mặt phẳng vuông góc với hai mặt phẳng (P) và (Q),
đồng thời hợp với các mặt phẳng tọa độ thành khối tứ diện có thể tích bằng
4
15
.
Câu 7 (1,0 điểm).Trong mặt phẳng với hệ trục tọa độ Oxy, cho hình bình hành ABCD có phương
trình
đường chéo AC: 5x + y + 4 = 0. Gọi
23 15
;
7 7
H
 
− 
 
là
trực tâm của tam giác ABC và
2
;4
3
G
 
− 
 
là trọng tâm tam giác ACD . Tìm tọa độ các đỉnh của hình bình hành A, B, C, D.
Câu 8 (1,0 điểm). Giải hệ phương trình:
( )( ) 2 3
32 3
1 3 3 ( 1)
2 4 2( 2)
y x y x y x
x y x y
 − − + − = −

 − + − = −
Câu 9 (1,0 điểm).Tìm tất cả các số thực m sao cho hệ phương trình sau có 4 nghiệm thực phân biệt.
( ) 2
2 2
2.3 .2 7.2
( 4) 2 3 5 8 32
x y x y x y
x y
m x y y x y
− − + −
 + − =

+ + + = + +
---- Hết ---
GIẢI ĐỀ SỐ 2
Câu 1(2,0điểm).Cho hàm số
2 1
2
x
y
x
+
=
+
có đồ thị (C).
Chứng minh rằng đường thẳng d: y x m=− + luôn cắt đồ thị (C) tại hai điểm phân biệt A và B.
Tìm m để đoạn AB có độ dài nhỏ nhất.
Bài giải:
Phương trình hoành độ giao điểm của (C) và d là
2 1
(1)
2
x
x m
x
+
=− +
+
ĐK: 2x ≠ −
2
(1) ( ) (4 ) 2 1 0 (2)f x x m x m⇔ = + − − + =
Ta có :
( )
2 2
2
4 4( 2 1) 12 0 ,
( 2) ( 2) (4 )( 2) 2 1 0 ,
m m m m R
f m m m R
∆= − − − + = + > ∀ ∈
⇔
− = − + − − − + ≠ ∀ ∈
(2) có hai nghiệm phân biệt khác -2
⇔ (1) có hai nghiệm phân biệt , m R∀ ∈ ⇔ d cắt (C) tại hai điểm A, B phân biệt ( )dfcm .
Gọi 1 2,x x là hai nghiệm phân biệt của (2), theo talet ta có: 1 2 1 24 , . 2 1x x m x x m+ =− =− +
Khi đó ta có : ( )1 1;A x x m− + và ( )2 2;B x x m− + là hai giao điểm
( ) [ ] ( ) ( )
22 2 2
1 2 1 2 1 2 1 2 1 2( ) ( ) 2 2 4AB x x x m x m x x x x x x= − + − + − − + = − = + −
( ) ( )
2 2
2 4 4 2 1 2 12 24m m m= − − − += + ≥
Vậy ( ) 24Min AB = khi m = 0
Câu 3 (1,0 điểm).
1. Giải phương trình:2sin 2 2 sin 2 5sin 3cos 3
4
x x x x
π 
+ + + − = 
 
Bài giải: 2sin 2 (sin 2 cos2 ) 5sin 3cos 3 0pt x x x x x⇔ + + + − − =
( )
( ) ( )
( ) ( )( )
( )( )
2
2
6sin cos 1 2sin 5sin 3cos 3 0
3cos 2sin 1 2sin 5sin 2 0
3cos 2sin 1 sin 2 2sin 1 0
2sin 1 3cos sin 2 0
2sin 1 0 (1)
sin 3cos 2 (2)
x x x x x
x x x x
x x x x
x x x
x
x x
⇔ + − + − − =
⇔ − − − + =
⇔ − − − − =
⇔ − − + =
− =
⇔  − =
2
1 6
(1) sin
52
2
6
x k
x
x k
π
π
π
π

= +
⇔ =⇔ 
= +

1 3 2 2
(2) sin cos cos cos sin sin
10 10 10 10
x x x xα α⇔ + = ⇔ + =
( )cos cos 2x x kα β α β π⇔ − = ⇔ = ± +
Với:
3
cos
10
1
sin
10
α
α

=

 =

và
2
cos
10
β =
2. Giải phương trình: ( )2
32
4 8log ( 1) 2 log 4 log 4x x x+ += − + +
Bài giải: ĐK:
4 4
1
x
x
− < <

≠ −
( ) ( )
( ) ( )
( ) ( )( )
( )
2 3
1
22
2
2
1
32 2
2
22 2
2 2 2
2
2
22 2
2 2
log ( 1) log 2 log 4 log 4
log 1 log 4 log 4 log 4
log 4 1 log 4 4
16 0 4 4
2
4 4 164 4 16 4 12 0
2 2 6
4 4 16 4 20 0
pt x x x
x x x
x x x
x x
x
x xx x x x
x
x x x x
⇔ + + = − + +
⇔ + + = − + +
⇔ += − +  
 − ≥ − ≤ ≤
 = + = − ⇔ + = − ⇔ ⇔ ⇔+ − =  
 = −  + =− − − − = 
Chú ý : Trong bài này sai lầm thường mắt phải là: ( ) ( )2
2
22
log 1 log 1x x+ = + . Cần nhớ 2
log 2 logn
a ab n b=
Câu 3 (1,0 điểm).
1. Tính tích phân:
( ) ( )
2
1
30
1 12 16 4
1 3 1
x x
I dx
x x
− + +
=
+ +
∫
Bài giải:
( )( )
( ) ( )( ) ( ) ( )( )
1 1 1
1 20 0 0
1 4 1 3 1 1 1
4 4
11 1 3 1 1 1 3 1
x x
I dx dx dx I I
xx x x x x x
− + +
= = − =−
++ + + + + +
∫ ∫ ∫
1
1
0
11
( 1) ln 1 ln 2
01
I d x x
x
= + = + =
+∫
( )
1
2 0
2
1
3 1
1
1
I dx
x
x
x
=
+
+
+
∫
Đặt:
( )
2
2
3 1 3 1 1
1 1 1
x x
t t tdt dx
x x x
+ +
= ⇔ = ⇒ =
+ + +
Đổi cận
1 2
0 1
x t
x t
 = ⇒ =

= ⇒ =
2 2
2
1 1
1 2
2 1
1
I tdt dt t
t
= = = = −∫ ∫
Vậy : 2 1 4ln 2I= − −
2. Tính diện tích hình phẳng được giới hạn bởi các đường sau :
2
2
4 ,
3
x
y x y= − =
Bài giải: Phương trình hoành độ giao điểm :
2
2 2 2 4
4 3 4 9 36 0 3
3
x
x x x x x x− = ⇔ − = ⇔ + − =⇔ =±
3 2
2
3
4
3
x
S x dx
−
= − −∫
2
2
2 2
2
4 1
3 , 3 0 3 4 0
31
3
x
x
x x xx
 − ≥
 ∀ ∈ − ⇔ ≤ ≤ ⇒ ⇒ − − ≥ 
≤

3 3 32 2 3
2 2
1 1
3 3 3
3 2 3
4 4
3 3 9 33
x x x
S x dx x dx dx I I
− − −
   
= − − = − − = − = −   
−   
∫ ∫ ∫
Đặt : 2sin 2cosx t dx tdt= ⇒ =
Đổi cận:
3
3
3
3
x t
x t
π
π

=− ⇒ =−

 = ⇒ =

( ) ( )
3 3 3
2 2
1
3 3 3
1 43
4 1 sin 2cos 4 cos 2 1 cos2 2 sin 2 3
2 3
3
I t tdt tdt t dt t t
π π π
π π π
π
π
π
− − −
 
= − = = + =+ =+ 
 
−
∫ ∫ ∫
Vậy diện tích hình phẳng cần tìm là :
4 3
3 3
S
π
= + (đvdt)
Câu 4 (1,0 điểm).
1. Cho hai số phức 1z và 2z thỏa mãn điều kiện 1 22, 5z z= = và 1 2 4z z− =. Hãy tính mô đun của
số phức 1 2z z z= + .
Bài giải:
Gọi 1 1 1z x y i= + và 2 2 2z x y i= + với 1 2 1 2, , ,x x y y R∈
Khi đó ta có :
( ) ( )
2 2 2 2 2 2
1 1 1 1 1 1 1
2 2 2 2 2 2
2 2 2 2 2 2 2
2 2 2 2 2 2
1 2 1 21 1 2 2 1 2 1 21 2 1 2 1 2
2 4 4 4
5 25 25 25
2 2 1316 2 24 16
z x y x y x y
z x y x y x y
x x y yx y x y x x y yz z x x y y
   = += += +=
   
= ⇔ + = ⇔ + = ⇔ + =   
    + =+ + + = + +− = − + − =  
( ) ( )
2 2 2 2 2 2
1 2 1 2 1 2 1 1 2 2 1 2 1 22 2 4 25 13 42z z z x x y y x y x y x x y y= + = + + + = + + + + + = + + =
2. Tổ một có 3 học sinh nam và 4 học sinh nữ. Tổ hai có 5 học sinh nam và 2 học sinh nữ. Chọn ngẫu
nhiên mỗi tổ một học sinh đi làm nhiệm vụ. Tính xác suất để chọn được hai học sinh gồm một nam
và một nữ.
Bài giải:
Gọi Ω là không gian mẫu của phép thử chọn ngẫu nhiên mỗi tổ một học sinh
Khi đó ta có số phần tử của không gian mẫu là : 1 1
7 7 49C CΩ= × =
Gọi A là biến cố hai học sinh được chọn là một học sinh nam và một học sinh nữ.
Khi đó ta có số phần tử của không gian biến cố A được tính như sau:
TH1: Chọn 1 học sinh nam từ tổ một và 1 học sinh nữ từ tổ hai ⇒có 1 1
3 2 6C C× =cách chọn
TH1: Chọn 1 học sinh nữ từ tổ một và 1 học sinh nam từ tổ hai ⇒có 1 1
4 5 20C C× = cách chọn
Theo quy tắc cộng ta có: 6 20 26AΩ = + =
Vậy xác suất cần tìm là :
26
( )
49
A
P A
Ω
= =
Ω
Câu 5 (1,0 điểm).Cho hình chóp SABC, có cạnh SA vuông góc với đáy và AB = a, AC = 2a,
 0
120BAC = . Mặt phẳng (SBC) tạo với đáy một góc 600
. Tính thể tích của khối chóp SABC và
khoảng cách giữa hai đường thẳng SB và AC theo a .
Bài giải:
Dựng AM ⊥ BC ( với M BC∈ )
⇒BC ⊥ (SAM) (vì SA BC⊥ )
( ) ( ) ( ),( ) ,SBC ABC MA MS SMA= = (Vì  0
90SAM = )
Từ  0
60gt SMA⇒ =
Theo định lý cosin cho tam giác ABC ta có:
C
S
A
0
120 0
60
2 2
2 . .cos 7BC AB AC AB BC BAC a= + − =
1 1
. .sin .
2 2
ABCS AB AC BAC AM BC= =
. .sin 21
7
AB AC BAC a
AM
BC
⇔= =
Xét SAM∆ ta có:   7
tan tan
7
SA a
SMA SA AM SMA
AM
= ⇔ = =

3
1 1 1 21
. . . .sin
3 3 2 42
SABC ABC
a
V SA S SA AB AC BAC= = = (đvdt)
Dựng hình bình hành ACBD như hình vẽ .
Khi đó ta có : AC // BC ⇒ AC// (SBC)
3
( , ) ( ,( ) (1)SABD
SBD
V
d AC SB d A SBD
S
⇒ = =
Ta có:
3
21
42
SABD SABC
a
V V= =
Theo Pitago ta có: 2 2 5 14
7
a
SD SA AD= + = , 2 2 2 14
7
a
SB SA AB= + =
Theo định lý cosin cho SBD∆ ta có: 
( )
2 2 2
3
cos
2. . 4
SB SD BD
BSD
SB SD
+ −
= =  2 7
sin 1 cos
4
BSD BSD⇒ =− =

( )
2
1 5 7
. .sin
2 14
SBD
a
S SB SD BSD= =
3
2
21
3
342(1) ( , )
55 7
14
a
a
d AC SB
a
⇔ = = (đvđd)
Câu 6 (1,0 điểm).Trong không gian với hệ trục tọa độ Oxy z, cho hai mặt phẳng (P): 2x + z – 3 = 0 và
(Q): x + 2y + 3z – 4 = 0. Viết phương trình mặt phẳng vuông góc với hai mặt phẳng (P) và (Q),
đồng thời hợp với các mặt phẳng tọa độ thành khối tứ diện có thể tích bằng
4
15
.
Bài giải:
( )( ) : 2 3 0 2; 0;1pP x z n+ − = ⇒ =

là VTPT của mặt phẳng (P)
( )( ) : 2 3 4 0 1; 2; 3QP x y z n+ + − = ⇒ =

là VTPT của mặt phẳng (Q)
Ta có:
( ) ( )
( ) ( )
R P
R Q
⊥
⇒
⊥
mp (R) có VTPT là ( ), 2 5 4R p Qn n n = = − − + 
  
( ) : 2 5 4 0R x y y m⇒ + − + =
( ) ; 0; 0
2 2
( ) 0; ; 0
5 5
( ) 0; 0;
4 4
mm
A R Ox A OA
mm
B R Oy B OB
mm
C R Oz B OB
  
= ∩ ⇒ − ⇒ =  
 
  
= ∩ ⇒ − ⇒ =  
 
  
= ∩ ⇒ ⇒ =  
 
3 34 1 4
. . 4 4 4
15 6 15
OABCV OAOB OC m m m= ⇔ = ⇔ = ⇔ = ⇔ = ±
Vậy có hai mặt phẳng cần tìm là: ( ) : 2 5 4 4 0R x y y+ − + = và ( ) : 2 5 4 4 0R x y y+ − − =
Câu 7 (1,0 điểm).Trong mặt phẳng với hệ trục tọa độ Oxy, cho hình bình hành ABCD có phương
trình đường chéo AC: 5x + y + 4 = 0. Gọi
23 15
;
7 7
H
 
− 
 
là trực tâm của tam giác ABC và
2
;4
3
G
 
− 
 
là trọng tâm tam giác ACD . Tìm tọa độ các đỉnh của hình bình hành A, B, C, D.
Bài giải:
BH đi qua H và vuông góc AC : 5 14 0BH x y⇒ − + =
( )5 14;B BH B m m∈ ⇒ −
Gọi I là giao điểm của AC và DB ( ; 5 4)I AC I a a⇒ ∈ ⇒ − −
G là trọng tâm tam giác ACD 4BG IG⇒ =
 
( ) ( )
2 2 3 3 7
5 14 4 ;
3 3 2 2 2
4 4 4 5 4 2 4 ; 2
m a a I
m a m B
    
− − + = − − =− ⇒ −    
⇔ ⇔    
 − = + + = ⇒ − 
I là trung điểm của BD ( )
2 2 1
1; 5
2 2 5
B D I D I B
B D I D I B
x x x x x x
D
y y y y y y
+ = = − = 
⇒ ⇔ ⇒ 
+ = = − = 
Ta có ( ); 5 4A AC A a a∈ ⇒ − −
Lại có I là trung điểm của AC ( )3 ;11 5C a a⇒ − − +
( )4 ; 6 5AB a a= − − +

và
7 2 35 62
;
7 7
a a
CH
− − − 
=  
 

Ta có H là trực tâm của tam giác ABC . 0 ,AB CH a A C⇒ = ⇒ ⇒
 
Câu 8 (1,0 điểm). Giải hệ phương trình:
( )( ) 2 3
32 3
1 3 3 ( 1) (1)
2 4 2( 2) (2)
y x y x y x
x y x y
 − − + − = −

 − + − = −
A
B
C
D
. 2
;4
3
G
 
− 
 
23 15
;
7 7
H
 
− 
 
5x + y + 4 = 0
I
Bình luận:Dễ thấy phương trình (1) chỉ chứa hai nhóm biến là : x và 1y − điều này khiến ta nghĩ ngay
đến phương trình thuần nhất (đồng bậc) hoặc cấu trúc hàm số ( ) ( )1f x f y= − hoặc phương trình tích.
Từ đó để thực hiện được ý tưởng này ta nên đặt hai ẩn phụ để quy phương trình (1) về dạng đa thức trong
sáng hơn, sau đó ta mới xem thử nó cụ thể là thuộc dạng nào.
Bài giải: ĐK:
2
1
0
x y
x
 ≥ ≥

≥
Dễ thấy y = 1 không thỏa hệ phương trình. Nên ta chỉ xét 1y > . Đặt :
1 0
0
u y
v x
 = − >

= ≥
( )2 2 2 4 3 4 2 2 3 4
(1) 3 3 0u v u v u v v u v u v u⇔ − − − = ⇔ + + − =
Chia hai vế cho 4
u và đặt 0
v
t
u
= ≥ ta có
( ) ( )4 2 3 2
3 2
1 1 1
1 3 0 ( 1) 2 3 0
2 3 0 ( , 0)`
t u v y x y x
t t t t t t t
t t t VN vi t
 = ⇔ = ⇔ − = ⇔ = +
⇔ + + − = ⇔ − + + + = ⇔ 
+ + += >
Thay 1y x= + vào phương trình (2) ta có: 32 3
1 2 4 2( 1) (*)x x x x− − + − = −
Bình luận:pt (*) chứa 2 loại căn và 3
điều này khiến ta nghĩ đến các hướng phân tích sau:
Hướng 1: sử dụng hàm số để chứng minh nghiệm duy nhất (phải đoán được 1 nghiệm),
Hướng 2: phân tích về phương trình tích bằng nhân liên hợp (phải đoán được nghiệm hoặc
đoán được nhóm phần tử chung),
Hướng 3: hoặc quy về phương trình thuần nhất bậc 3. Đặt 3
t = sau đó bình phương hai vế.
Hướng 4: Đặt hai ẩn phụ để quy về phương trình bậc 3 theo hai biến hoặc quy về hệ phương
trình hai biến không căn thức.
Nhưng pt(*) không đoan nghiệm được và trong 3
là biểu thức bậc 3 theo biến x , nên ta chỉ có thể đi theo
hướng nhân liên hợp để xây dựng được nhóm chung là biểu thức bậc 2.Vấn đề bây giờ là làm sao có thể
đoán được nhóm chung này, ta thử xem :
3
3 3 3 2
4 ( 1) 3( 1)x x x x− − − = − − . Từ đây ta có lời giải:
32 3
(*) 1 2 4 ( 1) 0x x x x ⇔ − − + − − − =
 
2
2
2
3 33 3 2
3( 1)
1 2 0
4 ( 2) 4 ( 2)
x x
x x
x x x x
− −
⇔ − − + =
− + − − + −
2
2
2
3 33 3 2
6 1
1 1 0
4 ( 2) 4 ( 2)
x x
x x
x x x x
 − −
 ⇔ − − + =
 − + − − + − 
2
2
2 2
3 3
1 0
6 1
1 0 ( )
2 3( 2)
4
2 4
x x
x x
VN
x x
x
 − − =

− −⇔ + =
− −  − + +   
⇔
1 5
2
x
−
= (loại) hoặc
1 5
2
x
+
=
3 5
2
y
+
⇒ =
Vậy hệ phương trình có nghiệm duy nhất: ( )
1 5 3 5
; ;
2 2
x y
 + +
=   
 
Kinh nghiệm: Khi dùng máy tính để bấm nghiệm pt(*) ta nên kiểm tra xem nó có giống nghiệm của một trong
các biểu thức nằm dưới dấu căn không.
Bài tập tương tự: Giải hệ phương trình :
( ) ( )
2
3 2 2 2 2 2
12 2 2 2 4 (1)
4 3 5 4 8 (2)
x x y y
x y y y x y x y
 + − = − −


+ + − = + +
HD: ( ) ( )24 2 2
(2) 2 3 8 5y t y t t⇔ − = − + − . Sử dụng BBT của hàm 2
( ) 3 8 5f t t t= − + − ta chứng
minh được ( ) 0f t ≤ dấu bằng xảy ra khi t = 2. Từ đó: (2) 2x y⇔ =
Thay x = 2y vào (1) :
2
2 3y y y y− − = − + (*)
Để (*) có nghiệm thì : 2 1
2 0
2
y
y y
y
≤ −
− − ≥ ⇔  ≥
kết hợp với ĐK 0 3y≤ ≤ ta có 2 3y≤ ≤
Kỷ năng: Với bài này ta không được may mắn như bài trên . Vì thể ta sẽ đi tìm nhóm chung bằng kỷ thuật
them bớt như sau:
( )
2
2 2 2 2
2
(*) 3 2 ( ) 3 ( ) 0
(2 1) 3 (2 1)
3 2 0
( ) 3 ( )
y y m n y m y y n y
y m y m y n y n
y y m n
y m y y n y
⇔ − − − − + + − − + + − =
+ + + − + − +
⇔ − − − − + + =
+ + − + +
Khi đó ba nhóm trên có phần tử chung thì hệ sau phải có nghiệm:
2
2
2 1 3
2 1 3 2
3 2 1
2
m
n m
m m n n
n m n
+ =−
 − =− = −
⇔ 
− =− − − =−
 =− − −
2 2
2 2 3 1 3 1
(*) 3 1 ( 2) 3 ( 1) 0 3 1 0
2 3 1
y y y y
y y y y y y y y
y y y y
− + − +
⇔ − + + − − − + − − = ⇔ − + + + =
− + − − +
( ) [ ]2 21 1 1 1
3 1 1 0 3 1 0 1 0, 2 , 3`
2 3 1 2 3 1
y y y y vi y
y y y y y y y y
   
⇔ − + + + = ⇔ − + = + + > ∀ ∈      − + − − + − + − − +   
Câu 9 (1,0 điểm).Tìm tất cả các số thực m sao cho hệ phương trình sau có 4 nghiệm thực phân biệt.
( ) 2
2 2
2.3 .2 7.2 (1)
( 4) 2 3 5 8 32 (2)
x y x y x y
x y
m x y y x y
− − + −
 + − =

+ + + = + +
Bài giải:
Đặt: t x y= −
2 3
(1) 2.3 .2 7.2 2 4 7
2
t
t t t
t t+  
⇔ + = ⇔ + = 
 
Xét hàm số :
3
( ) 2 4
2
t
f t t
 
= + 
 
ta có:
3 3
'( ) 2 ln 4 0 ,
2 2
t
f t t R
   
= + > ∀ ∈   
   
Suy ra hàm số:
3
( ) 2 4
2
t
f t t
 
= + 
 
liên tục và đơn điệu tang trên R.
Mà (1) ( ) (1)f t f⇔ = nên t = 1 là nghiệm duy nhất của phương trình(1).
1 1 1t x y y x= ⇔ − = ⇔ = −
Thay 1y x= − vào phương trình (2) ta có: ( ) 2 2
4 2 5 8 24 (3)m x x x x+ + = + +
Dễ thấy x = -4 không phải là nghiệm của phương trình m R∀ ∈
( ) ( )
2222 2
2
4 2
(3) 4 2 4 4 2 4
42
x x
m x x x x m
xx
+ +
⇔ + + = + + + ⇔ = +
++
Đăt:
2
4
2
x
u
x
+
=
+
3
2
2 4
' 0 2
2
x
u x
x
−
= = ⇔ =
+
,
2
4
lim lim 1
2x x
x
u
x→−∞ →−∞
+
= = −
+
và
2
4
lim lim 1
2x x
x
u
x→+∞ →+∞
+
= =
+
BBT:
Từ BBT ta thấy ( 1, 6x R t ∀ ∈ ⇒ ∈ − 
4
(3) ( )m t g t
t
⇔ = + =
2
2 2
24 4
'( ) 1 0
2 ( )
tt
g t
t loait t
=−
=− = = ⇔  = −

BBT:
Từ BBT ta có:
Hệ phương trình có nghiệm
x
'u
u
−∞ +∞2
0+ -
-1
1
6
t
'( )g t
1− 62
0- +
10 6
0
-
+∞
⇔ Phương trình (3) có nghiệm x
⇔ Phương trình ( )g t m= có nghiệm ( 1, 6t ∈ − 
4
5
m
m
≥
⇔  < −
Vậy hệ phương trình có nghiệm khi 4m ≥ hoặc 5m < −

ĐỀ SỐ 3
Câu 1( 2,0 điểm):Cho hàm số ( )2 3 2
5 6 6 6y m m x mx x=− + + + − Với m là tham số
1. Khảo sát sự biến thiên và vẽ đồ thị của hàm số trên khi m = 1.
2. Tìm m để hàm số trên đơn điệu. Khi đó hàm số trên đồng biến hay nghịch biến.
Câu 2(1,0 điểm).
1. Giải phương trình
1
2cos sin 2
6 6 2
x x
π π   
− − − =   
   
2. Giải phương trình: ( ) ( ) ( )2 2 2
3 4 16log 1 .log 1 log 1x x x x x x− − + − = − −
Câu 3 (1,0 điểm).
1. Tính tích phân:
5
4
40
cos5 .cos 1
cos
x x
I dx
x
π
+
= ∫
2. Tính tích phân :
3 2
1
( 1)ln 2 1
2 ln
e x x x
I dx
x x
+ + +
=
+∫
Câu 4 (1,0 điểm).
1. Tìm tập hợp điểm biểu diển của số phức 1w z i= + − biết rằng số phức ( ) 2
2 i z iz z+ − − là một số
thuần ảo.
2. Có 40 tấm thẻ được đánh số từ 1 đến 40. Chọn ngẫu nhiên 10 tấm . Tính xác suất
để có 5 tấm thẻ mang số lẻ, 5 tấm thẻ mang số chẵn trong đó có đúng một tâm thẻ mang số
chia hết cho 6.
Câu 5 (1,0 điểm). Cho hình chóp SABCD có đáy là hình chữ nhật tâm O, AB = a, AD = 2a; M là điểm
thuộc cạnh AB sao cho MA = 2MB, tam giác SOM cân tại đỉnh S và nằm trong mặt phẳng vuông
góc với (ABCD). Biết mặt bên (SBC) hợp với mặt phẳng đáy (ABCD) một góc bằng 600
. Tính
theo a thể tích khối chóp SAMOD và tính khoảng cách từ M đến mặt phẳng (SCD).
Câu 6 (1,0 điểm).Trong không gian với hệ trục Oxyz, cho đường thẳng
1 6
:
1 1 1
x y z
d
− +
= = Viết
phương trình mặt phẳng chứa đường thẳng d và tạo với mặt phẳng (Oyz) một góc bằng 450
.
Câu 7 (1,0 điểm).Trong mặt phẳng với hệ trục tọa độ Oxy, cho hình vuông ABCD có I là giao điểm hai
đường chéo, G là trọng tâm của tam giác ABI và điểm E(7; -2) thuộc đường chéo BD sao cho
GA = GE. Tìm tọa độ các đỉnh của hình vuông A, B, C, D biết đường thẳng GA có phương trình
3x – y -13 = 0 và điểm A có hoành độ nhỏ hơn 4.
Câu 8 (1,0 điểm). Giải Bất phương trình sau:
( )2 2
2
3 3 8 3 2 1
0
1 3 4 5
x x x x
x x
+ + + − +
≤
− − +
Câu 9 (1,0 điểm).Tìm giá trị lớn nhất, nhỏ nhất của hàm số sau :
5 4 1
5 4 2 1 6
x x
y
x x
− − +
=
− + + +
--- Hết ---
GIẢI ĐỀ SỐ 3
Câu 1( 2,0 điểm):Cho hàm số ( )2 3 2
5 6 6 6y m m x mx x=− + + + − Với m là tham số
Tìm m để hàm số trên đơn điệu. Khi đó hàm số trên đồng biến hay nghịch biến.
Bình luận :Để hàm số 3 2
( )y f x ax bx cx d= = + + + đơn điệu( một chiều biến thiên) thì có hai khả năng :
hoặc là hàm số đó quy về bậc một (tức a = b = 0) hoặc '( )f x không đổi dấu trên TXĐ .
Bài giải :
TH1 : ( )2
5 0 0 5m m m m− + =⇔ =∨ =−
Với m = 0 ta có hàm số trở thành : 6 6y x= − (vì a = 6 > 0)đơn điệu tăng trên R ⇒ m = 0 thỏa
Với m = -5 ta có hàm số trở thành : 2
30 6 6y x x= + − có đồ thị là một parabol nên không đơn
điệu⇒ m = -5 không thỏa đề toán
TH2 : ( )2 0
5 0 (*)
5
m
m m
m
≠
− + ≠ ⇔ 
≠ −
Khi đó : ( )2 2
' '( ) 3 5 12 6y f x m m x mx= =− + + + là một tam thức bậc hai, nên ta có :
Hàm số đơn điệu khi '( )f x không đổi dấu trên TXĐ 2
54 90 0m m⇔ ∆= + ≤
5
0
3
m⇔ − ≤ ≤ Kết hợp điều kiện (*) ta có :
5
0
3
m− ≤ <
Vậy hàm số đơn điệu khi
5
0
3
m− < ≤ (vì m = 0 thỏa theo TH1)
m
( )2
3 5a m m=− +
-5 0
0 +
5
3
−
- -+
Bảng xét dấu của hệ số a của '( )f x :
Từ BXD trên ta thấy :
( )2
2
3 5 0 5 5
, , 0 '( ) 0, , , 0
3 354 90 0
a m m
m f x x R m
m m
 =− + >    
∀ ∈ − ⇔ ≥ ∀ ∈ ∀ ∈ −      ∆= + ≤
⇒hàm số ( )f x đơn điệu tăng khi
5
, 0
3
m
 
∈ −  
Vậy
5
, 0
3
m
 
∈ −  
thì hàm số đởn điệu tăng.
Câu 2(1,0 điểm).
1. Giải phương trình
1
2cos sin 2
6 6 2
x x
π π   
− − − =   
   
Bình luận:Với phương trình lương giác có chứa cung hằng số dạng ( ) ( ) ( )1 1 2 2, ,..., n na x a x a xα α α+ + + thì ta sẽ
có 4 định hướng sau :
Hướng 1: Sử dụng công thức công (thướng thì lời giải sẽ hơi cồng kềnh)
Hướng 2: Sử dụng cống thưc hạ bậc rồi dùng công thức cộng (thường chỉ dùng cho dạng
2
cos
4
ax
π 
± 
 
hoặc 2
sin
4
ax
π 
± 
 
)
Hướng 3: Sử dụng công thức tổng thành tích hoặc tích thành tổng (thường thì phương trình có
dạng tích của hàmlương giác chứa cung hằng số hoặc chứa các số:
1 3
;
2 2
± ± đi với cung ;
3 6
π π
)
Hướng 4: Đặt ẩn phụ ( )i it a x α= + với { }1 2, , ...,i na Min a a a=
Bài giải :Ta có :
1
sin
2 6
π
=
2cos sin 2 sin 2cos 2sin cos
6 6 6 6 6
pt x x x x x
π π π π π       
⇔ −= − + ⇔ −= −       
       
( ) ( )
sin 1
2
2cos 1 sin 0
26
cos 0
6 3
x x k
x x k Z
x x k
π
π
π
π π
π

= ⇔ = + 
⇔ − − =⇔ ∈ 
    − = ⇔ = +   
Bài tập tương tự:
2
1
sin3sin4
6
sin =++





− xxx
π
HD:
1
sin
2 6
π
= ,
sin3 sin 2sin 2 cos
sin 4 sin 2cos 2 sin 2
6 6 6
x x x x
x x x
π π π
+ =

    
− − = −   
   






+=





−
2
3
10
sin
2
1
210
3
sin
xx ππ
HD: Đặt
3
10 2
x
t
π
= −
xxxx 2cos
6
sin
6
sin.cos4 =





−





+
ππ
HD:
1 1 1
sin sin cos2 cos cos2
6 6 2 3 4 2
x x x x
π π π     
+ − =− − =−     
     






−=
+
+−
24
cos8
cos
)sin1(3
tantan3 2
2
3 x
x
x
xx
π
HD: 2
1 cos
2
8cos 8 4 4sin
4 2 2
x
x
x
π
π
 
+ − 
   − = = + 
 
2. Giải phương trình: ( ) ( ) ( )2 2 2
3 4 16log 1 .log 1 log 1x x x x x x− − + − = − −
Bài giải :ĐK : 1x ≥
( ) ( ) ( )2 2 2
3 4 16 3log 1 .log 1 log 3.log 1 0pt x x x x x x⇔ − − + − − − − =
( ) ( ) 2
2 2
3 4 4
log 1 log 1 log 3 0x x x x ⇔ − − + − − =
  
( )
( ) 2
2
3
2
4 4
log 1 0 (1)
log 1 log 3 (2)
x x
x x
 − − =

⇔ 
+ − =

2 2
2 2
1 0
(1) 1 1 1 1 1
1 2 1
x
x x x x x
x x x
− ≥
⇔ − − = ⇔ − = − ⇔ ⇔ =
− = − +
( )2 2
4 4 4 2 2
3 01 2 3
(2) log 1 log 3 log 3 1 3
2 31 2 3 3
x
x x x x x
x x x
 − ≥
⇔ + − = = ⇔ − = − ⇔ ⇔ =
− = − +
Vậy phương trình có hai nghiệm: 1x = và
2 3
3
x =
Câu 3 (1,0 điểm).
1. Tính tích phân:
5
4
40
cos5 .cos 1
cos
x x
I dx
x
π
+
= ∫
Bình luận:
Với tích phân lương giác không chứa căn thức và mẫu thức thì ta có thể dùng công thức hạ bâcvà công thức
tích thành tồng rồi sử dụng công thức: ( ) ( )
1
sin cosax dx ax c
a
=− +∫ và ( ) ( )
1
cos sinax dx ax c
a
= +∫
Bài giải :
5
4 4 4
1 24 40 0 0
cos5 .cos 1 1
cos5 .cos
cos cos
x x
I dx x xdx dx I I
x x
π π π
+
= = + =+∫ ∫ ∫
( )4
1 0
1 1 1 1 1
cos6 cos4 sin 6 sin 4 4
2 2 6 4 12
0
I x x dx x x
π π
 
= + = + =− 
 
∫
( )
3
24 4 4
2 4 2 20 0 0
1 1 1 tan 4
. tan 1 (tan ) tan 4
cos cos cos 3 3
0
x
I dx dx x d x x
x x x
π π π π
 
= = = + = + = 
 
∫ ∫ ∫
Vậy 1 2
1 4 5
12 3 4
I I I= + =− + =
2. Tính tích phân :
3 2
1
( 1)ln 2 1
2 ln
e x x x
I dx
x x
+ + +
=
+∫
Bình luận:Với tích phân có căn hoặc mẫu số chứa nhóm có dạng:
ln
ln , , sin , cos ,x x
x x xe x x x x
x
thì ta
thử đạo hàm nhóm đó và so sánh với những nhóm còn lại để chọn ra phương án giải là đặt ẩn phụ hay
tich phân từng phần (nếu cần thì ta cũng có thể tách tích phâncho phù hợp).cụ thể như ví dụ trên ta sẽ
xét: ( )2 ln ' 1 lnx x x+ =+ điều này khiến ta nghĩ đến việc tách tử số thành :
3 2 2
( 1)ln 2 1 (2 ln ) (1 ln )x x x x x x x+ + += + + +
Bài giải :
2
2
1 1 1
(2 ln ) (1 ln ) 1 ln
2 ln 2 ln
e e ex x x x x
I dx x dx dx
x x x x
+ + + +
= = +
+ +∫ ∫ ∫
( )
3
1
1
2 ln
13 2 ln
eex
d x x
x x
=+ +
+∫
3 3
1 1
ln 2 ln ln(2 ) ln 2
13 3 3 3
ee e
x x e= − + + = − + + −
ĐS:
3
1
ln(2 ) ln 2
3 3
e
I e= − + + −
Câu 4 (1,0 điểm).
1. Tìm tập hợp điểm biểu diển của số phức 1w z i= + − biết rằng số phức ( ) 2
2 i z iz z+ − − là một số
thuần ảo.
Bài giải :
Gọi , ,w x yi x y R= + ∀ ∈
Ta có: ( ) ( )
( ) ( )
( ) ( )
2 2 2
1 1
1 1 1 1
1 1
z x y i
w z i z w i x y i
z x y
 = − − +
= + − ⇔ = − + = − + + ⇒ 
= − + +
( ) ( ) ( )( ) ( ) ( ) ( ) ( )
2 2 2
' 2 2 1 1 1 1 1 1z i z iz z i x y i i x y i x y = + − − = + − − + − − + + − − + +    
( ) ( )2 2
4 3 2 1x y x y x y i= − − + − − + − − +
'z là số thuần ảo
( )
2 22 2
2
4 3 04 3 0
2 1 0 5 11 8 0 *
x y x yx y x y
x y y y
 + − + + =− − + − − = 
⇔ ⇔ 
− − + ≠ − + ≠ 
ĐK: ( )* đúng y R∀ ∈
Vậy tập hợp điểm biểu diển của w là một đường tròn tâm
1
2;
2
I
 
− 
 
và bán kính
5
2
R =
2. Có 40 tấm thẻ được đánh số từ 1 đến 40. Chọn ngẫu nhiên 10 tấm . Tính xác suất
để có 5 tấm thẻ mang số lẻ, 5 tấm thẻ mang số chẵn trong đó có đúng một tâm thẻ mang số
chia hết cho 6.
Bài giải :
Trong 40 tấm thẻ có 20 tấm mang số lẽ, 6 tấm mang số chia hết cho 6 và 14 tấm mang số chẵn
không chia hết cho 6
Gọi Ω là không gian mẫu của phép thử chọn 10 tấm thẻ từ 40 tấm thẻ
⇒số phần tử của không gian mẫu là : 10
40CΩ =
Gọi A là biến cố trong 10 tấm thẻ được chọn có 5 tấm thẻ mang số lẻ, 5 tấm thẻ mang số chẵn
trong đó có đúng một tâm thẻ mang số chia hết cho 6
Khi đó số phần tử của không gian biến cố A được xác định như sau:
Bước 1: chọn 5 tấm thẻ mang số lẻ từ 20 tấm thẻ ⇒ có 5
20C cách chọn
Bước 2: chọn 1 tấm thẻ mang số chia hết cho 6 từ 6 tấm thẻ ⇒ có 1
6C cách chọn
Bước 3: chọn 5 tấm thẻ mang số chẵn không chia hết cho 6 từ 14 tấm thẻ ⇒có 4
14C cách chọn
Theo quy tắc nhân ta có: 5 1 4
20 6 14A C C CΩ =
Vậy xác suất của biến cố A là
5 1 4
20 6 14
10
40
( ) ...A C C C
P A
C
Ω
= = =
Ω
Câu 5 (1,0 điểm). Cho hình chóp SABCD có đáy là hình chữ nhật tâm O, AB = a, AD = 2a; M là điểm
thuộc cạnh AB sao cho MA = 2MB, tam giác SOM cân tại đỉnh S và nằm trong mặt phẳng vuông
góc với (ABCD). Biết mặt bên (SBC) hợp với mặt phẳng đáy (ABCD) một góc bằng 600
. Tính
theo a thể tích khối chóp SAMOD và tính khoảng cách từ M đến mặt phẳng (SCD).
Bài giải :
Gọi H là trung điểm OM SH OM⇒ ⊥ (vì SMN∆ cân tại S)
Mặt khác ( )SOM và ( )ABCD vuông góc nhau theo gao tuyến OM
( )SH ABCD⇒ ⊥ .
Dựng HK ⊥ BC (với K∈ BC)
Khi đó ta có : ( )SHK BC⊥
( ) ( ),( )SBC ABCD SKH⇒ =,  0
60gt SKH⇒ =
Gọi E là trung điểm BC . Khi đó BMOE là hình thang có
HK là đường trùng bình :
5
2 12
BM EO a
HK
+
⇒= =
Xét tam giác SHK vuông tại H ta có :
S
A
B
D
C
H
O
M
K E
F
 05 5 3
tan tan 60
12 12
SH a a
SKH SH
HK
= ⇔ = =
( )
2
1 1 1 1 5 5
. . 0, . . .
2 2 2 2 3 2 12 6
AMOD ABD OMB
AB AD a
S S S AB AD MB d AB AB AD AB AD= − = − = − = =
2 3
1 1 5 3 5 25 3
.
3 3 12 6 216
SAMOD AMOD
a a a
V SH S= = = (đvtt)
Ta có MA // (SCD) ( ) ( )
3
,( ) ,( ) SACD
SCD
V
d M SCD d A SCD
S
⇒ = =
3
1 1 5 3
. .
3 2 36
SACD
a
V SH AD AB= =
Dựng HF ⊥ CD (với F ∈ CD) . Khi đó ta có :
3 3
4 2
a
HF KC AD= = =
Theo Pi ta go cho tam giác SHF ta có :
2 2
2 2 5 3 3 399
12 2 12
a a a
SF SH HF
   
= + = + =       
2
1 399
.
2 24
SCD
a
S SF CD= =
( ) ( )
3 10 10 133
,( ) ,( )
133133
SACD
SCD
V a a
d M SCD d A SCD
S
= = = =
ĐS :
3
5 3
36
SACD
a
V = (đvtt) và ( )
10 133
,( )
133
a
d M SCD = (đvđd)
Câu 6 (1,0 điểm).Trong không gian với hệ trục Oxyz, cho đường thẳng
1 6
:
1 1 1
x y z
d
− +
= = Viết
phương trình mặt phẳng chứa đường thẳng d và tạo với mặt phẳng (Oyz) một góc bằng 450
.
Bình luận :Với bài toán lập phương trình mặt phẳng có liên qua đến góc hoặc khoảng cách thì thông thường
ta giải bằng cách gọi phương trình tổng quát ( ): 0P ax by cz d+ + + =( ĐK : 2 2 2
0a b c+ + > )
Sau đó sử dụng 2 dữ kiện bậc một (đi qua điểm hoặc VTPT vuông góc vectơ cho trước) để chuyển (P) theo hai
tham số. cuối cùng sử dụng công thức góc hoặc khoảng cách để lập phương trình bậc hai theo hai tham số đó
rồi chọn giá trị cho một tham số để tìm tham số còn lại.
Bài giải :
Ta có tọa độ (1;0; 6)A − và (0; 1; 12)B − − đồng thời thỏa phương trình d A d⇒ ∈ , B d∈
(Oyz): 0x = có VTPT là (1;0;0)Oyzn =

Gọi (P): 0ax by cz d+ + + =( ĐK : 2 2 2
0a b c+ + > )có VTPT là ( ; ; )Pn a b c=

Ta có :
( )
( ): ( 6 ) 6 0( ) 6 0 6
( )
( ) 12 0 6 ; 6 ;P
P ax a c y cz c aA P a c d d c a
d P
B P b c d b a c n a a c c
+ − − + + − =∈ − + = = −   
⊂ ⇒ ⇔ ⇔ ⇒   
∈ − − + = =− − = − −   

( ) ( )
( )
22 2
.
cos ( ),( ) cos ,
6
P Oyz
P Oyz
P Oyz
n n a
P Oyz n n
n n a a c c
= = =
+ − − +
 
 
 
2 2 2
2 2
0
1
2 2 12 37 12
22 12 37
37
c
a
gt a a ac c a
ca ac c
=
⇔ = ⇔ = + + ⇔
 = −+ +

Với 0 ( ): 1 0c P x y= ⇒ − − =
Với
12
( ):37 35 12 109 0
37
a
c P x y z=− ⇒ + − − =
Vậy phươg ntrình mặt phẳng cần tìm là 37 35 12 109 0x y z+ − − =
Câu 7 (1,0 điểm).Trong mặt phẳng với hệ trục tọa độ Oxy, cho hình vuông ABCD có I là giao điểm hai
đường chéo, G là trọng tâm của tam giác ABI và điểm E(7; -2) thuộc đường chéo BD sao cho
GA = GE. Tìm tọa độ các đỉnh của hình vuông A, B, C, D biết đường thẳng GA có phương trình
3x – y -13 = 0 và điểm A có hoành độ nhỏ hơn 4.
Bình luận :Với bài toán hình vuông ABCD trong tọa độ phẳng ta có thể giải bằng cách xét một hình
đồng dạng với nó là A’B’C’D’ với các đỉnh ta đều xác định cụ thể . Ta đi xác định các tính chất về
góc và tỷ số độ dài trong hình A’B’C’D’. Vì hai hình đồng dạng nhau thì tính chất về góc và tỷ số
độ dài không thay đổi nên các tính chất đó ta có thể áp đặt vào hình ABCD của đề toán .
Bài giải :
Xét hình vuông A’B’C’D’ cạnh bằng 2, khi đó tọa độ được xác định như hình vẽ:
Ta có : ( )' ' ': 2 0 ' ; 2E B D x y E m m∈ + − = ⇒ −
( )
2 2
2 21 1
' ' ' ' 1 1
3 3
G E G A m m
   
= ⇔ − + − = +   
   
2
0 ( )
6 8 0 4
3
m loai
m m
m
=
⇔ − = ⇔
 =

 4 2
' ;
3 3
E
 
⇒  
 
1
' ' ; 1
3
G A
 
= − − 
 

và
1
' ' 1;
3
G E
 
= − 
 

Ta có : ' '. ' ' 0 ' ' ' 'G A G E G A G E=⇒ ⊥
 
và
2 2
2 2
4 4
3 3' ' 4 6 8
.
' ' 3 5 55 5
6 6
E B
E F
   
+   
   
= = =
   
+   
   
Vì A’B’C’D’ và BACD đồng dạng nhau nên tính chất về góc và tỷ số độ dài của hai hình sẽ không
đổi
(1)
' ' 8
(2)
' ' 5
AG GE
EB E B
EF E F
⊥

⇒ 
= =
Từ (1) kết hợp với giã thuyết ta có : GE qua E và vuông góc GA : 3 1 0EG x y⇒ + − =
G là giao điểm của AG và EG ( )4; 1G⇒ −
( ); 3 13A AG A m m∈ ⇒ − (Với m < 4).
A’(0; 0)
B’(0; 2)
D’(2; 0)
C’(2; 2)
I’(1; 1)G(
1
3
; 1)
x
y
E’
F’(
1 3
;
2 2
)
Ta có: ( ) ( )
2 2 3
4 3 12 10 4 1 (3; 4)
5 ( )
m
AG GE m m m A
m loai
=
= ⇔ − + − = ⇔ − = ⇔ ⇒ − =

Gọi F(a ; b) là trung điểm BI ta có:
4 3 2( 4) 9 1
2 ;
1 ( 4) 2( 1) 2 2
a
AG GF F
b
− = −  
=⇔ ⇒  − − − = +  
 
( )
9
5( 7) 8 7
32
(2) 5 8 (3;2)
21
5 2 8 2
2
B
B
B
B
x
x
EB EF B
y
y
  
− = −  =  
⇔ = ⇔ ⇔ ⇒ 
=   + = +   
 
F là trung điểm BI (6; 1)I⇒ −
I là trung điểm AC ( )9; 2C⇒ .
I là trung điểm BD ( )9; 4D⇒ −
Vậy các điểm cần tìm là: (3; 4)A − , (3;2)B , ( )9; 2C và ( )9; 4D −
Bình luận:
• Trong bài này ta có thể chứng minh AG GE⊥ bằng cách khác như sau: Dễ thấy IG là trung trực của
AB ⇒ GA = GB ⇒ G là tâm đường tròn ngoại tiếp tam giác ABE   0
2 90AGE ABE⇒ = = .Nhưng khi
đó để tìm điểm B ta cũng phải sử dụng talet tính tỷ số độ dài
EB
EF
(tương đối phức tạp)
• Phương pháp trên có thể sử dụng cho bất cứ hình nào miễn sao hình đó đặt vào hệ trục ta có thể xác
định được tọa độ các đỉnh. Cụ thể như: hình vuông, hình chữ nhật có tỷ số độ dài, hình thoi có tỷ số hai
đường chéo, tam giác vuông có tỷ số hai cạnh góc vuông, tam giác đều….
Câu 8 (1,0 điểm). Giải Bất phương trình sau:
( )2 2
2
3 3 8 3 2 1
0
1 3 4 5
x x x x
x x
+ + + − +
≤
− − +
Bài giải :
Ta có : 2 2 211 11
3 4 5 3 4 5 1 1 3 4 5 0
4 3 3
x x x x x x
a
∆
− + ≥ − = ⇔ − + ≥ > ⇔ − − + <
( )2 2
( ) 3 3 8 3 2 1 0 (*)bpt f x x x x x⇔ = + + + − + ≥
Bình luận :Với BPT có dạng: ( ) ( ) ( ) 0f x g x h x+ ≥ thì thông thường ta nghĩ đến các hương biến đổi sau:
Hướng 1 : Sử dụng đồng nhất thức : ( )
222 2
3 3 8 3 2 1x x m x n x+ += − + + để chuyển về thuần nhất
Hướng 2 :Chuyển về dạng phương trình bậc hai theo nhóm biến 2
2 1x + .
Hướng 3 : Thêm bới để nhân liên hợp rồi chuyển về dạng tích
Hướng 4 :Phân tích về dạng ( ) ( )( ) ( )f u x f v x= sau đó xét hàm dặt trưng ( )f t
Bài này ta có thể đi theo hướng 2: Ta sẽ tìm lượng thêm bớt để đưa về phương trình bậc 2 như sau :
( )
2
2 2 2
( ) 2 1 8 3 2 1 (2 3) 3 0f x m x x x m x x m= + + − + − − + + −=
( ) ( ) ( )2 2 2 2 2
8 3 4 (2 3) 3 8 12 64 2(2 24) 4 12 9x m m x x m m m x m x m m∆ = − − − + + − = − + + − + + − +
Xét 0∆ = có nghiệm kép (để ∆ có thể đưa về dạng hằng đẳng thức)
( )( )2 2 2
' (2 24) 8 12 64 4 12 9 0m m m m m∆∆= + − − + + − + = bấm máy tính ta có nghiệm nguyên 3m =
Xét
2
2 2 2
( ) 0 3 2 1 (8 3) 2 1 3 0f x x x x x x= ⇔ + + − + − + =
( )
22 2
(8 3) 4.3( 3 ) 10 3x x x x∆= − − − + = −
( )
( )
2
2
(8 3) 10 3
2 1 2 1
6
( ) 0
(8 3) 10 3
2 1
6 3
x x
x x
f x
x x x
x
− − − −
+ = =− +
= ⇔
− − + −
+= =

( )2 2
(*) 2 1 2 1 2 1 0
3
x
x x x
 
⇔ + + − + − ≥ 
 
Dễ thấy :
2 2 2
2 271
2 1 1 2 1 0 ,
36 36 36 6 6 6
x x x x x x
x x x R+ = + + > = ≥ ⇔ + − > ∀ ∈
2
2 2
2
1 2 0
2 1 0
(*) 2 1 2 1 0 2 1 1 2 0
1 2 0
2 4 0
x
x
x x x x x
x
x x
 − <

+ ≥
⇔ + + − ≥ ⇔ + ≥ − ⇔ ⇔ ≥ − ≥
 − ≤
Vậy bất phương trình có tập nghiệm là : [ )0 ,S= + ∞
Câu 9 (1,0 điểm).Tìm giá trị lớn nhất, nhỏ nhất của hàm số sau :
5 4 1
5 4 2 1 6
x x
y
x x
− − +
=
− + + +
Bài giải :
Nhận thấy : ( ) ( )
2 2
5 4 2 1 9x x− + + =
Đăt :
5 4 3sin 0
2 1 3cos 0
x u
x u
 − = ≥

+= ≥
ĐK : 0 ,
2
u
π 
∈  
(vì sin 0u ≥ ,cos 0u ≥ )
2sin cos
2sin 2cos 4
u u
y
u u
−
=
+ +
.
Đặt : 2
2
tan sin
2 1
u t
t u
t
= ⇒ =
+
và
2
2
1
cos
1
t
u
t
−
=
+
2
22 2
2 2
2 2
2 1
2
4 11 1( )
2 1 6 4 2
2 2 4
1 1
t t
t tt ty f t
t t t t
t t
−
−
+ −+ += = =
− + +
− +
+ +
,
[ ]0 , 0 , t 0 , 1
2 2 4
u
u
π π   
∀ ∈ ⇒ ∈ ⇒ ∈      
( )
2
22
20 16 12 4 31
'( ) 0
56 4 2
t t
f t t
t t
− + + ±
= = ⇔ =
+ +
BBT :
Từ BBT ta có :
[ ]
[ ]
[ ]
[ ]
0,1
0,1
1
in ( ) 0
2
1
( ) 1
3
M f t khi t
Max f t khi t

=− =

 = =

Vậy ( )
1
2
Min y = − khi
5
5 4 3sin 0 0
4
x x− = = ⇔ =
( )
1
3
Max y = khi 2 1 3cos 0 1
2
x x
π
+ = = ⇔ =−
ĐỀ SỐ 4
Câu 1 (2,0 điểm). Cho hàm số : 3 2
3 2y x mx= − + (với m là tham số)
1. Khảo sát sự biến thiên và vẽ đồ thị hàm số trên khi m = 1.
2. Tìm m để đường thẳng đi qua hai điểm cực trị của đồ thị hàm số trên tạo với các trục tọa độ thành
một tam giác có diện tích bằng 4
Câu 2 (1,0 điểm).
1. Giải phương trình : sin 4 2 cos3 4sin cosx x x x+= + +
2. Giải hệ phương trình :
( )( ) ( )2 2 2
2 2
2 11 17 2 2 6
log 1 1 1 log
y x y x x
x y x x y y
 + + + + − = +

 − + + −= +  
Câu 3 (1,0 điểm).
1. Tính tích phân :
2 2
3
1
(1 )ln
e
x x x
I dx
x
+ +
= ∫
2. Tính thể tích khối tròn xoay tạo thành khi quay hình phẳng được giới hạn bởi các đường
1
, 0, 0
1 4 3
y y x
x
= = =
+ −
và 1x = xung quanh trục hoành.
Câu 4 (1,0 điểm).
1. Cho số phức z thỏa mãn 2 3z z i+ = + . Tính tổng: 2014 2015 2016
T z z z= + +
2. Gọi E là tập hợp các số tự nhiên có ba chữ số sao cho số đứng sau lớn hơn số đứng trước. Lấy
ngẫu nhiên từ tập E ra một số , Tính xác suất để số được chọn là một số chẵn.
Câu 5 (1,0 điểm). Cho hình lăng trụ tam giác ABCA’B’C’ có AB = a, BC = 2a,  0
30ACB = , hình
4 31
5
− 4 31
5
+
0 0
0 1
+
+
++- -
t
'( )f t
( )f t
1
2
−
1
3
chiếu vuông góc của A’ lên mặt phẳng (ABC) trùng với trọng tâm G của tam giác ABC và góc
giữa AA’ tạo với mặt phẳng (ABC) bằng 0
60 . Tính theo a thể tích của khối đa diện BCC’B’A’
và khoảng cách giữa hai đường thẳng B’C’ và A’C.
Câu 6( 1,0 điểm): Trong không gian với hệ trục tọa độ Oxyz, cho các điểm A(1; 1; -1), B(1; 1; 2),
C(-1; 2; -2) và mặt phẳng (P) có phương trình x – 2y + 2z +1 = 0. Viết phương trình mặt phẳng đi
qua A , vuông góc mặt phẳng (P) và cắt đoạn thẳng BC tại điểm I sao cho IB = 2IC
Câu 7 (1,0 điểm). Trong mặt phẳng với hệ trục tọa độ Oxy, cho tam giác ABC có trực tâm H(5; 5) và
đường thẳng BC : x + y – 8 = 0. Đường tròn ngoại tiếp tam giác ABC đi qua điểm ( )1; 5D . Tìm tọa độ
các đỉnh của tam giác ABC, biết rằng độ dài cạnh BC bằng 4 2 và hoành độ của điểm B lớn hơn 3.
Câu 8 (1,0 điểm).Giải hệ phương trình:
( )( )
( )3 2 2
2 3 1 4 1 0
1
2 5 4 1 4
x y y x y
x x x x x y x
x
 − + + + + + + =


 + + += + + +

Câu 9 (1,0 điểm). Tìm giá trị nhỏ nhất của biểu thức :
2
2 2 2 2 20 9 42
2 1 4 2 1 3 (3 4 )
5
y y
P x y x x y x x x y
− +
= + − + + + + + + − +
--- Hết ---
GIẢI ĐỀ SỐ 4
Câu 1 (2,0 điểm). Cho hàm số : 3 2
3 2y x mx= − + (với m là tham số)
Tìm m để đường thẳng đi qua hai điểm cực trị của đồ thị hàm số trên tạo với các trục tọa độ
thành một tam giác có diện tích bằng 4.
Bài giải:
Ta có: ( )2
3
0 2
' 3 6 0 3 2 0
2 2 4
x y
y x mx x x m
x m y m
= ⇒ =
= − =⇔ − =⇔ 
= ⇒ = −
Hàm số có hai cực trị ⇔ phương trình ' 0y = phải có hai nghiệm phân biệt 2 0 0m m⇔ ≠ ⇔ ≠ (*)
Khi đó hai điểm cực trị của đồ thị là : ( )0; 2A và ( )3
2 ; 2 4B m m−
( )3
2 ; 4AB m m= −

là VTCP của AB 2
3
0 2
: : 2 2 0
2 4
x y
AB AB m x y
m m
− −
⇒ = ⇔ + −=
−
(vì 0m ≠ )
Gọi 2 2
1 1
; 0M AB Ox M OM
m m
 
= ∩ ⇒ ⇒ = 
 
Ta có: ( )0 ; 2 2A AB Oy OA= ∩ ⇒ =
2
1 1 1
4 . 4 4
2 2
AMOgt S OAOM m
m
⇒ = ⇔ = ⇔ = ⇔ =± thỏa điều kiện(*)
Vậy
1
2
m = ± thỏa yêu cầu bài toán.
Câu 2 (1,0 điểm).
1. Giải phương trình : sin 4 2 cos3 4sin cosx x x x+= + +
Bình luận: Với phương trình lượng giác chứa hai số hạng cùng hệ số, cùng loại hàm, cùng tính
chẵn hoặc lẽ của cung (hai số hạng được gạch dưới) thì ta sử dụng công thức tổng thành tích . Sau
đó Tìm mọi cách để biến đổi các số hạng còn lại để xuất hiện nhóm chung với biểu thức tích ở trên
.cuối cùng ta chuyển phương trình về dạng tích.
Bài giải:
( ) ( )
( )
4sin cos cos2 2cos2 cos 4sin 2 0
cos cos2 2sin 1 2sin 1 0
2sin 1 0 (1)
2sin 1 (cos cos2 1) 0
cos cos2 1 0 (2)
pt x x x x x x
x x x x
x
x x x
x x
⇔ − − + =
⇔ − − − =
− =
⇔ − − = ⇔  − =
1 5
(1) sin 2 2
2 6 6
x x k x k
π π
π π⇔ = ⇔ = + ∨ = +
( )( )3 2
(2) 2cos cos 1 0 cos 1 2cos 2cos 1 0 cos 1 2x x x x x x k π⇔ − − = ⇔ − + + = ⇔ = ⇔ =
Vậy phương trình có ba họ nghiệm
5
2 2 2
6 6
x k x k x k
π π
π π π= + ∨ = + ∨ =
2. Giải hệ phương trình :
( )( ) ( )2 2 2
2 2
2 11 17 2 2 6 (1)
log 1 1 1 log (2)
y x y x x
x y x x y y
 + + + + − = +

 − + + −= +  
Bài giải:
Điều kiện :
( )( )2
2
1 1 0 1
00
2 11 17 02 11 17 0
2 2 02 2 0
x y x
yx y
y xy x
y xy x
 − + >  >
 
> > ⇔ 
+ + ≥+ + ≥ 
  + − ≥+ − ≥ 
(*)
( ) ( )2 2 2
2 2 2 2(2) log 1 log 1 log log 1x x x y y y⇔ − − + −= − + +
( )
2
2
2 22
( 1)
log 1 log (3)
( 1) 1 1
x y
x y
x y
 −
⇔ + −= + 
− + + 
Xét hàm số: ( )2( ) log 0 ,
1
t
f t t t
t
= + ∀ + ∞
+
Ta có : ( )
1
'( ) 1 0 0 ,
( 1)ln 2
f t t
t t
= + > ∀ ∈ + ∞ ⇒
+
( )f t đơn điệu tăng trên ( )0 , + ∞
Do đó 2 2
(2) ( ) ( 1) 1f y f x y x⇔ = − ⇔ = −
Thay 2
1y x= − vào phương trình (1) ta có: 2 2
2 11 15 2 3 6 (3)x x x x x+ + + + − = +
Dễ thấy x = - 3 không phải là nghiệm của phương trình
Xét 3x ≠ − ta có:
( )( ) ( )( )2 2
(3) 3 2 11 15 2 3 3 6x x x x x x x⇔ + + + + + − = + +
( )( )
2 2
2 2 2 2
3 2 11 15 2 3 2 11 15 2 3 0x x x x x x x x x ⇔ + + + + + − − + + − + − = 
 
2 2
2 2
2 11 15 2 3 0 3( )
2 11 15 2 3 3 (4)
x x x x x loai
x x x x x
 + + + + − = ⇔ =−
⇔
 + + − + − = +

Lây (3) trừ (4) vế theo vế ta có phương trình hệ quả :
2 2 7 3
2 2 3 3 4 8 21 0
2 2
x x x x x x+ − =⇔ + − =⇔ =− ∨ =
Thay
7
2
x = − và
3
2
x = vào (3) đều thỏa mãn .
Với
7 45
2 4
x y=− ⇒ =
Với
3 5
2 4
x y= ⇒ =
Vậy hệ phương trình có hai nghiệm: ( )
7 45
; ;
4 4
x y
 
= − 
 
và ( )
3 5
; ;
4 4
x y
 
=  
 
Bình luận:
 Ở cấu trúc pt(2) ta thấy vừa chứa biến trong lôga vừa chứa biến ở dạng đa thức (tức là có hai dạng hàm
khác nhau cùng xuất hiện trong một phương trình). Vì thế để giải được (2) thì nhất định ta phải dùng
phương pháp hàm số hoặc phương pháp đánh giá (không sử dụng biến đổi đại số thông thường được).
Lại thấy (2) có hai biến có thể cô lập ở hai vế khác nhau được nên ta nghĩ ngay đến dạng toán sử dụng
tính đơn điệu với cấu trúc ( ) ( )f u f v= (với u, v là các biểu thức theo x, y)
 Ở pt(3) ta nhận thấy :
2 2
2 2 2
2 11 15 2 3 9 18 ( 3)( 6)x x x x x x x x+ + − + − = + + = + + nên ta nghĩ ngay đến
phép nhân liên hợp.
Ngoài ra pt(3) còn có thể giải cách khác: ( )2 2
2 11 15 ( ) 2 3 6 0x x x m x x m   + + − + + + − − − =
   
xét
ĐNT: 2 2 2 2
(11 2 ) 15 2 39 12 ,x m x m x x m m x R+ − + − = + − + − ∀ ∈ ⇒
9
2
m = sau đó ta nhân liên hợp
Hoặc : Đặt ( )
2
2 2
2
2 11 15
3 ( 6)
2 3
u x x
u v x x
v x x
 = + +
⇒ − = + +
= + −
khi đó
6
(3)
3
u v x
u v x
+ = +
⇒ 
− = +
Câu 3 (1,0 điểm).
1. Tính tích phân :
2 2
3
1
(1 )ln
e
x x x
I dx
x
+ +
= ∫
Bài giải:
2 2
1 2 33 3
1 1 1 1
ln ln ln ln 1
e e e e
x x x x x x
I dx dx dx dx I I I
x x x x
+ +
= = + + = + +∫ ∫ ∫ ∫
Giải 1I :
3 2
1
ln
1 1
2
u x du dx
x
dv dx v
x x

= ⇒ =

 = ⇒ =−

1 2 3 2 2 2 2 2
1
1 1 1 1 1 1 1 1 1 1 3
ln
1 12 2 2 4 2 4 4 4 4
e
e e
I x dx
x x e x e e e
=− + =− − =− − + =−∫
2
2
1
ln 1
ln (ln )
12 2
e
ex
I x d x= = =∫ 3 ln 1
1
e
I x= =
Vậy 2
7 3
4 4
I
e
= −
2. Tính thể tích khối tròn xoay tạo thành khi quay hình phẳng được giới hạn bởi các đường
1
, 0, 0
1 4 3
y y x
x
= = =
+ −
và 1x = xung quanh trục hoành.
Bài giải:
Thể tích của khối tròn xoay theo đề được tính theo công thức :
21
0
1
1 4 3
V dx
x
π
 
=  
+ − 
∫
Đặt: 2 2
1 4 3 4 3 ( 1) 3 2( 1) ( 1)
3
t x x t dx t dt dx t dt= + − ⇔ − = − ⇒ − = − ⇔ = − −
Đổi cận: 1 2x t= ⇒ = 0 3x t= ⇒ =
2 3
2 2
3 2
32 1 2 1 1 2 1 2 1
ln ln3 ln 2
23 3 3 3 6
t
V dt dt t
t t t t
π π π π−      
=− = − = + = − −     
     
∫ ∫ (đvtt)
Câu 4 (1,0 điểm).
1. Cho số phức z thỏa mãn 2 3z z i+ = + . Tính tổng: 2014 2015 2016
T z z z= + +
Bài giải: Gọi ( ),z x yi x y R=+ ∈
( )
1
2 3 2 3 3 3 1
1
x
z z i x yi x yi i x yi i z i
y
=
+ = + ⇔ + + − = + ⇔ − = + ⇔ ⇒ = −
= −
Ta có :
( )
3
2
24
1
1 1
i i
i
i
 = −
=− ⇔ 
=− =
( ) ( ) ( )
10071007 25122014 2 1007 1007 1007 4 3 1007
1 2 2 2z z i i i i i = =− =− =− =
 
( ) ( )2014 2 1007 1007 1008
1 2 1 1 2 3.2 2T z z z i i i i= + += + − −= +
2. Gọi E là tập hợp các số tự nhiên có ba chữ số sao cho số đứng sau lớn hơn số đứng trước. Lấy
ngẫu nhiên từ tập E ra một số , Tính xác suất để số được chọn là một số chẵn.
Bài giải:Gọi 1 2 3x x x x= với 1 2 3x x x< <
Từ đó ta thấy trong số x không có chữ số 0 và cứ mỗi cách chọn ra một bộ 3 chữ số khác
nhau từ tập hợp X = {1, 2, 3, 4, 5, 6, 7, 8, 9} thì ta chỉ thiết lập được duy nhất một số x
Số phần tử của tập E là số cách chọn 3 chữ số từ tập X ⇒ số phần tử của tập E là: 3
9 84E C= =
Gọi Y là tập con của E và chỉ chứa các số chẵn :
Để tính số phần tử của tập Y ta chia thành 3 trường hợp sau (vì 3 2x ≠ )
TH1: 3 4x ≡ khi đó 1 2,x x chỉ được chọn từ {1,2,3} ⇒ có 2
3C số x có chữ số cuối là 4.
TH2: 3 6x ≡ khi đó 1 2,x x chỉ được chọn từ {1,2,3,4,5} ⇒ có 2
5C số x có chữ số cuối là 6.
TH3: 3 8x ≡ khi đó 1 2,x x chỉ được chọn từ {1,2,3,4,5,6,7} ⇒ có 2
7C số x có chữ số cuối là 8.
Theo quy tắc cộng ta có số phần tử của tập X là: 2 2 2
3 5 7 34X C C C= + + =
Gọi Ω là không gian mẫu của phép thử lấy một số từ tập E ⇒ số phần tử củaΩ là 84Ω =
Gọi A là biến cố lấy ra được số chẵn (tức là lấy một số từ Y)⇒số phần tử của AΩ là 34AΩ =
Vậy xác suất của biến cố A là
34 17
( )
84 42
A
P A
Ω
= = =
Ω
Câu 5 (1,0 điểm). Cho hình lăng trụ tam giác ABCA’B’C’ có AB = a, BC = 2a,  0
30ACB = , hình
chiếu vuông góc của A’ lên mặt phẳng (ABC) trùng với trọng tâm G của tam giác ABC và góc
giữa AA’ tạo với mặt phẳng (ABC) bằng 0
60 . Tính theo a thể tích của khối đa diện BCC’B’A’
và khoảng cách giữa hai đường thẳng B’C’ và A’C.
Bài giải:
Gọi M là trung điểm BC.Đặt : AC x=
Ta có AG là hình chiếu của AA’ lên (ABC)
⇒ ( ) ( ) ',( ) ', 'AA ABC AA AG A AG= =
 0
' 60gt A AG⇒ =
Theo định lý cosin trong ABC∆ ta có:

( ) ( )
2 2 2
22 2 0 2 2
2 . .cos
2 2 . 2 .cos30 2 3 3 0 3 3
AB CA CB CACB ACB
a x a x a x ax a x a AC a
= + −
⇔ = + − ⇔ − + = ⇔ = ⇒ =
Ta có: 2 2 2 2 2 2
3 4AB AC a a a BC ABC+ = + = = ⇒ ∆ vuông tại A.
2 2 2
3 3 2 3
BC a
AG AM⇒ = = =
Xét 'AA G∆ ta có:  ' 2 3
tan ' ' tan '
3
A G a
A AG A G AG A AG
AG
= ⇔ = =
Thể tích khối lăng trụ ABCA’B’C’ là 31 1 2 3
' . ' . . . 3
2 2 3
ABC
a
V A G S A G AB AC a a V a= = = ⇔= (đvtt)
Ta có B’C’ // BC ⇒ B’C’ // (A’BC) ( ) ( ) ' '
'
3
' ', ' ',( ' ) B A BC
A BC
V
d B C A C d B A BC
S
⇒ = =
Lại có:
3
' ' ' ' ' ' ' ' ' '
1 1 1
3 3 3 3
B A BC A ABC CA B C B A BC B A BC
a
V V V V V V V V V= + + = + + ⇔ = =
Theo Pita go ta có:
2
22 7
3 2 3
AC a
GB AB
 
= + = 
 
2
22 13
3 2 3
AB a
GC AC
 
= + = 
 
2 2 19
' '
3
a
A B GA GB= + = 2 2 5
' '
3
a
A C GA GC= + =
Theo định lý cosin:
  
2 2 2
2' ' 7 51
' sin ' 1 cos '
2 '. 10 10
CA CB A B
CosBCA BCA BCA
CA CB
+ −
= =⇒ =− =

2
'
1 51
. 'sin '
2 6
A BC
a
S CB CA BCA= =
( )
3
' '
2
'
3 6 3 51
' ', '
1751
B A BC
A BC
V a a
d B C A C
S a
= = = (đvđd)
A
A’
C B
B’
C’
0
30
0
60 a
2a
G
M
Câu 6( 1,0 điểm): Trong không gian với hệ trục tọa độ Oxyz, cho các điểm A(1; 1; -1), B(1; 1; 2),
C(-1; 2; -2) và mặt phẳng (P) có phương trình x – 2y + 2z +1 = 0. Viết phương trình mặt phẳng đi
qua A , vuông góc mặt phẳng (P) và cắt đoạn thẳng BC tại điểm I sao cho IB = 2IC
Bài giải :
Gọi (Q) là mặt phẳng cần tìm
Ta có : điểm I thuộc đoạn thẳng BC sao cho IB = 2IC 2 2IB IC IC BC IC⇒ =− ⇔ + =−
    
[ ]
( )
[ ]
1 ( 1) 3 ( 1)
1 5 2 4 2 1
3 1 2 3 2 ; ; ; ;
3 3 3 3 3 3
2 ( 2) 3 ( 2)
I
I
I
x
CB CI y I IA
z
 − −= − −
    
⇔ = ⇔ − = − ⇒ − − ⇒ = − −    
   
− −= − −
  
(P) :x – 2y + 2z +1 = 0 ( )P⇒ có VTPT là ( )1; 2;1Pn= −

Mặt phẳng (Q) qua A, I và vuông góc với (P)
4 5 6
, ; ;
3 3 3
Pn n AB
  ⇒= =     
  
là VTPT của(Q)
Do đó mặt phẳng (Q) qua A và có VTPT là n

nên: ( ): 4 5 6 3 0Q x y z+ + − =
Vậy mặt phẳng cần tìm có phương trình là :4 5 6 3 0x y z+ + − =
Câu 7 (1,0 điểm). Trong mặt phẳng với hệ trục tọa độ Oxy, cho tam giác ABC có trực tâm H(5; 5) và
đường thẳng BC có phương trình x + y – 8 = 0. Đường tròn ngoại tiếp tam giác ABC đi qua điểm
( )1; 5D . Tìm tọa độ các đỉnh của tam giác ABC, biết rằng độ dài cạnh BC bằng 4 2 và hoành độ
của điểm B lớn hơn 3.
Bài giải :
Gọi đường tròn ngoại tiếp ∆ABClà (C) có tâm I (a; b) và bán kính R
Gọi E là giao điểm khác A của AH và (C)
Ta có :  EBC EAC= (cùng chắn cung EC )
mà  HBC EAC= (cùng phụ với góc ACB )
BC⇒ là đường phân giác của HBE
⇒K, H đối xứng nhau qua BC.
AH qua H và vuông góc BC : 0AH x y⇒ − =
Gọi K là giao điểm của AH và BC ( )(4;4) 3;3K E⇒ ⇒
.
A
B C
H(5; 5)
I
D(1; 5)
E
K
8 0x y+ − =
4 2BC =
Ta có: ( ) ( )
2 22 2
3 ( 3) 1 ( 5) 2 ( ; 2)IE ID a b a b b a I a a= ⇔ − + − = − + − ⇔ = + ⇒ +
Theo Pitago ta có: [ ]
2
22 2
( , )
2
BC
IE R d I BC
 
= = +  
 
( )
22
2 2
2 2
(4; 6)2 8 4 2
3 ( 1) 4
2 101 1
Ia a
a a a
R IE
   + + −  
⇔ − + − = + ⇔ = ⇒      = =+   
( )
2 2
( ) : 4 ( 6) 10C x y⇒ − + − =
A là giao điểm khác E của AH và (C)⇒A(7; 7)
B, C là hai giao điểm của BC và (C)
⇒tọa độ B,C là nghiệm của hệ: 2
8 5 1
3 710 21 0
x y x x
y yy y
= − = =  
⇔ ∨  
= =− + =  
mà hoành độ điểm B lớn hơn 3 nên: (5;3)B và (1;7)C
Vậy tam giác ABC có ba đỉnh là A(7; 7) , (5;3)B và (1;7)C
Bình luận:
• Với những bài toán có yếu tố đường thẳng cắt đường tròn thì nhất định ta phải sử dụng công thức
bitago: [ ]
2
22
( , )
2
BC
R d I BC
 
= +  
 
để viết phương trình đường thẳng hoặc đường tròn
• Với bài toán cho trực tâm, đường tròn ngoại tiếp tam giác
+ Nếu đề cho phương trình một cạnh thì ta phải đối xứng trục tâm qua cạnh đó để có thêm
điểm thuộc đường tròn ngoại tiếp .
+ Nếu đề cho trung điểmM của cạnh BC thì ta dựng dường kính AA’ sau đó chứng minh
BHCA’ là hình bình hành , suy ra M là trung điểm của HA’
Câu 8 (1,0 điểm).Giải hệ phương trình:
( )( )
( )3 2 2
2 3 1 4 1 0 (1)
1
2 5 4 1 4 (2)
x y y x y
x x x x x y x
x
 − + + + + + + =


 + + += + + +

Bài giải :
Điều kiện:
2 3 0
4 0
1 0
y
x y
x x
+ ≥

+ + ≥
 ≤ − ∨ ≥
(*)
( ) ( )(1) 1 2 3 4 2 3 1 0x y y x y y x y⇔ − + + + + + − + + − + =
( )
1
1 2 3 1 0 1 0 1
4 2 3
x y y x y y x
x y y
 
⇔ − + + + + = ⇔ − + = ⇔ = +  + + + + 
Vì
1
2 3 1 0
4 2 3
y
x y y
+ + + >
+ + + +
với mọi ,x y thỏa ĐK (*)
Thay 1y x= + vào (2) ta có: ( )3 2 2 1
2 5 4 1 2 5x x x x x x
x
+ + += + +
( ) ( )2 2 2 21 1
2 5 4 2 5 2 5 5 4 0 (3)x x x x t x t x x
x x
⇔ + + + = + + ⇔ − + + + + = với 2 1
t x
x
= +
Xem (3) là phương trình bật hai theo biến t và xem x như là một tham số ta có:
( ) ( )2 2
2 5 4 5 4 9t x x x∆= + − + + =
2 5 3
1 (4)
2
(3)
2 5 3
4 (5)
2
x
t x
x
t x
+ −
= = +
⇔ 
+ + = = +

2
2 2 2
1 1 0
11
(4) 1 1 1 3
2 1 2 1 0
2 2
x x y
x
x x
x x x x x yx x
x
≥ − =− ⇒ = ≥ − ⇔ + = + ⇔ ⇔ ⇔ 
+ = + + = ⇒ =+ − = 
2
2 2 2
3 2 3 2
4 111 4 4(5) 4 1
8 16 8 16 1 0 3 2 3 2
1
4 4
x x yx
x x
x x x x x x
x x y

≥ − =− − ⇒ =− ≥ − ⇔ + = + ⇔ ⇔ ⇔ 
+ = + + + − = =− + ⇒ =

Vậy hệ phương trình có 4 nghiệm:
1
0
x
y
= −

=
,
1
2
3
2
x
y

=

 =

,
3 2
1
4
3 2
4
x
y

=− −


 = −
và
3 2
1
4
3 2
4
x
y

=− +


 =
Bình luận:
• Để có thể thêm bớt được hợp lý trong cách giải của phương trình (1) thì trước tiên ta phải dự đoán
được nhóm chung sẽ xuất hiên sau khi nhân liên hợp là gi. Ta sẽ tìm nó bằng cách sau:
Xét 1x = thì ( )( )(1) 1 2 3 1 5 1 0 2y y y y⇔ − + + + + + = ⇒ =
Xét 2x = thì ( )( )(1) 2 2 3 1 6 1 0 3y y y y⇒ − + + + + + = ⇒ = Từ đó nhóm
chung ax by c+ + có hai nghiệm ( )1, 2 và ( )2, 3
2 0
2 3 0
a b c a c
a b c b c
+ += = 
⇒ ⇔ 
+ + = =− 
vậy nhóm chung là 1cx cy c x y− + = − +
• Ngoài ra phương trình (1) cũng có thể giải bằng phép đặt ẩn phụ như sau:
2 2
2 3 0
1
4 0
u y
x y v u
v x y
= + ≥
⇒ − = − −
= + + ≥
.
( )( ) ( )( ) ( )2 2 2 2
(1) 1 1 1 0 1 0v u u v v u u v u⇔ − − + + + = ⇔ − + + − =
( ) ( )( )1 1 0 4 2 3 1v u u v u v u x y y y x⇔ − + + + = ⇔ = ⇔ + + = + ⇔ = +  
• Ở phương trình (3) ta thấy có nhóm 2 1
x
x
+ là nổi bật nhất . Rỏ ràng nhóm này ta không thể phân
tích được (sai lầm phổ biến:
3 3
2 1 1 1x x
x
x x x
+ +
+= = khi đó điều kiện bị thay đổi ) Vì thể nên
ta phải nghĩ đến việc biến đổi các số hạng còn lại để có được cấu trúc đồng dạng với 2 1
x
x
+ sau
đó đặt ẩn phụ .Nếu may mắn thì ta sẽ chuyển tất cả qua ẩn phụ được còn không ta phải đối diện với
phương trình hai biến x và t khi đó chỉ còn một hướng để đi là phải chuyển được về phương trình
tich.
Câu 9 (1,0 điểm). Tìm giá trị nhỏ nhất của biểu thức :
2
2 2 2 2 20 9 42
2 1 4 2 1 3 (3 4 )
5
y y
P x y x x y x x x y
− +
= + − + + + + + + − +
Bài giải :
( ) ( ) ( ) ( )
2 2 2 22 9 42
1 1 2 3 2
5 5
P x y x y x y y= − + + − − + + − − +
Xét các vectơ: ( ) ( ) ( )1; , 1; 2 2; 3u x y v x y u v y= − = − − ⇒ + = −
   
Ta có: ( ) ( ) ( )
2 2 22 2
1 1 2 4 9u v u v x y x y y+ ≥ + ⇔ − + + − − + ≥ +
   
Khi đó ta có: 2 9 42
4 9
5 5
P y y≥ + − + (vì ( )
2
3 2 0x y− ≥ )
Xét hàm sô: 2 9 42
( ) 4 9
5 5
f y y y= + − +
Ta có:
( )2
2 2
9 5 4 99 9
'( )
54 9 5 4 9
y yy
f y
y y
− +
= −=
+ +
1
'( ) 0
2
f y y= ⇔ = ( lại có '(0) 0f < và '(1) 0f > )
BBT:
Từ BBT ta thấy:
( ) 10Min y = khi
1
2
y =
Suy ra 10MinP = khi
1
3
x = và
1
2
y =
--- Hết ---
ĐỀ SỐ 5
Câu 1 (2,0 điểm). Cho hàm số : 3 23
6 10
2
y x x x= − − + có đồ thị (C)
1. Khảo sát sự biến thiên và vẽ đồ thị (C) của hàm số trên
2. Sử dụng đồ thị (C) của hàm số trên để biện luận theo m số nghiệm của phương trình:
( )( )
10 2 10
2 2 1
x m
x x
x
+ −
− + =
Câu 2 (1,0 điểm).
1. Giải phương trình:
1
2tan cot 2 3
sin 2
x x
x
+ = +
2. Giải bất phương trình:
2 3
2 3
2
log ( 1) log ( 1)
0
3 4
x x
x x
+ − +
>
− −
Câu 3 (1,0 điểm). Tính tích phân:
ln2
0
1
1x
I dx
e
=
−
∫
Câu 4(1,0 điểm):
1. Cho số phức z thỏa mãn điều kiện: ( )1 2 2(1 2 )z i z i+ − = − . Tìm phần thực và phần ảo của số phức
2
3w z z= −
2. Cho tập E = {1; 2; 3; 4; 5}. Viết ngẫu nhiên lên bảng hai số tự nhiên , mỗi số gồm ba chữ số đôi
một khác nhau thuộc tập E . Tính xác suất để trong hai số đó có đúng một số chẵn.
y
'( )f y
( )f y
1
2
0- +
10
Câu 5(1,0 điểm): Cho hình chóp SABCD có đáy ABCD là hình chữ nhật có AB =a và SA vuông góc đáy .
đường thẳng SC tạo với mặt phẳng (ABCD) một góc 300
. Gọi M là hình chiếu của A lên đường
thẳng SB. Biết
3
3
a
AM = . Tính theo a thể tích của khối chóp SABCD và khoảng cách từ điểm
M đến mặt phẳng (SCD).
Câu 6(1,0 điểm):Trong không gian với hệ trục Oxyz, cho mặt phẳng (P): 2x + 2y + z – 5 = 0 và đường
thẳng
2 2 3
:
1 1 2
x y z
d
− − −
= = . Gọi ∆ là đường thẳng vừa cắt vừa vuông góc với d. Viết phương
trình đường thẳng ∆ . Biết khoảng cách từ ∆ đến mặt phẳng (P) bằng 10.
Câu 7(1,0 điểm): Trong mặt phẳng với hệ trục tọa độ Oxy, cho hình thang ABCD với cạnh đáy AB và
CD = 2AB. Gọi H là chân đường vuông góc hạ từ D xuống AC và M là trung điểm của HC. Biết tọa
độ đỉnh B(5; 6), phương trình đường thẳng DH: 2x – y = 0, phương trình đường thẳng
DM: x – 3y + 5 = 0, tìm tọa độ các đỉnh của hình thang ABCD.
Câu 8(1,0 điểm): Giải hệ phương trình :
2 2
4 ( 5) 1 4 2 2
4 ( 4) 2 1
x y x y x y
y x x x
 + − −= − +

− + = −
Câu 9(1,0 điểm): Cho x, y, z là các số thực dương thỏa mãn xyz + x + z = y . Tìm giá trị lớn nhất của
biểu thức
( )2 2 2 2 2
2 2 4 3
1 1 1 1 1
z z
P
x y z z z
= − − +
+ + + + +
--- Hết ---
GIẢI ĐỀ 5
Câu 1 (2,0 điểm). Cho hàm số : 3 23
6 10
2
y x x x= − − + có đồ thị (C)
1. Khảo sát sự biến thiên và vẽ đồ thị (C) của hàm số trên
Bài giải:
• Tập xác định : D = R
• Sự biến thiên:
2
' 3 3 6y x x= − − ; ' 0 1y x= ⇔ =− hoặc 2x =
Chiều biến thiên: Hàm số đồng biến trên các khoảng :( ); 1−∞ − và ( )2 ; + ∞
Hàm số ngịch biến trên khoảng :( )1; 2−
Cực trị : Hàm số đạt cực đại tại x = -1 với giá trị cực đại là
27
2
CDy =
Hàm số đạt cực tiểu tại x = 2 với giá trị cực tiểu là 0CTy =
Giới hạn tại vô cực : lim
x
y
→−∞
= −∞ ; lim
x
y
→+∞
= +∞
Bảng biến thiên:
• Đồ thị :
'' 6 3y x= − ;
1
'' 0
2
y x= ⇔ = . Đồ thị có điểm uống là
1 27
;
2 4
I
 
 
 
Đồ thị có hai điểm chung với Ox là
5
; 0
2
 
− 
 
và ( )2; 0 và có điểm chung với Oy là ( )0;10
2. Sử dụng đồ thị (C) của hàm số trên để biện luận theo m số nghiệm của phương trình:
( )( )
10 2 10
2 2 1 (*)
x m
x x
x
+ −
− + =
Bài giải:
Điều kiện: 0x ≠
Khi đó ta có: ( ) 2 10 2 10
* 2 3 2
x m
x x
x
+ −
⇔ − − = 3 2
2 3 12 10 2x x x m⇔ − − + =
3 23
6 10 5 (1)
2
x x x m⇔ − − + = +
−∞ +∞-1 2
0 0+ - +
−∞
+∞27
2
0
x
'y
y
0-15
2
− 2
.. .
.
y
x
27
2
Suy ra : (1) là phương trình hoành độ giao điểm của đồ thị (C) và đường thẳng d: y = m + 5 (luôn song
song với trục Ox).
Nên số nghiệm của (1) là số giao điểm của đường thẳng d và đồ thị (C).
Từ đồ thị (C) ta có:
• Với 5m < − (tức là: 5 0m + < ) thì d và (C) có một điểm
chung duy nhất nên (1) có nghiệm duy nhất
• Với m = - 5 (tức là : m + 5 = 0) thì d và (C) có hai điểm chung
nên (1) có hai nghiệm
• Với
17
0
2
m< < (tức là:
27
0 5
2
m< + < ) thì d và (C) có 3 điểm chung
nên (1) có ba nghiệm.
• Với
17
2
m = ( tức là:
27
5
2
m + = ) thì d và (C) có 2 điểm chung
nên (1) có hai nghiệm.
• Với
17
2
m > ( tức là:
27
5
2
m + > ) thì d và (C) có 1 điểm chung
duy nhất, nên (1) có một nghiệmduy nhất.
Mặt khác (1) có nghiệm x = 0 ⇔ m + 5 = 10 ⇔ m = 5
Kết luận:
Khi
5
17
2
m
m
< −

 >

thì phương trình (*) có một nghiệm duy nhất
Khi
5
17
2
5
m
m
m
= −

 =

 =
thì phương trình (*) có hai nghiệm phân biệt.
Khi
17
5
2
5
m
m

− < <

 ≠
thif phương trình (*) có ba nghiệm phân biệt
Câu 2 (1,0 điểm).
1. Giải phương trình:
1
2tan cot 2 3
sin 2
x x
x
+ = +
Bài giải:
Điều kiện: sin 2 0 ( ) (*)
2
x x k k Z
π
≠ ⇔ ≠ ∈
0-15
2
− 2
.. .
.
y
x
27
2
10
Ta có:
sin cos2 cos2 cos sin 2 sin cos 1
tan cot 2
cos sin 2 cos sin 2 cos sin 2 sin 2
x x x x x x x
x x
x x x x x x x
+
+ = + = = =
1
tan tan cot 2 3
sin 2
pt x x x
x
⇔ + + = +
1 1
tan 3
sin 2 sin 2
x
x x
⇔ + = +
tan 3
3
x x k
π
π⇔ = ⇔ = + ( thỏa điều kiện (*))
Vây phương trình có một họ nghiệm duy nhất là:
3
x k
π
π= + với k Z∈
2. Giải bất phương trình:
2 3
2 3
2
log ( 1) log ( 1)
0 (1)
3 4
x x
x x
+ − +
>
− −
Bài giải:
Điều kiện: 2
1 0 1
(*)
43 4 0
x x
xx x
+ > > − 
⇔ 
≠− − ≠ 
Khi đó ta có:
( )
( )( )
2 32 3
2
log ( 1) 2 3log 22log ( 1) 3log ( 1)
(1) 0 0
3 4 1 4
xx x
x x x x
+ −+ − +
⇔ > ⇔ >
− − + −
( )
2log ( 1)
0
4
x
x
+
⇔ >
−
Vì 32 3log 2 0
1 0x
− >

+ >
2log ( 1) 0
4 0
x
x
+ >
⇔ 
− >
hoặc
2log ( 1) 0
4 0
x
x
+ <

− <
0
4
x
x
>
⇔ 
>
hoặc
0
4
x
x
<

<
0
4
x
x
<
⇔  >
Kết hợp với điều kiện (*) ta có
1 0
4
x
x
− < <
 >
Vậy bất phương trình có tập nghiệm là ( ) ( )1, 0 4 ,S = − ∪ + ∞
Câu 3 (1,0 điểm). Tính tích phân:
ln2
0
1
1x
I dx
e
=
−
∫
Bài giải:
ln 2
0
1
1
x
x x
I e dx
e e
=
−
∫
Đặt: 2
1 1 2x x x
u e u e udu e dx= − ⇔ + = ⇔ =
Đổi cận: ln 2 1x u= ⇒ =
0 0x u= ⇒ =
( )
1 1
22
0 0
1 1
2 2
11
I udu du
uu u
= =
++∫ ∫
Đặt: tanu t= với ,
2 2
t
π π 
∈ − 
 
( )2
2
1
tan 1
cos
du dt t dt
t
⇒ = = +
Đổi cận 1
4
u t
π
= ⇒ =
0 0u t= ⇒ =
( )
4 4
2
2
0 0
1
2 tan 1 2 2 4
tan 1 2
0
I t dt dt t
t
π π
π
π
= + = = =
+∫ ∫
ĐS:
2
I
π
=
Câu 4(1,0 điểm):
1. Cho số phức z thỏa mãn điều kiện: ( )1 2 2(1 2 )z i z i+ − = − . Tìm phần thực và phần ảo của số phức
2
3w z z= −
Bài giải:
Gọi ( , )z x yi x y R=+ ∈
( ) ( ) ( )( )1 2 2(1 2 ) 1 2 2(1 2 )z i z i x yi i x yi i+ − = − ⇔ + + − − = −
( ) ( ) ( )2 2 2 2 4x y x i i⇔ − + − = + −
2 2 2 1
2
2 4 2
x y y
z i
x x
−= = 
⇔ ⇔ ⇔ = + 
− =− = 
Khi đó: ( )
22
3 2 3(2 ) 3w z z i i i= − = + − + =− +
Vậy w có phần thực là 3− và phần ảo là 1
2. Cho tập E = {1; 2; 3; 4; 5}. Viết ngẫu nhiên lên bảng hai số tự nhiên , mỗi số gồm ba chữ số đôi
một khác nhau thuộc tập E . Tính xác suất để trong hai số đó có đúng một số chẵn.
Bài giải:
Gọi X là tập hợp tất cả các số có 3 chữ số khác nhau lấy từ tập E ⇒số phần tử của X là 3
5 60X A= =
Ta thấy trong 60 số của tập X bao gồm 2
42 24A× = số chẵn và 60 24 36− = số lẽ
Gọi Ω là không gian mẫu của phép thử theo đề (tức là phép thử chọn hai số từ tập X)
Suy ra số phần tử của không gian mẫu là: 2
60 1770CΩ= =
Gọi A là biến cố trong hai số lấy ra có đúng một số chẵn
Suy ra số phần tử của không gian biến cố A là 1 1
24 36 864A C CΩ = × =
Vậy Xác suất của biến cố A là:
864 144
( )
1770 295
A
P A
Ω
= = =
Ω
Câu 5(1,0 điểm): Cho hình chóp SABCD có đáy ABCD là hình chữ nhật có AB = a và SA vuông góc đáy .
đường thẳng SC tạo với mặt phẳng (ABCD) một góc 300
. Gọi M là hình chiếu của A lên đường
thẳng SB. Biết
3
3
a
AM = . Tính theo a thể tích của khối chóp SABCD và khoảng cách từ điểm
M đến mặt phẳng (SCD).
Bài giải:
Ta có ( )SA ABCD⊥ ⇒ AC là hình chiếu cuae SC lên mp(ABCD)
( ) ( ) ,( ) ,SC ABCD SC AC SCA⇒ = = (vì  0
90SAC = )
 0
30gt SCA⇒ =
Xét SAB∆ ta có: 2 2 2 2
1 1 1 2
2
a
SA
AS AH AB a
= − = ⇒ =
Xét SAC∆ ta có:  6
.cot
2
a
AC SA SCA= =
Theo Pitago cho ABC∆ ta có: 2 2
2
a
BC AC AB= − =
3
1 1
. . .
3 3 6
SABCD ABCD
a
V SA S SA AB AD= = =
S
A
B C
D
M
K
0
30
3
3
a
a
Xét SAB∆ ta có:
2 2
2
2 2 2
1
.
3
SM SA SA
SM SB SA
SB SB SA AB
= ⇔ = = =
+
Mặt khác BM cắt mp(SCD) tại S nên theo talet ta có:
( )
( )
( ) ( ) ( )
,( ) 1 1 1
,( ) ,( ) ,( ) (1)
,( ) 3 3 3
d M SCD MS
d M SCD d B SCD d A SCD
d B SCD BS
==⇔ = = (vì AB // (SCD))
Ta có: ( )
( ) ( )
( ) ( )
SCD SADCD SA
CD SAD
CD AD SCD SAD SD
⊥⊥ 
⇒ ⊥ ⇒ 
⊥ ∩ = 
Dựng AK SD⊥ (với K SD∈ )
( )AK SCD⇒ ⊥ ⇒ K là hình chiếu vuông góc của A lên mp(SCD) ( ),( ) (2)d A SCD AK⇒ =
Xét SAD∆ ta có: 2 2 2 2
1 1 1 4
2
a
AK
AK AD AS a
= + = ⇔ = (3)
Từ (1), (2) và (3) ta có: ( ),( )
6
a
d M SCD =
Vậy thể tích khối chóp SABCD là
3
6
SABCD
a
V = (đvtt) và ( ),( )
6
a
d M SCD = (đvđd)
Câu 6(1,0 điểm):Trong không gian với hệ trục Oxyz, cho mặt phẳng (P): 2x + 2y + z – 5 = 0 và đường
thẳng
2 2 3
:
1 1 2
x y z
d
− − −
= = . Gọi ∆ là đường thẳng vừa cắt vừa vuông góc với d. Viết phương
trình đường thẳng ∆ . Biết khoảng cách từ ∆ đến mặt phẳng (P) bằng 10.
Bài giải:
Mặt phẳng (P): 2x + 2y + z – 5 = 0 ⇒ (P) có VTPT là ( )2; 2;1Pn =

Đường thẳng
2
: 2
3 2
x t
d y t
z t
= +

= +
 = +
⇒ d có VTCP là ( )1;1; 2dn =

( )
( ) / /
, 3; 3; 0p d
P
gt u n u
d
∆
∆
 ⇒ ⇒ = = −  ⊥ ∆
  
là VTCP của ∆
Gọi M là giao điểm của d và ∆ ( )2 ; 2 ; 3 2M t t t⇒ + + + ∈∆
2 2 2
4 (6 ; 6 ;11)4 2 4 2 3 2 5
( ,( )) 10 10 1 5
6 ( 4 ; 4 ; 9)2 2 1
t Mt t t
d M P t
t M
= ⇒+ + + + + − 
= ⇔ = ⇔ + = ⇔  =− ⇒ − − −+ + 
Vậy có hai đường thẳng ∆ cần tìm là
6
: 6
11
x t
y t
z
= +

∆ =−
 =
và
4
: 4
9
x t
y t
z
=− +

∆ =− −
 = −
Câu 7(1,0 điểm): Trong mặt phẳng với hệ trục tọa độ Oxy, cho hình thang ABCD với cạnh đáy AB và
CD = 2AB. Gọi H là chân đường vuông góc hạ từ D xuống AC và M là trung điểm của HC. Biết tọa
độ đỉnh B(5; 6), phương trình đường thẳng DH: 2x – y = 0, phương trình đường thẳng
DM: x – 3y + 5 = 0, tìm tọa độ các đỉnh của hình thang ABCD.
Bài giải:
Gọi I là giao điểm của AC và BD
D DH DM= ∩ ⇒tọa độ D là nghiệm của hệ phương trình:
2 0
3 5 0
x y
x y
− =

− + =
( )1; 2D⇒
Theo talet ta có:
( )
1 5 3( 5) 11 14
2 3 ;
2 6 3 6 3 3
I
I
xID DC
BD BI I
yIB AB
− = −  
= =⇔ = ⇔ ⇒  −= −  
 
Đường thẳng AC đi qua I và vuông góc DH
⇒ AC qua
11 14
;
3 3
I
 
 
 
và có VTPT là ( )1; 2n =

: 2 13 0AC x y⇒ + − =
Ta có: M AC DM= ∩
⇒tọa độ D là nghiệm của hệ phương trình:
2 13 0
3 5 0
x y
x y
+ − =

− + =
29 18
;
5 5
M
 
⇒  
 
H AC DH= ∩ ⇒tọa độ H là nghiệm của hệ phương trình:
2 13 0
2 0
x y
x y
+ − =

− =
13 26
;
5 5
H
 
⇒  
 
M là trung điểm HC ( )
2 7
7; 2
2 2
C M H
C M H
x x x
C
y y y
= − =
⇔ ⇒
= − =
( )
( )
( )
7 1 2 5
2 2; 6
2 2 2 6
A
A
x
gt DC AB A
y
−= −
⇒ = ⇔ ⇒
− = −
 
Vậy các đỉnh cần tìm là : ( )2; 6A , ( )7; 2C và ( )1; 2D
Bình luận :
Với những bài toán có giả thuyết liên quan đến tỷ số độ dài thì ta nghĩ ngay đến kỹ thuật tìm điểm
A B(5; 6)
D C
2 0x y− =
3 5 0x y− + =
M
H
I
bằngđẳng thức vectơ và viết phương trình đường liên quan đến góc.
Câu 8(1,0 điểm): Giải hệ phương trình :
2 2
4 ( 5) 1 4 2 2
4 ( 4) 2 1
x y x y x y
y x x x
 + − −= − +

− + = −
Bài giải:
Điều kiện:
1
0
x
y
≥

≥
2 2
4 20 1 4 2 2 (1)
4 16 2 1 (2)
x xy y y x y
hpt
xy y x x
 + − −= − +
⇔ 
− + = −
Lấy (1) trừ (2) vế theo vế ta có:
2 2
4 1 4 2 2 2 1x y x y x y x− − − = − + − −
2 2
2 1 1 4 2 2 4 (3)x x y y y⇔ + − −= + +
Đặt :
2
2
1 0 1
22 0
u x x u
y vv y
 = − ≥ = + 
⇒ 
== ≥  
( )
22 4 2 4 2 4 2
(3) 1 2 1 2 2 2 2 2 2 (4)u u v v v u u u v v v⇔ + + − = + + ⇔ + + = + +
Xét hàm số: 4 2
( ) 2 2f t t t t= + + với [ )0 ,t∀ ∈ + ∞
Ta có: 3
'( ) 4 4 2 0f t t t= + + > , [ )0 ,t∀ ∈ + ∞ ( )f t⇒ liên tục và đơn điệu tăng trên [ )0 , + ∞
Do đó: (4) ( ) ( ) 1 2 2 1f u f v u v x y y x⇔ = ⇔ = ⇔ − = ⇔ = −
Thay 2 1y x= − vào (2) ta có: 2
2 ( 1) 8( 1) 2 1 2 9 8 2 1 (5)x x x x x x x x− − − + = − ⇔ − + = −
( )
2
2 2
17
5
1 1 17 45 2 9 8 2 1 0 2 10 0
12 2 2
2 1
2
x x
x x x x x x x
x x
− +
   
⇔ − + − − + − − − = ⇔ − + + =   
     − + − 
 
2 2
5 2 2 3 2 2
17 1 17 2 45 2 0 5 0
14 4 5 2 2 3 2 22 1
2 2 4
x y
x x x x
x x x y
  + +
= ⇒=    ⇔ − + + = ⇔ − + = ⇔ 
     − −− + −  = ⇒=     
Vậy hệ phương trình có hai nghiệm:
5 2 2
2
3 2 2
4
x
y
 +
=


+ =
và
5 2 2
2
3 2 2
4
x
y
 −
=


− =
Bình luận:
• Với những hệ có cấu trúc của hai phương trình lệch nhau thì thông thường ta sẽ biến đổi (nhân liên
hợp , đặt ẩn phụ để quy về dạng tích hoặc thuần nhất, hàm số…)một phương trình rồi thế vào phương
trình còn lại.
• Với những hệ có cấu trúc của hai phương trình tương đồng nhau thì thông thường ta phải sử lý đồng
thời cả hai phương trình cùng một lúc như sau:
+) Cộng hoặc trừ hai phương trình lại để có thể quy chúng về dạng tích hoặc cấu trúc hàm số.
+) Biến đổi hệ theo 2 nhóm biến nào đó, sau đó đặt ẩn phụ để quy hệ về dạng đơn giản hơn
• Đối với pt(5) để tìm ra nhóm
1
2
x
 
− 
 
thì ta cần xét phép thử sau:
( ) ( )
( )
2 2
2 2
2
(2 4) 4
2 9 8 2 1 0 2 10 8
2 1
x m m
x x x m x m x x x m
x m x
+ − + +
− + − + + + − − = ⇔ − + − +
+ − −
Để có được nhóm chung thì :
2
1 2 4 4 1
2 10 8 2
m m
m
m
− +
= = ⇒ =−
− −
lượng thêm bớt là
1
2
x
 
− 
 
• Ta có thể giải (5) theo cách khác như sau: ( )
22
2
(5) 2 8 8 2 1 2( 2) 1 1x x x x x x ⇔ − + = + − ⇔ − = − + 
Câu 9(1,0 điểm): Cho x, y, z là các số thực dương thỏa mãn xyz + x + z = y . Tìm giá trị lớn nhất của
biểu thức
( )2 2 2 2 2
2 2 4 3
1 1 1 1 1
z z
P
x y z z z
= − − +
+ + + + +
Bài giải:
Ta có : ( )1
1
x z
xyz x z y y xz
xz
+
+ + = ⇔ = ≠
−
Ta có:
( )
( )( ) ( )( )
2 2
22 2 2 2 2 2 2 2
2 (1 ) 22 12 2 2 2 2
(1)
1 1 1 1 1 1 1 1
1
1
z z x xxz
x y x x x z x zx z
xz
 − +−  − = − = − =
+ + + + + + + ++ 
+ 
− 
Theo BĐT bu-nhi-a-cop-xki ta có:
( ) ( ) ( )22 2 2 2 2 2 2
(1 ) 2 .(1 ) 1. 2 1 (1 ) 2 1 1 (2)z x x z x x z x x z x− + ≤ − + ≤ + − + = + +
Dấu “=” xảy ra khi
2
2
2
(1 ) 2 0
1 21 2
0
1
z x x
x xzx x
z
 − + ≥

⇔ − = −
= >

(vì x > 0 và z > 0)
Từ (1) và (2) ta có:
( )
( )( )
2 2
2 2 2 2 2
2 1 12 2 2
1 1 1 1 1
z z x z
x y x z z
+ +
− ≤ =
+ + + + +
Khi đó :
( ) ( )2 2 2 2 2 2 2
2 4 3 3 2
1 1 1 1 1 1 1
z z z z z
P
z z z z z z z
≤ − + = −
+ + + + + + +
Đặt : ( )2
0 ,1
1
z
t
z
= ∈
+
(vì : 0z > và 2 2
2
1 1
1
z
z z z
z
+ > = ⇔ <
+
)
2
2 2
2 2 2
1 1
1 1
1 1 1
z
t t
z z z
= =− ⇔ =−
+ + +
Khi đó ta có: ( )2 3
3 1 2 ( ) 3P t t t P f t t t≤ − − ⇔ ≤ =− +
Xét hàm số: 3
( ) 3f t t t=− + với ( )0 ,1t ∈
Ta có: 2
'( ) 9 1f t t=− + ,
1
'( ) 0
3
f t x= ⇔ = hoặc
1
3
x = −
Bảng biến thiên:
0 0
0 1
+ -+- -
t
1
3
1
3
−
0
-2
8
9
'( )f t
( )f t
Từ Bảng biến thiên ta có: ( )
8
( ) , 0 ,1
9
f t t≤ ∈ Dấu bằng xảy ra khi
2
1 1 2
3 3 41
z
t z
z
= ⇔ = ⇔ =
+
Vậy giá trị lớn nhất của
8
9
P = khi
1
2
2 2
x z
y

= =

 =
--- Hết---

More Related Content

What's hot

đề thi và đáp án chuyên vĩn phúc 2014
đề thi và đáp án chuyên vĩn phúc 2014đề thi và đáp án chuyên vĩn phúc 2014
đề thi và đáp án chuyên vĩn phúc 2014Oanh MJ
 
3 đề thi thử môn Toán năm 2017 có đáp án chi tiết - Mẫn Ngọc Quang
3 đề thi thử môn Toán năm 2017 có đáp án chi tiết - Mẫn Ngọc Quang3 đề thi thử môn Toán năm 2017 có đáp án chi tiết - Mẫn Ngọc Quang
3 đề thi thử môn Toán năm 2017 có đáp án chi tiết - Mẫn Ngọc Quang
haic2hv.net
 
14 đề thi thử kì thi Quốc gia 2015 có đáp án
14 đề thi thử kì thi Quốc gia 2015 có đáp án14 đề thi thử kì thi Quốc gia 2015 có đáp án
14 đề thi thử kì thi Quốc gia 2015 có đáp án
Tôi Học Tốt
 
Đề thi thử Toán - Chuyên Vĩnh Phúc 2014 lần 4 Khối B,D
Đề thi thử Toán - Chuyên Vĩnh Phúc 2014 lần 4 Khối B,DĐề thi thử Toán - Chuyên Vĩnh Phúc 2014 lần 4 Khối B,D
Đề thi thử Toán - Chuyên Vĩnh Phúc 2014 lần 4 Khối B,D
dlinh123
 
TÍCH VÔ HƯỚNG VÀ CÁC ỨNG DỤNG
TÍCH VÔ HƯỚNG VÀ CÁC ỨNG DỤNGTÍCH VÔ HƯỚNG VÀ CÁC ỨNG DỤNG
TÍCH VÔ HƯỚNG VÀ CÁC ỨNG DỤNG
DANAMATH
 
[Vnmath.com] de thi thu chuye ha tinh lan 1 2015
[Vnmath.com] de thi thu chuye ha tinh lan 1 2015[Vnmath.com] de thi thu chuye ha tinh lan 1 2015
[Vnmath.com] de thi thu chuye ha tinh lan 1 2015Marco Reus Le
 
đề Và đáp án thi thử cvp truonghocso.com
đề Và đáp án thi thử cvp   truonghocso.comđề Và đáp án thi thử cvp   truonghocso.com
đề Và đáp án thi thử cvp truonghocso.comThế Giới Tinh Hoa
 
Bộ đề thi thử Đại học môn Toán có đáp án chi tiết
Bộ đề thi thử Đại học môn Toán có đáp án chi tiếtBộ đề thi thử Đại học môn Toán có đáp án chi tiết
Bộ đề thi thử Đại học môn Toán có đáp án chi tiết
Webdiemthi.vn - Trang Thông tin tuyển sinh và Du học
 
25 Đề Thi thử quốc gia năm 2015 môn Toán Hay
25 Đề Thi thử quốc gia năm 2015 môn Toán Hay25 Đề Thi thử quốc gia năm 2015 môn Toán Hay
25 Đề Thi thử quốc gia năm 2015 môn Toán Hay
Zaj Bé Đẹp
 
đề thi vào lớp 10
đề thi vào lớp 10đề thi vào lớp 10
đề thi vào lớp 10
Hạnh Nguyễn
 
chuong 1 hinh hoc 11 - phep doi hinh dong dang bien soan cong phu - hay nhat ...
chuong 1 hinh hoc 11 - phep doi hinh dong dang bien soan cong phu - hay nhat ...chuong 1 hinh hoc 11 - phep doi hinh dong dang bien soan cong phu - hay nhat ...
chuong 1 hinh hoc 11 - phep doi hinh dong dang bien soan cong phu - hay nhat ...
Hoàng Thái Việt
 
[Vnmath.com] de thi thu toan 2015 dang thuc hua 2015
[Vnmath.com] de thi thu toan 2015 dang thuc hua 2015[Vnmath.com] de thi thu toan 2015 dang thuc hua 2015
[Vnmath.com] de thi thu toan 2015 dang thuc hua 2015Marco Reus Le
 
[Vnmath.com] de thi thpt qg 2015 quynh luu 3
[Vnmath.com]  de thi thpt qg 2015 quynh luu 3[Vnmath.com]  de thi thpt qg 2015 quynh luu 3
[Vnmath.com] de thi thpt qg 2015 quynh luu 3Dang_Khoi
 
60 đề thi thử toán của các trường thpt 2015 có đáp án chi tiết
60 đề thi thử toán của các trường thpt 2015   có đáp án chi tiết60 đề thi thử toán của các trường thpt 2015   có đáp án chi tiết
60 đề thi thử toán của các trường thpt 2015 có đáp án chi tiết
Dương Ngọc Taeny
 
300 câu hỏi trắc nghiệm tích phân và ứng dụng - Nhóm Toán
300 câu hỏi trắc nghiệm tích phân và ứng dụng - Nhóm Toán300 câu hỏi trắc nghiệm tích phân và ứng dụng - Nhóm Toán
300 câu hỏi trắc nghiệm tích phân và ứng dụng - Nhóm Toán
haic2hv.net
 
chuyên đề ôn thi chuyển cấp môn toán 2017- 2018 mới nhất
chuyên đề ôn thi chuyển cấp môn toán 2017- 2018 mới nhấtchuyên đề ôn thi chuyển cấp môn toán 2017- 2018 mới nhất
chuyên đề ôn thi chuyển cấp môn toán 2017- 2018 mới nhất
Hoàng Thái Việt
 
20 đề thi thử kì thi quốc gia có đáp án
20 đề thi thử kì thi quốc gia có đáp án20 đề thi thử kì thi quốc gia có đáp án
20 đề thi thử kì thi quốc gia có đáp án
Tôi Học Tốt
 
Toan pt.de028.2011
Toan pt.de028.2011Toan pt.de028.2011
Toan pt.de028.2011BẢO Hí
 
5 đề trắc nghiệm toán 12 khảo sát hàm số - iHoc.me | Tài liệu toán học
5 đề trắc nghiệm toán 12 khảo sát hàm số - iHoc.me | Tài liệu toán học5 đề trắc nghiệm toán 12 khảo sát hàm số - iHoc.me | Tài liệu toán học
5 đề trắc nghiệm toán 12 khảo sát hàm số - iHoc.me | Tài liệu toán học
haic2hv.net
 

What's hot (20)

đề thi và đáp án chuyên vĩn phúc 2014
đề thi và đáp án chuyên vĩn phúc 2014đề thi và đáp án chuyên vĩn phúc 2014
đề thi và đáp án chuyên vĩn phúc 2014
 
3 đề thi thử môn Toán năm 2017 có đáp án chi tiết - Mẫn Ngọc Quang
3 đề thi thử môn Toán năm 2017 có đáp án chi tiết - Mẫn Ngọc Quang3 đề thi thử môn Toán năm 2017 có đáp án chi tiết - Mẫn Ngọc Quang
3 đề thi thử môn Toán năm 2017 có đáp án chi tiết - Mẫn Ngọc Quang
 
14 đề thi thử kì thi Quốc gia 2015 có đáp án
14 đề thi thử kì thi Quốc gia 2015 có đáp án14 đề thi thử kì thi Quốc gia 2015 có đáp án
14 đề thi thử kì thi Quốc gia 2015 có đáp án
 
Đề thi thử Toán - Chuyên Vĩnh Phúc 2014 lần 4 Khối B,D
Đề thi thử Toán - Chuyên Vĩnh Phúc 2014 lần 4 Khối B,DĐề thi thử Toán - Chuyên Vĩnh Phúc 2014 lần 4 Khối B,D
Đề thi thử Toán - Chuyên Vĩnh Phúc 2014 lần 4 Khối B,D
 
TÍCH VÔ HƯỚNG VÀ CÁC ỨNG DỤNG
TÍCH VÔ HƯỚNG VÀ CÁC ỨNG DỤNGTÍCH VÔ HƯỚNG VÀ CÁC ỨNG DỤNG
TÍCH VÔ HƯỚNG VÀ CÁC ỨNG DỤNG
 
[Vnmath.com] de thi thu chuye ha tinh lan 1 2015
[Vnmath.com] de thi thu chuye ha tinh lan 1 2015[Vnmath.com] de thi thu chuye ha tinh lan 1 2015
[Vnmath.com] de thi thu chuye ha tinh lan 1 2015
 
đề Và đáp án thi thử cvp truonghocso.com
đề Và đáp án thi thử cvp   truonghocso.comđề Và đáp án thi thử cvp   truonghocso.com
đề Và đáp án thi thử cvp truonghocso.com
 
Bộ đề thi thử Đại học môn Toán có đáp án chi tiết
Bộ đề thi thử Đại học môn Toán có đáp án chi tiếtBộ đề thi thử Đại học môn Toán có đáp án chi tiết
Bộ đề thi thử Đại học môn Toán có đáp án chi tiết
 
25 Đề Thi thử quốc gia năm 2015 môn Toán Hay
25 Đề Thi thử quốc gia năm 2015 môn Toán Hay25 Đề Thi thử quốc gia năm 2015 môn Toán Hay
25 Đề Thi thử quốc gia năm 2015 môn Toán Hay
 
đề thi vào lớp 10
đề thi vào lớp 10đề thi vào lớp 10
đề thi vào lớp 10
 
chuong 1 hinh hoc 11 - phep doi hinh dong dang bien soan cong phu - hay nhat ...
chuong 1 hinh hoc 11 - phep doi hinh dong dang bien soan cong phu - hay nhat ...chuong 1 hinh hoc 11 - phep doi hinh dong dang bien soan cong phu - hay nhat ...
chuong 1 hinh hoc 11 - phep doi hinh dong dang bien soan cong phu - hay nhat ...
 
[Vnmath.com] de thi thu toan 2015 dang thuc hua 2015
[Vnmath.com] de thi thu toan 2015 dang thuc hua 2015[Vnmath.com] de thi thu toan 2015 dang thuc hua 2015
[Vnmath.com] de thi thu toan 2015 dang thuc hua 2015
 
[Vnmath.com] de thi thpt qg 2015 quynh luu 3
[Vnmath.com]  de thi thpt qg 2015 quynh luu 3[Vnmath.com]  de thi thpt qg 2015 quynh luu 3
[Vnmath.com] de thi thpt qg 2015 quynh luu 3
 
60 đề thi thử toán của các trường thpt 2015 có đáp án chi tiết
60 đề thi thử toán của các trường thpt 2015   có đáp án chi tiết60 đề thi thử toán của các trường thpt 2015   có đáp án chi tiết
60 đề thi thử toán của các trường thpt 2015 có đáp án chi tiết
 
300 câu hỏi trắc nghiệm tích phân và ứng dụng - Nhóm Toán
300 câu hỏi trắc nghiệm tích phân và ứng dụng - Nhóm Toán300 câu hỏi trắc nghiệm tích phân và ứng dụng - Nhóm Toán
300 câu hỏi trắc nghiệm tích phân và ứng dụng - Nhóm Toán
 
chuyên đề ôn thi chuyển cấp môn toán 2017- 2018 mới nhất
chuyên đề ôn thi chuyển cấp môn toán 2017- 2018 mới nhấtchuyên đề ôn thi chuyển cấp môn toán 2017- 2018 mới nhất
chuyên đề ôn thi chuyển cấp môn toán 2017- 2018 mới nhất
 
20 đề thi thử kì thi quốc gia có đáp án
20 đề thi thử kì thi quốc gia có đáp án20 đề thi thử kì thi quốc gia có đáp án
20 đề thi thử kì thi quốc gia có đáp án
 
Toan pt.de028.2011
Toan pt.de028.2011Toan pt.de028.2011
Toan pt.de028.2011
 
Hinh hoc-affine
Hinh hoc-affineHinh hoc-affine
Hinh hoc-affine
 
5 đề trắc nghiệm toán 12 khảo sát hàm số - iHoc.me | Tài liệu toán học
5 đề trắc nghiệm toán 12 khảo sát hàm số - iHoc.me | Tài liệu toán học5 đề trắc nghiệm toán 12 khảo sát hàm số - iHoc.me | Tài liệu toán học
5 đề trắc nghiệm toán 12 khảo sát hàm số - iHoc.me | Tài liệu toán học
 

Similar to BỘ ĐỀ THI QUỐC GIA DANAMATH

De thi thu dai hoc so 88
De thi thu dai hoc so 88De thi thu dai hoc so 88
De thi thu dai hoc so 88
Trần Văn Khoa Tieuphong
 
Toan pt.de022.2012
Toan pt.de022.2012Toan pt.de022.2012
Toan pt.de022.2012BẢO Hí
 
Tai lieu luyen thi mon toan de thi dh mon toan khoi a - nam 2008
Tai lieu luyen thi mon toan   de thi dh mon toan khoi a - nam 2008Tai lieu luyen thi mon toan   de thi dh mon toan khoi a - nam 2008
Tai lieu luyen thi mon toan de thi dh mon toan khoi a - nam 2008
Trungtâmluyệnthi Qsc
 
Đề thi thử ĐH Toán Chuyên Quốc Học Huế 2014 - Khối D - Lần 1
Đề thi thử ĐH Toán Chuyên Quốc Học Huế 2014 - Khối D - Lần 1Đề thi thử ĐH Toán Chuyên Quốc Học Huế 2014 - Khối D - Lần 1
Đề thi thử ĐH Toán Chuyên Quốc Học Huế 2014 - Khối D - Lần 1
Jo Calderone
 
Toan pt.de038.2012
Toan pt.de038.2012Toan pt.de038.2012
Toan pt.de038.2012BẢO Hí
 
Thi thử toán THPT Lý Thái Tổ BN lần 1 2014
Thi thử toán THPT Lý Thái Tổ BN lần 1 2014Thi thử toán THPT Lý Thái Tổ BN lần 1 2014
Thi thử toán THPT Lý Thái Tổ BN lần 1 2014
dlinh123
 
3 Đề thi thử môn toán 2015 from http://toanphothong.com/
3 Đề thi thử môn toán 2015 from http://toanphothong.com/3 Đề thi thử môn toán 2015 from http://toanphothong.com/
3 Đề thi thử môn toán 2015 from http://toanphothong.com/
Vui Lên Bạn Nhé
 
3 Đề thi thử môn toán 2015 from http://toanphothong.com/
3 Đề thi thử môn toán 2015 from http://toanphothong.com/3 Đề thi thử môn toán 2015 from http://toanphothong.com/
3 Đề thi thử môn toán 2015 from http://toanphothong.com/
Vui Lên Bạn Nhé
 
Dap an chi tiet cao dang tu 2002-2004
Dap an chi tiet  cao dang tu  2002-2004Dap an chi tiet  cao dang tu  2002-2004
Dap an chi tiet cao dang tu 2002-2004
Thiên Đường Tình Yêu
 
De thi thử 2013-2014
De thi thử 2013-2014De thi thử 2013-2014
De thi thử 2013-2014
Nhi Triệu Yến
 
Lý thuyết, hướng dẫn giải hàm số - Bài tập hè - Toán cấp 3
Lý thuyết, hướng dẫn giải hàm số - Bài tập hè - Toán cấp 3Lý thuyết, hướng dẫn giải hàm số - Bài tập hè - Toán cấp 3
Lý thuyết, hướng dẫn giải hàm số - Bài tập hè - Toán cấp 3
VuKirikou
 
De thi-thu-dh-lan1-khoi-a-2015
De thi-thu-dh-lan1-khoi-a-2015De thi-thu-dh-lan1-khoi-a-2015
De thi-thu-dh-lan1-khoi-a-2015
onthitot .com
 
Tai lieu luyen thi mon toan de thi dh mon toan khoi d - nam 2008
Tai lieu luyen thi mon toan   de thi dh mon toan khoi d - nam 2008Tai lieu luyen thi mon toan   de thi dh mon toan khoi d - nam 2008
Tai lieu luyen thi mon toan de thi dh mon toan khoi d - nam 2008
Trungtâmluyệnthi Qsc
 
Tai lieu luyen thi dai hoc de thi dh mon toan khoi a - nam 2009
Tai lieu luyen thi dai hoc   de thi dh mon toan khoi a - nam 2009Tai lieu luyen thi dai hoc   de thi dh mon toan khoi a - nam 2009
Tai lieu luyen thi dai hoc de thi dh mon toan khoi a - nam 2009Trungtâmluyệnthi Qsc
 
Tai lieu luyen thi dai hoc de thi dh mon toan khoi a - nam 2009
Tai lieu luyen thi dai hoc   de thi dh mon toan khoi a - nam 2009Tai lieu luyen thi dai hoc   de thi dh mon toan khoi a - nam 2009
Tai lieu luyen thi dai hoc de thi dh mon toan khoi a - nam 2009Trungtâmluyệnthi Qsc
 
Toan pt.de049.2010
Toan pt.de049.2010Toan pt.de049.2010
Toan pt.de049.2010
BẢO Hí
 
Thi thử toán THPT Chu Văn An TN lần 2 2014
Thi thử toán THPT Chu Văn An TN lần 2 2014Thi thử toán THPT Chu Văn An TN lần 2 2014
Thi thử toán THPT Chu Văn An TN lần 2 2014
dlinh123
 

Similar to BỘ ĐỀ THI QUỐC GIA DANAMATH (20)

De thi thu dai hoc so 88
De thi thu dai hoc so 88De thi thu dai hoc so 88
De thi thu dai hoc so 88
 
Toan pt.de022.2012
Toan pt.de022.2012Toan pt.de022.2012
Toan pt.de022.2012
 
Da toan b_2
Da toan b_2Da toan b_2
Da toan b_2
 
Tai lieu luyen thi mon toan de thi dh mon toan khoi a - nam 2008
Tai lieu luyen thi mon toan   de thi dh mon toan khoi a - nam 2008Tai lieu luyen thi mon toan   de thi dh mon toan khoi a - nam 2008
Tai lieu luyen thi mon toan de thi dh mon toan khoi a - nam 2008
 
Da toan d_2
Da toan d_2Da toan d_2
Da toan d_2
 
Đề thi thử ĐH Toán Chuyên Quốc Học Huế 2014 - Khối D - Lần 1
Đề thi thử ĐH Toán Chuyên Quốc Học Huế 2014 - Khối D - Lần 1Đề thi thử ĐH Toán Chuyên Quốc Học Huế 2014 - Khối D - Lần 1
Đề thi thử ĐH Toán Chuyên Quốc Học Huế 2014 - Khối D - Lần 1
 
Toan pt.de038.2012
Toan pt.de038.2012Toan pt.de038.2012
Toan pt.de038.2012
 
Thi thử toán THPT Lý Thái Tổ BN lần 1 2014
Thi thử toán THPT Lý Thái Tổ BN lần 1 2014Thi thử toán THPT Lý Thái Tổ BN lần 1 2014
Thi thử toán THPT Lý Thái Tổ BN lần 1 2014
 
3 Đề thi thử môn toán 2015 from http://toanphothong.com/
3 Đề thi thử môn toán 2015 from http://toanphothong.com/3 Đề thi thử môn toán 2015 from http://toanphothong.com/
3 Đề thi thử môn toán 2015 from http://toanphothong.com/
 
3 Đề thi thử môn toán 2015 from http://toanphothong.com/
3 Đề thi thử môn toán 2015 from http://toanphothong.com/3 Đề thi thử môn toán 2015 from http://toanphothong.com/
3 Đề thi thử môn toán 2015 from http://toanphothong.com/
 
Dap an chi tiet cao dang tu 2002-2004
Dap an chi tiet  cao dang tu  2002-2004Dap an chi tiet  cao dang tu  2002-2004
Dap an chi tiet cao dang tu 2002-2004
 
De thi thử 2013-2014
De thi thử 2013-2014De thi thử 2013-2014
De thi thử 2013-2014
 
Lý thuyết, hướng dẫn giải hàm số - Bài tập hè - Toán cấp 3
Lý thuyết, hướng dẫn giải hàm số - Bài tập hè - Toán cấp 3Lý thuyết, hướng dẫn giải hàm số - Bài tập hè - Toán cấp 3
Lý thuyết, hướng dẫn giải hàm số - Bài tập hè - Toán cấp 3
 
De thi-thu-dh-lan1-khoi-a-2015
De thi-thu-dh-lan1-khoi-a-2015De thi-thu-dh-lan1-khoi-a-2015
De thi-thu-dh-lan1-khoi-a-2015
 
Tai lieu luyen thi mon toan de thi dh mon toan khoi d - nam 2008
Tai lieu luyen thi mon toan   de thi dh mon toan khoi d - nam 2008Tai lieu luyen thi mon toan   de thi dh mon toan khoi d - nam 2008
Tai lieu luyen thi mon toan de thi dh mon toan khoi d - nam 2008
 
Tai lieu luyen thi dai hoc de thi dh mon toan khoi a - nam 2009
Tai lieu luyen thi dai hoc   de thi dh mon toan khoi a - nam 2009Tai lieu luyen thi dai hoc   de thi dh mon toan khoi a - nam 2009
Tai lieu luyen thi dai hoc de thi dh mon toan khoi a - nam 2009
 
Tai lieu luyen thi dai hoc de thi dh mon toan khoi a - nam 2009
Tai lieu luyen thi dai hoc   de thi dh mon toan khoi a - nam 2009Tai lieu luyen thi dai hoc   de thi dh mon toan khoi a - nam 2009
Tai lieu luyen thi dai hoc de thi dh mon toan khoi a - nam 2009
 
Da toan b
Da toan bDa toan b
Da toan b
 
Toan pt.de049.2010
Toan pt.de049.2010Toan pt.de049.2010
Toan pt.de049.2010
 
Thi thử toán THPT Chu Văn An TN lần 2 2014
Thi thử toán THPT Chu Văn An TN lần 2 2014Thi thử toán THPT Chu Văn An TN lần 2 2014
Thi thử toán THPT Chu Văn An TN lần 2 2014
 

More from DANAMATH

SƠ LƯỢC VỀ PHƯƠNG PHÁP ĐÁNH GIÁ
SƠ LƯỢC VỀ PHƯƠNG PHÁP ĐÁNH GIÁSƠ LƯỢC VỀ PHƯƠNG PHÁP ĐÁNH GIÁ
SƠ LƯỢC VỀ PHƯƠNG PHÁP ĐÁNH GIÁ
DANAMATH
 
DÃY SỐ - CẤP SỐ
DÃY SỐ - CẤP SỐDÃY SỐ - CẤP SỐ
DÃY SỐ - CẤP SỐ
DANAMATH
 
TÍCH PHÂN VÀ CÁC ỨNG DỤNG
TÍCH PHÂN VÀ CÁC ỨNG DỤNGTÍCH PHÂN VÀ CÁC ỨNG DỤNG
TÍCH PHÂN VÀ CÁC ỨNG DỤNG
DANAMATH
 
HÀM SỐ MŨ & LOGARIT
HÀM SỐ MŨ & LOGARITHÀM SỐ MŨ & LOGARIT
HÀM SỐ MŨ & LOGARIT
DANAMATH
 
ĐẠI SỐ TỔ HỢP
ĐẠI SỐ TỔ HỢPĐẠI SỐ TỔ HỢP
ĐẠI SỐ TỔ HỢP
DANAMATH
 
GIẢI TAM GIÁC TRONG TỌA ĐỘ PHẲNG
GIẢI TAM GIÁC TRONG TỌA ĐỘ PHẲNGGIẢI TAM GIÁC TRONG TỌA ĐỘ PHẲNG
GIẢI TAM GIÁC TRONG TỌA ĐỘ PHẲNG
DANAMATH
 
PHÉP QUAY & PHÉP VỊ TỰ
PHÉP QUAY & PHÉP VỊ TỰPHÉP QUAY & PHÉP VỊ TỰ
PHÉP QUAY & PHÉP VỊ TỰ
DANAMATH
 
THAM SỐ HÓA TRONG TỌA ĐỘ PHẲNG
THAM SỐ HÓA TRONG TỌA ĐỘ PHẲNGTHAM SỐ HÓA TRONG TỌA ĐỘ PHẲNG
THAM SỐ HÓA TRONG TỌA ĐỘ PHẲNG
DANAMATH
 
PHÉP ĐỐI XỨNG TRỤC VÀ ĐỐI XỨNG TÂM
PHÉP ĐỐI XỨNG TRỤC VÀ ĐỐI XỨNG TÂMPHÉP ĐỐI XỨNG TRỤC VÀ ĐỐI XỨNG TÂM
PHÉP ĐỐI XỨNG TRỤC VÀ ĐỐI XỨNG TÂM
DANAMATH
 
CHUYÊN ĐỀ :TỌA ĐỘ PHẲNG - PHƯƠNG PHÁP VECTƠ
CHUYÊN ĐỀ :TỌA ĐỘ PHẲNG - PHƯƠNG PHÁP VECTƠCHUYÊN ĐỀ :TỌA ĐỘ PHẲNG - PHƯƠNG PHÁP VECTƠ
CHUYÊN ĐỀ :TỌA ĐỘ PHẲNG - PHƯƠNG PHÁP VECTƠ
DANAMATH
 
SƠ LƯỢC VỀ PHƯƠNG TRÌNH TÍCH TRONG LƯỢNG GIÁC
SƠ LƯỢC VỀ PHƯƠNG TRÌNH TÍCH TRONG LƯỢNG GIÁCSƠ LƯỢC VỀ PHƯƠNG TRÌNH TÍCH TRONG LƯỢNG GIÁC
SƠ LƯỢC VỀ PHƯƠNG TRÌNH TÍCH TRONG LƯỢNG GIÁC
DANAMATH
 
PHƯƠNG TRÌNH LƯỢNG GIÁC BẬC MỘT THEO SIN ,COS
PHƯƠNG TRÌNH LƯỢNG GIÁC BẬC MỘT THEO SIN ,COSPHƯƠNG TRÌNH LƯỢNG GIÁC BẬC MỘT THEO SIN ,COS
PHƯƠNG TRÌNH LƯỢNG GIÁC BẬC MỘT THEO SIN ,COS
DANAMATH
 
CÁC DẠNG TOÁN TRONG VECTƠ
CÁC DẠNG TOÁN TRONG VECTƠCÁC DẠNG TOÁN TRONG VECTƠ
CÁC DẠNG TOÁN TRONG VECTƠ
DANAMATH
 
PHÉP NHÂN MỘT SỐ VỚI MỘT VECTƠ
PHÉP NHÂN MỘT SỐ VỚI MỘT VECTƠPHÉP NHÂN MỘT SỐ VỚI MỘT VECTƠ
PHÉP NHÂN MỘT SỐ VỚI MỘT VECTƠ
DANAMATH
 
VECTƠ VÀ CÁC ỨNG DỤNG
VECTƠ VÀ CÁC ỨNG DỤNGVECTƠ VÀ CÁC ỨNG DỤNG
VECTƠ VÀ CÁC ỨNG DỤNG
DANAMATH
 
HÀM SỐ LƯỢNG GIÁC
HÀM SỐ LƯỢNG GIÁCHÀM SỐ LƯỢNG GIÁC
HÀM SỐ LƯỢNG GIÁC
DANAMATH
 

More from DANAMATH (16)

SƠ LƯỢC VỀ PHƯƠNG PHÁP ĐÁNH GIÁ
SƠ LƯỢC VỀ PHƯƠNG PHÁP ĐÁNH GIÁSƠ LƯỢC VỀ PHƯƠNG PHÁP ĐÁNH GIÁ
SƠ LƯỢC VỀ PHƯƠNG PHÁP ĐÁNH GIÁ
 
DÃY SỐ - CẤP SỐ
DÃY SỐ - CẤP SỐDÃY SỐ - CẤP SỐ
DÃY SỐ - CẤP SỐ
 
TÍCH PHÂN VÀ CÁC ỨNG DỤNG
TÍCH PHÂN VÀ CÁC ỨNG DỤNGTÍCH PHÂN VÀ CÁC ỨNG DỤNG
TÍCH PHÂN VÀ CÁC ỨNG DỤNG
 
HÀM SỐ MŨ & LOGARIT
HÀM SỐ MŨ & LOGARITHÀM SỐ MŨ & LOGARIT
HÀM SỐ MŨ & LOGARIT
 
ĐẠI SỐ TỔ HỢP
ĐẠI SỐ TỔ HỢPĐẠI SỐ TỔ HỢP
ĐẠI SỐ TỔ HỢP
 
GIẢI TAM GIÁC TRONG TỌA ĐỘ PHẲNG
GIẢI TAM GIÁC TRONG TỌA ĐỘ PHẲNGGIẢI TAM GIÁC TRONG TỌA ĐỘ PHẲNG
GIẢI TAM GIÁC TRONG TỌA ĐỘ PHẲNG
 
PHÉP QUAY & PHÉP VỊ TỰ
PHÉP QUAY & PHÉP VỊ TỰPHÉP QUAY & PHÉP VỊ TỰ
PHÉP QUAY & PHÉP VỊ TỰ
 
THAM SỐ HÓA TRONG TỌA ĐỘ PHẲNG
THAM SỐ HÓA TRONG TỌA ĐỘ PHẲNGTHAM SỐ HÓA TRONG TỌA ĐỘ PHẲNG
THAM SỐ HÓA TRONG TỌA ĐỘ PHẲNG
 
PHÉP ĐỐI XỨNG TRỤC VÀ ĐỐI XỨNG TÂM
PHÉP ĐỐI XỨNG TRỤC VÀ ĐỐI XỨNG TÂMPHÉP ĐỐI XỨNG TRỤC VÀ ĐỐI XỨNG TÂM
PHÉP ĐỐI XỨNG TRỤC VÀ ĐỐI XỨNG TÂM
 
CHUYÊN ĐỀ :TỌA ĐỘ PHẲNG - PHƯƠNG PHÁP VECTƠ
CHUYÊN ĐỀ :TỌA ĐỘ PHẲNG - PHƯƠNG PHÁP VECTƠCHUYÊN ĐỀ :TỌA ĐỘ PHẲNG - PHƯƠNG PHÁP VECTƠ
CHUYÊN ĐỀ :TỌA ĐỘ PHẲNG - PHƯƠNG PHÁP VECTƠ
 
SƠ LƯỢC VỀ PHƯƠNG TRÌNH TÍCH TRONG LƯỢNG GIÁC
SƠ LƯỢC VỀ PHƯƠNG TRÌNH TÍCH TRONG LƯỢNG GIÁCSƠ LƯỢC VỀ PHƯƠNG TRÌNH TÍCH TRONG LƯỢNG GIÁC
SƠ LƯỢC VỀ PHƯƠNG TRÌNH TÍCH TRONG LƯỢNG GIÁC
 
PHƯƠNG TRÌNH LƯỢNG GIÁC BẬC MỘT THEO SIN ,COS
PHƯƠNG TRÌNH LƯỢNG GIÁC BẬC MỘT THEO SIN ,COSPHƯƠNG TRÌNH LƯỢNG GIÁC BẬC MỘT THEO SIN ,COS
PHƯƠNG TRÌNH LƯỢNG GIÁC BẬC MỘT THEO SIN ,COS
 
CÁC DẠNG TOÁN TRONG VECTƠ
CÁC DẠNG TOÁN TRONG VECTƠCÁC DẠNG TOÁN TRONG VECTƠ
CÁC DẠNG TOÁN TRONG VECTƠ
 
PHÉP NHÂN MỘT SỐ VỚI MỘT VECTƠ
PHÉP NHÂN MỘT SỐ VỚI MỘT VECTƠPHÉP NHÂN MỘT SỐ VỚI MỘT VECTƠ
PHÉP NHÂN MỘT SỐ VỚI MỘT VECTƠ
 
VECTƠ VÀ CÁC ỨNG DỤNG
VECTƠ VÀ CÁC ỨNG DỤNGVECTƠ VÀ CÁC ỨNG DỤNG
VECTƠ VÀ CÁC ỨNG DỤNG
 
HÀM SỐ LƯỢNG GIÁC
HÀM SỐ LƯỢNG GIÁCHÀM SỐ LƯỢNG GIÁC
HÀM SỐ LƯỢNG GIÁC
 

Recently uploaded

Chương III (Nội dung vẽ sơ đồ tư duy chương 3)
Chương III (Nội dung vẽ sơ đồ tư duy chương 3)Chương III (Nội dung vẽ sơ đồ tư duy chương 3)
Chương III (Nội dung vẽ sơ đồ tư duy chương 3)
duykhoacao
 
Dẫn luận ngôn ngữ - Tu vung ngu nghia.pptx
Dẫn luận ngôn ngữ - Tu vung ngu nghia.pptxDẫn luận ngôn ngữ - Tu vung ngu nghia.pptx
Dẫn luận ngôn ngữ - Tu vung ngu nghia.pptx
nvlinhchi1612
 
GIÁO TRÌNH 2-TÀI LIỆU SỬA CHỮA BOARD MONO TỦ LẠNH MÁY GIẶT ĐIỀU HÒA.pdf
GIÁO TRÌNH 2-TÀI LIỆU SỬA CHỮA BOARD MONO TỦ LẠNH MÁY GIẶT ĐIỀU HÒA.pdfGIÁO TRÌNH 2-TÀI LIỆU SỬA CHỮA BOARD MONO TỦ LẠNH MÁY GIẶT ĐIỀU HÒA.pdf
GIÁO TRÌNH 2-TÀI LIỆU SỬA CHỮA BOARD MONO TỦ LẠNH MÁY GIẶT ĐIỀU HÒA.pdf
Điện Lạnh Bách Khoa Hà Nội
 
98 BÀI LUYỆN NGHE TUYỂN SINH VÀO LỚP 10 TIẾNG ANH DẠNG TRẮC NGHIỆM 4 CÂU TRẢ ...
98 BÀI LUYỆN NGHE TUYỂN SINH VÀO LỚP 10 TIẾNG ANH DẠNG TRẮC NGHIỆM 4 CÂU TRẢ ...98 BÀI LUYỆN NGHE TUYỂN SINH VÀO LỚP 10 TIẾNG ANH DẠNG TRẮC NGHIỆM 4 CÂU TRẢ ...
98 BÀI LUYỆN NGHE TUYỂN SINH VÀO LỚP 10 TIẾNG ANH DẠNG TRẮC NGHIỆM 4 CÂU TRẢ ...
Nguyen Thanh Tu Collection
 
AV6 - PIE CHART WRITING skill in english
AV6 - PIE CHART WRITING skill in englishAV6 - PIE CHART WRITING skill in english
AV6 - PIE CHART WRITING skill in english
Qucbo964093
 
BAI TAP ON HE LOP 2 LEN 3 MON TIENG VIET.pdf
BAI TAP ON HE LOP 2 LEN 3 MON TIENG VIET.pdfBAI TAP ON HE LOP 2 LEN 3 MON TIENG VIET.pdf
BAI TAP ON HE LOP 2 LEN 3 MON TIENG VIET.pdf
phamthuhoai20102005
 
30 - ĐỀ THI HSG - HÓA HỌC 9 - NĂM HỌC 2021 - 2022.pdf
30 - ĐỀ THI HSG - HÓA HỌC 9 - NĂM HỌC 2021 - 2022.pdf30 - ĐỀ THI HSG - HÓA HỌC 9 - NĂM HỌC 2021 - 2022.pdf
30 - ĐỀ THI HSG - HÓA HỌC 9 - NĂM HỌC 2021 - 2022.pdf
ngocnguyensp1
 
Khoá luận tốt nghiệp ngành Truyền thông đa phương tiện Xây dựng kế hoạch truy...
Khoá luận tốt nghiệp ngành Truyền thông đa phương tiện Xây dựng kế hoạch truy...Khoá luận tốt nghiệp ngành Truyền thông đa phương tiện Xây dựng kế hoạch truy...
Khoá luận tốt nghiệp ngành Truyền thông đa phương tiện Xây dựng kế hoạch truy...
https://www.facebook.com/garmentspace
 
CHUYÊN ĐỀ BỒI DƯỠNG HỌC SINH GIỎI KHOA HỌC TỰ NHIÊN 9 CHƯƠNG TRÌNH MỚI - PHẦN...
CHUYÊN ĐỀ BỒI DƯỠNG HỌC SINH GIỎI KHOA HỌC TỰ NHIÊN 9 CHƯƠNG TRÌNH MỚI - PHẦN...CHUYÊN ĐỀ BỒI DƯỠNG HỌC SINH GIỎI KHOA HỌC TỰ NHIÊN 9 CHƯƠNG TRÌNH MỚI - PHẦN...
CHUYÊN ĐỀ BỒI DƯỠNG HỌC SINH GIỎI KHOA HỌC TỰ NHIÊN 9 CHƯƠNG TRÌNH MỚI - PHẦN...
Nguyen Thanh Tu Collection
 
Ảnh hưởng của nhân sinh quan Phật giáo đến đời sống tinh thần Việt Nam hiện nay
Ảnh hưởng của nhân sinh quan Phật giáo đến đời sống tinh thần Việt Nam hiện nayẢnh hưởng của nhân sinh quan Phật giáo đến đời sống tinh thần Việt Nam hiện nay
Ảnh hưởng của nhân sinh quan Phật giáo đến đời sống tinh thần Việt Nam hiện nay
chinhkt50
 

Recently uploaded (10)

Chương III (Nội dung vẽ sơ đồ tư duy chương 3)
Chương III (Nội dung vẽ sơ đồ tư duy chương 3)Chương III (Nội dung vẽ sơ đồ tư duy chương 3)
Chương III (Nội dung vẽ sơ đồ tư duy chương 3)
 
Dẫn luận ngôn ngữ - Tu vung ngu nghia.pptx
Dẫn luận ngôn ngữ - Tu vung ngu nghia.pptxDẫn luận ngôn ngữ - Tu vung ngu nghia.pptx
Dẫn luận ngôn ngữ - Tu vung ngu nghia.pptx
 
GIÁO TRÌNH 2-TÀI LIỆU SỬA CHỮA BOARD MONO TỦ LẠNH MÁY GIẶT ĐIỀU HÒA.pdf
GIÁO TRÌNH 2-TÀI LIỆU SỬA CHỮA BOARD MONO TỦ LẠNH MÁY GIẶT ĐIỀU HÒA.pdfGIÁO TRÌNH 2-TÀI LIỆU SỬA CHỮA BOARD MONO TỦ LẠNH MÁY GIẶT ĐIỀU HÒA.pdf
GIÁO TRÌNH 2-TÀI LIỆU SỬA CHỮA BOARD MONO TỦ LẠNH MÁY GIẶT ĐIỀU HÒA.pdf
 
98 BÀI LUYỆN NGHE TUYỂN SINH VÀO LỚP 10 TIẾNG ANH DẠNG TRẮC NGHIỆM 4 CÂU TRẢ ...
98 BÀI LUYỆN NGHE TUYỂN SINH VÀO LỚP 10 TIẾNG ANH DẠNG TRẮC NGHIỆM 4 CÂU TRẢ ...98 BÀI LUYỆN NGHE TUYỂN SINH VÀO LỚP 10 TIẾNG ANH DẠNG TRẮC NGHIỆM 4 CÂU TRẢ ...
98 BÀI LUYỆN NGHE TUYỂN SINH VÀO LỚP 10 TIẾNG ANH DẠNG TRẮC NGHIỆM 4 CÂU TRẢ ...
 
AV6 - PIE CHART WRITING skill in english
AV6 - PIE CHART WRITING skill in englishAV6 - PIE CHART WRITING skill in english
AV6 - PIE CHART WRITING skill in english
 
BAI TAP ON HE LOP 2 LEN 3 MON TIENG VIET.pdf
BAI TAP ON HE LOP 2 LEN 3 MON TIENG VIET.pdfBAI TAP ON HE LOP 2 LEN 3 MON TIENG VIET.pdf
BAI TAP ON HE LOP 2 LEN 3 MON TIENG VIET.pdf
 
30 - ĐỀ THI HSG - HÓA HỌC 9 - NĂM HỌC 2021 - 2022.pdf
30 - ĐỀ THI HSG - HÓA HỌC 9 - NĂM HỌC 2021 - 2022.pdf30 - ĐỀ THI HSG - HÓA HỌC 9 - NĂM HỌC 2021 - 2022.pdf
30 - ĐỀ THI HSG - HÓA HỌC 9 - NĂM HỌC 2021 - 2022.pdf
 
Khoá luận tốt nghiệp ngành Truyền thông đa phương tiện Xây dựng kế hoạch truy...
Khoá luận tốt nghiệp ngành Truyền thông đa phương tiện Xây dựng kế hoạch truy...Khoá luận tốt nghiệp ngành Truyền thông đa phương tiện Xây dựng kế hoạch truy...
Khoá luận tốt nghiệp ngành Truyền thông đa phương tiện Xây dựng kế hoạch truy...
 
CHUYÊN ĐỀ BỒI DƯỠNG HỌC SINH GIỎI KHOA HỌC TỰ NHIÊN 9 CHƯƠNG TRÌNH MỚI - PHẦN...
CHUYÊN ĐỀ BỒI DƯỠNG HỌC SINH GIỎI KHOA HỌC TỰ NHIÊN 9 CHƯƠNG TRÌNH MỚI - PHẦN...CHUYÊN ĐỀ BỒI DƯỠNG HỌC SINH GIỎI KHOA HỌC TỰ NHIÊN 9 CHƯƠNG TRÌNH MỚI - PHẦN...
CHUYÊN ĐỀ BỒI DƯỠNG HỌC SINH GIỎI KHOA HỌC TỰ NHIÊN 9 CHƯƠNG TRÌNH MỚI - PHẦN...
 
Ảnh hưởng của nhân sinh quan Phật giáo đến đời sống tinh thần Việt Nam hiện nay
Ảnh hưởng của nhân sinh quan Phật giáo đến đời sống tinh thần Việt Nam hiện nayẢnh hưởng của nhân sinh quan Phật giáo đến đời sống tinh thần Việt Nam hiện nay
Ảnh hưởng của nhân sinh quan Phật giáo đến đời sống tinh thần Việt Nam hiện nay
 

BỘ ĐỀ THI QUỐC GIA DANAMATH

  • 2. ĐỀ SỐ 1 Câu 1(2,0 điểm). Cho hàm số 2 1 1 x y x + = + có đồ thị (C). 1. Khảo sát sự biến thiên và vẽ đồ thị (C) của hàm số đã cho. 2. Viết phương trình tiếp tuyến của đồ thị (C) biết tiếp tuyến đó cách đều hai điểm A(2; 4) và B(-4; - 2). Câu 2(1,0 điểm): 1. Giải phương trình sau: 2cos6 2cos4 3cos2 sin 2 3x x x x+ − = + 2. Giải phương trình : 3 3 33log 2 log log2 3 .2 12x x x x x+ = + Câu 3 (1,0điểm). 1. Tính tích phân: 4 20 sin 5sinx.cos x + 2cosx x I dx π = ∫ 2. Cho hình phẳng (H) được giới hạn bởi đồ thị của các hàm số sin3 , 0 , 0y x y x= = = và 6 x π = . Tính thể tích khối tròn xoay sinh bởi (H) khi quay quanh trục Ox. Câu 4 (1,0 điểm). 1. Tìm tập hợp các điểm biểu diển của số phức z thỏa mãn đẳng thức sau : 2 1 1z i iz i− + = + − . trong các số phức thỏa điều kiện trên , hãy tìm số phức z có môđun nhỏ nhất. 2. Chọn ngẫu nhiên hai số hạng trong khai triển nhị thức Newton ( ) 100 6 3 5− . Tính xác suất để hai số được chọn có một số là số hữu tỷ và một số là số vô tỷ. Câu 5(1,0 điểm).Cho hình chóp SABCD có đáy ABCD là hình vuông, tam giác SAB vuông góc tại S và nằm trong mặt phẳng vuông góc với đáy (ABCD). Biết khoảng cách giữa hai đường thẳng SC và AB bằng 2 57 19 a ; góc tạo bới SB và mặt phẳng (ABCD) bằng 300 . Tính thể tích của khối chóp SABCD và diện tích mặt cầu ngoại tiếp tứ diện SBCD. Câu 6(1,0điểm).Trong không gian với hệ trục tọa độ Oxyz, cho điểm M(0; 2; 0) và hai đường thẳng 1 1 2 1 : 2 2 1 x y z d − − + = = − và 2 3 1 : 2 2 1 x y z d − + = = − . Viết phương trình mặt phẳng (P) đi qua M song song vớitrục Ox, sao cho (P) cắt hai đường thẳng d1, d2 lần lượt tại A, B thỏa mãn AB = 1. Câu 7(1,0 điểm).Trong mặt phẳng với hệ trục tọa độ Oxy, cho hình thoi ABCD có góc  0 60BAD = . Trên cạnh AB, BC lấy các điểm M, N sao cho MB + NB = AB. Biết ( )3 ;1P thuộc đường thẳng DN và đường phân giác trong của góc MDN có phương trình 3 6 0x y− + = . Tìm tọa độ đỉnh D của hình thoi đã cho. Câu 8(1,0 điểm).Giải hệ phương trình: 2 2 2 1 3 2 2 3 6 2 2 3 10 2 2 1 4 6 4 2 x x x y x y x y x y x y x y + + − = + +   − − + + = − + + − +
  • 3. Câu 9 (1,0 điểm).Cho x, y là hai số thực dương thỏa mãn điều kiện : ( )2 2 2 2 2 x y x y+ = . Tìm giá trị nhỏ nhất của biểu thức : 2 2 1 1 1 1 x y P y x x y = + + + + + + --- Hết --- GIẢI ĐỀ SỐ 1 Câu 1(2,0 điểm). Cho hàm số 2 1 1 x y x + = + có đồ thị (C). 2.Viết phương trình tiếp tuyến của đồ thị (C) biết tiếp tuyến đó cách đều hai điểm A(2; 4) và B(-4; - 2). Phương pháp chung : Viết phương trình tiếp tuyến với (C): ( )y f x= thỏa mãn tính chất P. • Gọi M(xo; 0( )f x ) , 0 fx D∀ ∈ là tiếp điểm của tiếp tuyến cần tìm ⇒ hệ số góc của tiếp tuyến là 0'( )k f x= • Phương trình tiếp tuyến cần tìm có dạng : 0 0 0: '( )( ) ( )d y f x x x f x= − + {tới đây ta chỉ cần tìm 0x } • Phân Tích tính chất P. o Nếu tính chất P liên quan đến hệ số góc {song song đường thẳng cho trước hoặc vuông góc đường thẳng cho trước hoặc tạo với đường thẳng cho trước một góc cụ thể hoặc tạo với hệ trục Oxy thành tam giác có tính chất về tỷ số …} Thì ta sử dụng các công thức ở trên để tìm hệ số góc tiếp tuyến sau đó giải phương trình 0'( )k f x= để tìm 0x o Nếu tính chất P là tiếp tuyến kẻ từ hoặc đi qua ( )1 1;A x y thì ta có ( )1 0 1 0 0'( ) ( )A d y f x x x f x∈ ⇔= − + {với 1 1,x y cho trước} từ đó suy ra 0x o Nếu tính chất P liên quan đến khoảng cách , góc, diện tích, … thì ta sử dụng công thức tương ứng để lập phương trình theo biến 0x sau đó giải phương trình để tìm 0x Các chú ý về hệ số góc của đường thẳng : d: y = ax + b ⇒ hệ số góc của d là a = tan (d,chiều dươngOx) ĐB :d : x = a ⇒ d không có hệ số góc Cho d1 : y = a1x + b1 và d2 : y = a2x + b2 khi đó ta có • { }1 2 1 2 1 2/ /d d a a b b⇔ = ≠ • 1 2 1 2. 1d d a a⊥ ⇔ =−
  • 4. • ( ) 1 2 1 2 1 2 tan , 1 a a d d a a − = + • d1 tạo với trục Ox một góc α ⇒ αtan1 ±=a Nếu { } { } d Ox A d Oy B ∩ =  ∩ = Thì d OB K OA = { dK là hệ số góc của đường thẳng d} Cụ thể : Nếu . 0A Bx y < thì d OB K OA = Nếu . 0A Bx y > thì d OB K OA = − Bài giải : Gọi 0 0 0 2 1 ; 1 x M x x  +   +  (với 0 1x ≠ − ) là tiếp điểm của tiếp tuyến d cần tìm 2 20 0 0 0 02 0 0 2 11 : ( ) : ( 1) 2 2 1 0 ( 1) 1 x d y x x d x x y x x x x + = − + ⇔ − + + + += + + d cách đều hai điểm A, B ( ) 2 2 2 2 0 0 0 0 0 0 4 4 0 0 2 ( 1) 4 2 2 1 4 ( 1) ( 2) 2 2 1 , ( , ) 1 ( 1) 1 ( 1) x x x x x x d A d d B d x x − + + + + − − + − + + + = = = + + + + 2 0 0 0 02 2 0 0 0 0 2 0 0 0 2 0 0, 2 2 6 1 4 6 1 1 0 1, 1( ) x x x x x x x x x x x l  + =⇔ = =− ⇔ − − − = + − ⇔  − = ⇔ = =− 0 0 : 1x d y x=⇒ =+ 0 2 : 5x d y x=− ⇒ = + 0 1 5 1 : 4 4 x d y x=⇒ = + Vậy có ba tiếp tuyến thỏa yêu cầu bài toán: : 1d y x= + , : 5d y x= + và 1 5 : 4 4 d y x= + Câu 2(1,0 điểm): 1. Giải phương trình sau: 2cos6 2cos4 3cos2 sin 2 3x x x x+ − = + Dấu hiệu cần nhớ : Nếu phương trình lượng giác có chứa 3 thì thông thường ta sẽ xét hai trường hợp: • TH1: Nếu phương trình có chứa bật cao và dạng tích thì ta sử dụng công thức biến đổi tích thành tổng và công thức hạ bật để quy về dạng phương trình bật một theo sin, cos. • TH2: Nếu phương trình không có bật cao và dạng tích thì ta sử dụng công thức biến đổi tổng thành tích và công thức nhân đôi để biến đổi phương trình về dạng phương trình tích. Nếu phương trình lượng giác có chứa hai số hạng đồng thới thỏa mãn điều kiện: cùng hệ số, cùng hàm(sin hoặc cos), cùng tính chẳn hoặc lẻ của cung thì ta sử dụng công thức biến đổi tổng thành tích để quy phương trình về dạng phương trình tích
  • 5. Bài giải : ( ) ( )2 4cos5 cos 3 2cos 1 2sin cos 3 2cos 2cos5 3 cos sin 0pt x x x x x x x x x⇔ − −= + ⇔ − −= cos 0 2 3 cos sin 2cos5 (1) x x k x x x π π  = ⇔ = +⇔  + = 3 1 (1) cos sin cos5 2 2 x x x⇔ + = (chia hai vế cho 2 2 2 2 3 1 2a b+ = + = ) 5 2 6 24 2 cos cos sin sin cos5 cos5 cos 6 6 6 5 2 6 36 3 x x k x k x x x x x x x k x k π π π π π π π π π π π  =− + ⇔ =− +  ⇔ + = ⇔ = − ⇔      =− + + ⇔ = +  Vậy phương trình có ba nghiệm: 2 x k π π= + , 24 2 x k π π =− + và 36 3 x k π π = + với k Z∈ 2. Giải phương trình : 3 3 33log 2 log log2 3 .2 12x x x x x+ = + Dấu hiệu cần nhớ: Nếu phương trình lôga chứa số hạng có dạng: logb x a thì ta đặt log t bt x x b= ⇔= để chuyển phương trình về dạng phương trình mũ cơ bản Bài giải : ĐK: 0x > . Đặt : 3log 3t t x x= ⇔= ( ) 3 3 3 3 3 log 23log 2 log 8 log 8 3 3 3 8 t tt t x= = = = . ( ) ( ) 2 3 8 3 .2 12 3 8 18 12 27t t t t t t t t t pt ⇔ + = + ⇔ + = + 18 12 27 1 8 8 8 t t t       ⇔ + = +            3 2 2 3 3 3 3 3 3 1 0 1 1 0 1 0 1 2 2 2 2 2 2 t t t t t t t x                ⇔ − + − = ⇔ − + = ⇔ = ⇔ = ⇔ =                                 Vậy phương trình có nghiệm duy nhất x = 1. Câu 3 (1,0điểm). 1. Tính tích phân: 4 20 sin 5sinx.cos x + 2cosx x I dx π = ∫ Dấu hiệu cần nhớ: Nếu tích phân luong giác dạng phân số không căn và tất cả các số hạng cùng loại bậc chẳn hoặc cùng bật lẻ thì ta chia tử và mẫu cho cosn x để quy về dạng :
  • 6. 2 1 (tan ). cos f x dx x∫ sau đó đặt tant x= hoặc chiatử và mẫu cho sinn x để quy về dạng : 2 1 (cot ). sin f x dx x∫ sau đó đặt cott x= (với n là bậc cao nhất) Bài giải : 3 4 4 2 2 20 0 3 sin tan 1cos 5sinx.cos x + 2cosx 2tan 5tan 2 cos cos x xxI dx dx x x x x π π = + +∫ ∫ Đặt : 2 1 tan cos t x dt dx x = ⇒ = Đổi cận: 1 4 0 0 x t x t π = ⇒ =   = ⇒ = 1 1 20 0 1 2(2 1) ( 2) 2 5 2 3 (2 1)( 2) t t t I dt dt t t t t + − + = = + + + +∫ ∫ 1 0 11 2 1 2 1 1 2 ln 2 ln 2 1 ln3 ln 2 03 2 2 1 3 6 2 3 dt t t t t   = − = + − + = −  + +   ∫ 2. Cho hình phẳng (H) được giới hạn bởi đồ thị của các hàm số sin3 , 0 , 0y x y x= = = và 6 x π = . Tính thể tích khối tròn xoay sinh bởi (H) khi quay quanh trục Ox. Bài giải : ( ) ( ) 6 6 2 0 0 1 sin3 1 cos6 sin 6 ( )6 2 2 6 12 0 V x dx x dx x x dvtt π π π π π π π   = = − =− =    ∫ ∫ Câu 4 (1,0 điểm). 1. Tìm tập hợp các điểm biểu diển của số phức z thỏa mãn đẳng thức sau : 2 1 1z i iz i− + = + − . trong các số phức thỏa điều kiện trên , hãy tìm số phức z có môđun nhỏ nhất. Bài giải : Gọi z x yi= + với ,x y R∈ , z có điểm biểu diển là M(x ; y). ( ) ( ) ( ) ( ) ( ) ( ) ( ) ( ) 2 2 2 2 1 2 1 1 1 2 1 1 2 1 0gt x y i y x i x y y x y⇔ + + − = − − + + ⇔ + + − = − − + + ⇔ − = Vậy tập hợp điểm biểu diển của z là đường thẳng có phương trình là 2 1 0y − = 21 1 1 1 0 2 4 2 2 z x i z x Min z khi x= + ⇒ = + ≥ ⇒ = =
  • 7. Số phức có môđun nhỏ nhất thỏa yêu cầu bài toán là 1 2 z i= 2. Chọn ngẫu nhiên hai số hạng trong khai triển nhị thức Newton ( ) 100 6 3 5− . Tính xác suất để hai số được chọn có một số là số hữu tỷ và một số là số vô tỷ. Bài giải : Ta có khai triển: ( ) ( ) ( ) 100100 100 6 6 100 0 3 5 ( 1) 3 5 k k k k k C − = − = −∑ Số hạng tổng quát trong khai triển là: 100 62 1 100( 1) 3 5 kk k k kT C − + = − Vì 3 và 5 là các số nguyên tố nên 1kT + là số hữu tỷ thì 0 100 6 k k ≤ ≤ ⇒   có 17 số k thỏa điều kiện Vậy trong khai triển ta có 17 số hữu tỷ và 84 số vô tỷ. Gọi Ω là không gian mẫu của phép thử chọn ngẫu nhiên 2 số hạng trong 101 số hạng của khai triển: ⇒số phần tử củaΩ là 2 101 5050CΩ= = Gọi A là biến cố hai số hạng chọn ra có một số vô tỷ và một số hữu tỷ: • Chọn một số hữu tỷ từ 17 số hữu tỷ trong khai triển⇒ có 1 17 17C = cách chọn • Chọn một số vô tỷ từ 84 số vô tỷ trong khai triển⇒ có 1 84 84C = cách chọn Theo quy tắc nhân ta có số phần tử của không gian biến A là 17 84 1428AΩ = × = Vậy xác suất của biến cố A là 1428 ( ) ... 5050 A P A Ω = = = Ω Câu 5(1,0 điểm).Cho hình chóp SABCD có đáy ABCD là hình vuông, tam giác SAB vuông góc tại S và nằm trong mặt phẳng vuông góc với đáy (ABCD). Biết khoảng cách giữa hai đường thẳng SC và AB bằng 2 57 19 a ; góc tạo bới SB và mặt phẳng (ABCD) bằng 300 . Tính thể tích của khối chóp SABCD và diện tích mặt cầu ngoại tiếp tứ diện SBCD. Bài giải : Gọi H là hình chiếu của S lên AB mặt khác ta có (SAB) và (ABCD) vuông góc nhau theo giao tuyến AB ( )SH ABCD⇒ ⊥ S A D K
  • 8. Ta có BH là hình chiếu của BS lên mặt phẳng (ABCD) ( ) ( ) ,( ) ,SB ABCD BS BH SBH= = ,  0 30gt SBH⇒ = Ta có : ( )/ / / / ( , ) ( ,( ))AB CD AB SCD d AB SC d H SCD⇒ ⇒ = Dựng HM CD⊥ (với M CD∈ ) ( )CD SHM⇒ ⊥ ( vì SH CD⊥ ) ⇒ (SCD), (SHM) vuông góc nhau theo giao tuyến SM Dựng HK SM⊥ (với K SM∈ ) ( )HK SCD⇒ ⊥ ⇒ K là hình chiếu của H lên (SCD) ( , )d H SCD HK⇒ =, 2 57 19 a gt HK⇒ = Đặt: cạnh của hình vuông là b khi đó ta có HM = b. Xét ABS∆ ta có:  0 3 cos cos30 2 SB b SBA SB AB = = ⇒ = Xét SHB∆ ta có:  0 3 sin sin30 4 SH b SBH SH SB = = ⇒ = Xét SHM∆ ta có: 2 2 2 2 2 2 2 1 1 1 19 1 16 19 2 12 3 3 b a HK HS HM a b b b = + ⇔ = + = ⇔ = 3 2 SH a AB a  = ⇒  = 3 21 1 4 3 . . 3 3 3 SABCD ABCD a V SH S SH AB= = = (đvtt) Gọi I là tâm của ABCD, khi đó ta có: IB = IC = ID (1) Ta có : BC AB BC SA BC SH ⊥ ⇒ ⊥ ⊥ mà SA SB⊥ ( )SA SBC⇒ ⊥ SA SC SAC⇒ ⊥ ⇒ ∆ vuông tại S (2) IS IC= Từ (1) và (2) ta có I là tâm mật (S) cầu ngoại tiếp SBCD⇒bán kính mặt cầu (S) là 1 2 2 R IA AC a= = = Vậy diện tích mặt cầu ngoại tiếp hình chóp SBCD là: 2 2 ( ) 4 8SS R aπ π= = (đvdt) Câu 6(1,0điểm).Trong không gian với hệ trục tọa độ Oxyz, cho điểm M(0; 2; 0) và hai đường thẳng 1 1 2 1 : 2 2 1 x y z d − − + = = − và 2 3 1 : 2 2 1 x y z d − + = = − . Viết phương trình mặt phẳng (P) đi qua M song song vớitrục Ox, sao cho (P) cắt hai đường thẳng d1, d2 lần lượt tại A, B thỏa mãn AB = 1. Bài giải :
  • 9. Ta có: 1 1 2 : 2 2 1 x t d y t z t = +  = −  =− + và 2 3 2 : 1 2 x m d y m z m = +  =− −  = ( )1( ) 1 2 ; 2 2 ; 1 ( )A P d A t t t P= ∩ ⇒ + − − + ∈ , ( )2( ) 3 2 ; 1 2 ; ( )B P d B m m m P= ∩ ⇒ + − − ∈ ( )2 2 2; 2 2 3; 1AB m t m t m t= − + − + − − +  , ( )1 2 ; 2 ; 1AM t t t= − − − +  ( ) ( ) ( ) ( ) ( ) 2 2 2 2 1 1 2 2 2 2 2 3 1 1 9 22 13 0 13 9 m t AB m t m t m t m t m t m t − =− = ⇔ − + + − + − + − + = ⇔ − + − + = ⇔  − =−  Với 1m t− =− 1m t⇒ = − ( )0; 1; 0AB⇒ = −  ( ), 1; 0; 2 1pn AM AB t t = = − + +     là VTPT của (P). ( )/ /( ) . 0 1 0 1 0; 0; 3Ox p Ox p pOx P u n u n t t n⇒ ⊥ ⇔ = ⇔ − + = ⇔ = ⇒ =      (với Oxu  là VTCP của Ox) (P) qua M(0; 2; 0) và có VTPT là ( )0; 0; 3pn =  ( ) : 0P y⇒ =(loại vì (P) chứa Ox) Với 13 9 m t− =− 8 1 4 ; ; 9 9 9 AB   ⇒ =− − −     9 1 12 18 1 , ; ; 9 9 9 p t t n AM AB − + +  = = −        là VTPT của (P). 1 12 3 / /( ) . 0 9 1 0 0; ; 9 9 9 Ox p Ox p pOx P u n u n t t n   ⇒ ⊥ ⇔ = ⇔ − + = ⇔ = ⇒ = −         (với Oxu  là VTCP của Ox) (P) qua M(0; 2; 0) và có VTPT là 12 3 0; ; 9 9 pn   = −     ( ) : 4 8 0P y z⇒ − − =. Vậy mặt phẳng cần tìm là: ( ) : 4 8 0P y z− − = Câu 7(1,0 điểm).Trong mặt phẳng với hệ trục tọa độ Oxy, cho hình thoi ABCD có góc  0 60BAD = . Trên cạnh AB, BC lấy các điểm M, N sao cho MB + NB = AB. Biết ( )3 ;1P thuộc đường thẳng DN và đường phân giác trong của góc MDN có phương trình 3 6 0x y− + = . Tìm tọa độ đỉnh D của hình thoi đã cho. B : 3 6 0d x y− + = H P’ N M
  • 10. Bài giải : ( )3 6 ;D d D m m∈ ⇒ − MB + NB = AB NC MB CDN CDM NB MA NDB MDA = ∆ =∆  ⇒ ⇒  = ∆ =∆       0 60 NDC MDB MDN NDB MDB  = ⇒ ⇒ = = ( Vì  0 120ADC = ) Gọi P’ là điểm đối xứng với P qua d 'P DM⇒ ∈ 'DPP⇒ ∆ là tam giác đều có d là đường phân giác 2 ( , )DP d P d⇒ = ( ) ( ) 2 2 2 2 3 3 6 1 3 6 3 1 2 3 3 11 3 m m m m − + = ⇔ − − + −= ⇔  = ++ Vậy có hai điểm cần tìm ( )3 6 ;1D − và ( )3 3 ; 3 3 1D + + Câu 8(1,0 điểm).Giải hệ phương trình: 2 2 2 1 3 2 (1) 2 3 6 2 2 3 10 2 2 1 4 6 4 2 (2) x x x y x y x y x y x y x y + + − = + +   − − + + = − + + − + Bình luận: Vì phương trình (2) quá phức tạp và không có dấu hiệu để biến đổi nên ta sẽ tập trung vào (1). Ta sẽ biến đổi pt(1) về dạng tích bằng phương pháp nhân liên hợp,trước tiên ta phải đoán được nhóm chung cần tìm là gì: 2 2 (1) 2 3 2 2 1x x y x y x⇔ + − = + + + . Ta sẽ tìm nhóm chung bằng máy tính vớikỹ thuật sau: Xét 1x = ta có: 2 2 4 2 2 1 2 0y y y− = + + ⇒ = Xét x = 2 ta có: 2 1 2 10 2 2 2 3 2 y y y− = + + ⇒ = Nếu nhóm chung bật một ax by c+ + thì ta có : 0 2 11 22 0 2 a c c a x y b aa b c + = = − ⇔ ⇒ − −  = −+ + =  là nhóm chung. Đến đây ta sẽ đại lượng cần thêm vào các nhóm phần tử: ( ) ( ) ( ) 2 22 2 2 2 3 2 2 1 3 2 2 1 1 1x x y A x y A x x y x y x A x+ − − = − − ⇔ = + − − − − = + ⇒ = + Vậy đại lượng cần thêm vào 2 3 2x x y+ − chình là 1x + Bài giải : ĐK:
  • 11. ( ) ( ) 2 2 2 2 2 2 2 2 2 1 4 2 (1) 3 2 1 3 2 2 0 0 3 2 1 3 2 2 x y x y x y x x y x x x y x y x x y x x x y x y − − − − − ⇔ + − − + + + − − + = ⇔ + = + − + + + − + + ( ) ( )2 2 2 1 2 2 1 0 2 1 0 3 2 1 3 2 2 x y x y x y x x y x x x y x y  +  ⇔ − − + = ⇔ − − =  + − + + + − + +  (vì 2 0x y+ ≥ ) Thay 2 1y x= − vào phương trình (2) ta có: ( )2 3 6 1 8 5 2 1 4 3 1 3x x x x x− − + −= − + + Bình luận : Đến đây ta dễ dàng thấy được 2 6 1 2 1. 3 1x x x x− −= − + Điều này khiến ta nghĩ đến đến hai phép đặt ẩn phụ cơ bản: 2 1 4 3 1t x x= − + + hoặc 2 1 3 1 u x v x = −  = + để quy về pt bậc 2 theo 2 biến. Nhưng ta lại thấy phép đặt 2 1 4 3 1t x x= − + + không phù hợp với bài này vì không thể khử hết biến x trong phương trình Đặt : 2 1 0 3 1 0 u x v x = − ≥  = + ≥ . 2 6 1x x uv− − = , 2 2 8 5 2 6x u v− = + − ( )2 2 2 2 (3) 3 2 6 4 3 1 2 4 6 0uv u v u v u v u v v⇔ + + − = + ⇔ + − + − − = ( ) ( ) ( ) 2 22 3 1 4 2 4 6 5u v v v v∆ = − − − − = + ( ) ( ) ( ) ( ) 3 1 5 2 2 ( 0, 0) 2 (3) 3 1 5 3 2 v v u v loai vi u v v v u v − − − + = =− − ≥ ≥ ⇔ − − + + = =− +  2 9 5 0 3 2 1 3 1 3 2 (2 1)(3 1) 9 5 1 86 85 0 x u v x x x x x x x x − ≥ ⇔ + = ⇔ − + + = ⇔ − + = − ⇔ ⇔ = − + = Vậy ( ; ) (1;0)x y = là nghiệm duy nhất của phương trình. Bài tập tương tự: 1. ( ) ( ) 2 3 2 2 2 3 2 1 11 y x x y x y x y x  − + = −  + − − =
  • 12. 2. ( ) ( ) 2 1 2 1 2 3 6 1 2 2 4 5 3 y x y x x y y y x y x y x y  − − + = + − −  − + += − − − − 3. 2 16( ) 2 13 ( ) 3 5 x y x y xy x y y  + − + =  − + − − = Câu 9 (1,0 điểm).Cho x, y là hai số thực dương thỏa mãn điều kiện : ( )2 2 2 2 2 x y x y+ = . Tìm giá trị nhỏ nhất của biểu thức : 2 2 1 1 1 1 x y P y x x y = + + + + + + Bài giải : Ta có: ( ) ( ) 22 2 2 2 2x y x y x y xy x y= + ≥ + ⇒ ≥ + Do đó: ( ) ( ) ( ) ( ) 2 2 22 2 1 2 1 2 1 1x y x y xy x y x y x y+ + = + − + ≤ + − + + = + − 2 2 2 2 1 1 1 1 11 x y x y x yx y ⇒ + + ≤ + − ⇒ ≥ + −+ + ( ) ( ) 1 1 4 1 1 2 1 2 1 2 1 1 1 1 1 1 2 x y x y x y x y y x y x x y x y   + = + + + − = + + + − ≥ + + −  + + + + + + + +  Vậy ( ) 4 1 1 2 2 1 P x y x y x y ≥ + + + − + + + − Đặt: .t x y= + ta có : 2 ( ) 4 4( ) 4 4a b xy x y x y t+ ≥ ≥ + ⇒ + ≥ ⇒ ≥ Từ đó ta được : [ ) 4 4 1 2 ( ) , 4 , 2 1 t P f t t t t + ≥ + −= ∀ ∈ + ∞ + − Ta có: ( ) ( ) ( ) ( ) [ )2 2 22 3 44 1 '( ) 0 , 4 , ( 1)2 2 1 t t f t t tt t t − = − = ≥ ∀ ∈ + ∞ −+ + − Do đó ( )f t là hàm liên tục và đơn điệu tăng trên [ )4 , + ∞ [ ) [ ]4 , 5 5 ( ) (4) ( ) 3 3 f t f Min f x +∞ ⇒ ≥ ≥ ⇒ =khi 4t = Vậy ( ) 5 3 Min P = khi 2x y= = ĐỀ SỐ 2 Câu 1(2,0điểm).Cho hàm số 2 1 2 x y x + = + có đồ thị (C). 1. Khảo sát sự biến thiên và vẽ đồ thị (C) của hàm số trên.
  • 13. 2. Chứng minh rằng đường thẳng d: y x m=− + luôn cắt đồ thị (C) tại hai điểm phân biệt A và B. Tìm m để đoạn AB có độ dài nhỏ nhất. Câu 3 (1,0 điểm). 1. Giải phương trình:2sin 2 2 sin 2 5sin 3cos 3 4 x x x x π  + + + − =    2. Giải phương trình: ( )2 32 4 8log ( 1) 2 log 4 log 4x x x+ += − + + Câu 3 (1,0 điểm). 1. Tính tích phân: ( ) ( ) 2 1 30 1 12 16 4 1 3 1 x x I dx x x − + + = + + ∫ 2. Tính diện tích hình phẳng được giới hạn bởi các đường sau : 2 2 4 , 3 x y x y= − = Câu 4 (1,0 điểm). 1. Cho hai số phức 1z và 2z thỏa mãn điều kiện 1 22, 5z z= = và 1 2 4z z− =. Hãy tính mô đun của số phức 1 2z z z= + . 2. Tổ một có 3 học sinh nam và 4 học sinh nữ. Tổ hai có 5 học sinh nam và 2 học sinh nữ. Chọn ngẫu nhiên mỗi tổ một học sinh đi làm nhiệm vụ. Tính xác suất để chọn được hai học sinh gồm một nam và một nữ. Câu 5 (1,0 điểm).Cho hình chóp SABC, có cạnh SA vuông góc với đáy và AB = a, AC = 2a,  0 120BAC = . Mặt phẳng (SBC) tạo với đáy một góc 600 . Tính thể tích của khối chóp SABC và khoảng cách giữa hai đường thẳng SB và AC theo a . Câu 6 (1,0 điểm).Trong không gian với hệ trục tọa độ Oxy z, cho hai mặt phẳng (P): 2x + z – 3 = 0 và (Q): x + 2y + 3z – 4 = 0. Viết phương trình mặt phẳng vuông góc với hai mặt phẳng (P) và (Q), đồng thời hợp với các mặt phẳng tọa độ thành khối tứ diện có thể tích bằng 4 15 . Câu 7 (1,0 điểm).Trong mặt phẳng với hệ trục tọa độ Oxy, cho hình bình hành ABCD có phương trình đường chéo AC: 5x + y + 4 = 0. Gọi 23 15 ; 7 7 H   −    là trực tâm của tam giác ABC và 2 ;4 3 G   −    là trọng tâm tam giác ACD . Tìm tọa độ các đỉnh của hình bình hành A, B, C, D. Câu 8 (1,0 điểm). Giải hệ phương trình: ( )( ) 2 3 32 3 1 3 3 ( 1) 2 4 2( 2) y x y x y x x y x y  − − + − = −   − + − = − Câu 9 (1,0 điểm).Tìm tất cả các số thực m sao cho hệ phương trình sau có 4 nghiệm thực phân biệt.
  • 14. ( ) 2 2 2 2.3 .2 7.2 ( 4) 2 3 5 8 32 x y x y x y x y m x y y x y − − + −  + − =  + + + = + + ---- Hết --- GIẢI ĐỀ SỐ 2 Câu 1(2,0điểm).Cho hàm số 2 1 2 x y x + = + có đồ thị (C). Chứng minh rằng đường thẳng d: y x m=− + luôn cắt đồ thị (C) tại hai điểm phân biệt A và B. Tìm m để đoạn AB có độ dài nhỏ nhất. Bài giải: Phương trình hoành độ giao điểm của (C) và d là 2 1 (1) 2 x x m x + =− + + ĐK: 2x ≠ − 2 (1) ( ) (4 ) 2 1 0 (2)f x x m x m⇔ = + − − + = Ta có : ( ) 2 2 2 4 4( 2 1) 12 0 , ( 2) ( 2) (4 )( 2) 2 1 0 , m m m m R f m m m R ∆= − − − + = + > ∀ ∈ ⇔ − = − + − − − + ≠ ∀ ∈ (2) có hai nghiệm phân biệt khác -2 ⇔ (1) có hai nghiệm phân biệt , m R∀ ∈ ⇔ d cắt (C) tại hai điểm A, B phân biệt ( )dfcm . Gọi 1 2,x x là hai nghiệm phân biệt của (2), theo talet ta có: 1 2 1 24 , . 2 1x x m x x m+ =− =− + Khi đó ta có : ( )1 1;A x x m− + và ( )2 2;B x x m− + là hai giao điểm ( ) [ ] ( ) ( ) 22 2 2 1 2 1 2 1 2 1 2 1 2( ) ( ) 2 2 4AB x x x m x m x x x x x x= − + − + − − + = − = + − ( ) ( ) 2 2 2 4 4 2 1 2 12 24m m m= − − − += + ≥ Vậy ( ) 24Min AB = khi m = 0 Câu 3 (1,0 điểm). 1. Giải phương trình:2sin 2 2 sin 2 5sin 3cos 3 4 x x x x π  + + + − =    Bài giải: 2sin 2 (sin 2 cos2 ) 5sin 3cos 3 0pt x x x x x⇔ + + + − − = ( ) ( ) ( ) ( ) ( )( ) ( )( ) 2 2 6sin cos 1 2sin 5sin 3cos 3 0 3cos 2sin 1 2sin 5sin 2 0 3cos 2sin 1 sin 2 2sin 1 0 2sin 1 3cos sin 2 0 2sin 1 0 (1) sin 3cos 2 (2) x x x x x x x x x x x x x x x x x x x ⇔ + − + − − = ⇔ − − − + = ⇔ − − − − = ⇔ − − + = − = ⇔  − = 2 1 6 (1) sin 52 2 6 x k x x k π π π π  = + ⇔ =⇔  = + 
  • 15. 1 3 2 2 (2) sin cos cos cos sin sin 10 10 10 10 x x x xα α⇔ + = ⇔ + = ( )cos cos 2x x kα β α β π⇔ − = ⇔ = ± + Với: 3 cos 10 1 sin 10 α α  =   =  và 2 cos 10 β = 2. Giải phương trình: ( )2 32 4 8log ( 1) 2 log 4 log 4x x x+ += − + + Bài giải: ĐK: 4 4 1 x x − < <  ≠ − ( ) ( ) ( ) ( ) ( ) ( )( ) ( ) 2 3 1 22 2 2 1 32 2 2 22 2 2 2 2 2 2 22 2 2 2 log ( 1) log 2 log 4 log 4 log 1 log 4 log 4 log 4 log 4 1 log 4 4 16 0 4 4 2 4 4 164 4 16 4 12 0 2 2 6 4 4 16 4 20 0 pt x x x x x x x x x x x x x xx x x x x x x x x ⇔ + + = − + + ⇔ + + = − + + ⇔ += − +    − ≥ − ≤ ≤  = + = − ⇔ + = − ⇔ ⇔ ⇔+ − =    = −  + =− − − − =  Chú ý : Trong bài này sai lầm thường mắt phải là: ( ) ( )2 2 22 log 1 log 1x x+ = + . Cần nhớ 2 log 2 logn a ab n b= Câu 3 (1,0 điểm). 1. Tính tích phân: ( ) ( ) 2 1 30 1 12 16 4 1 3 1 x x I dx x x − + + = + + ∫ Bài giải: ( )( ) ( ) ( )( ) ( ) ( )( ) 1 1 1 1 20 0 0 1 4 1 3 1 1 1 4 4 11 1 3 1 1 1 3 1 x x I dx dx dx I I xx x x x x x − + + = = − =− ++ + + + + + ∫ ∫ ∫ 1 1 0 11 ( 1) ln 1 ln 2 01 I d x x x = + = + = +∫ ( ) 1 2 0 2 1 3 1 1 1 I dx x x x = + + + ∫ Đặt: ( ) 2 2 3 1 3 1 1 1 1 1 x x t t tdt dx x x x + + = ⇔ = ⇒ = + + +
  • 16. Đổi cận 1 2 0 1 x t x t  = ⇒ =  = ⇒ = 2 2 2 1 1 1 2 2 1 1 I tdt dt t t = = = = −∫ ∫ Vậy : 2 1 4ln 2I= − − 2. Tính diện tích hình phẳng được giới hạn bởi các đường sau : 2 2 4 , 3 x y x y= − = Bài giải: Phương trình hoành độ giao điểm : 2 2 2 2 4 4 3 4 9 36 0 3 3 x x x x x x x− = ⇔ − = ⇔ + − =⇔ =± 3 2 2 3 4 3 x S x dx − = − −∫ 2 2 2 2 2 4 1 3 , 3 0 3 4 0 31 3 x x x x xx  − ≥  ∀ ∈ − ⇔ ≤ ≤ ⇒ ⇒ − − ≥  ≤  3 3 32 2 3 2 2 1 1 3 3 3 3 2 3 4 4 3 3 9 33 x x x S x dx x dx dx I I − − −     = − − = − − = − = −    −    ∫ ∫ ∫ Đặt : 2sin 2cosx t dx tdt= ⇒ = Đổi cận: 3 3 3 3 x t x t π π  =− ⇒ =−   = ⇒ =  ( ) ( ) 3 3 3 2 2 1 3 3 3 1 43 4 1 sin 2cos 4 cos 2 1 cos2 2 sin 2 3 2 3 3 I t tdt tdt t dt t t π π π π π π π π π − − −   = − = = + =+ =+    − ∫ ∫ ∫ Vậy diện tích hình phẳng cần tìm là : 4 3 3 3 S π = + (đvdt) Câu 4 (1,0 điểm). 1. Cho hai số phức 1z và 2z thỏa mãn điều kiện 1 22, 5z z= = và 1 2 4z z− =. Hãy tính mô đun của số phức 1 2z z z= + . Bài giải:
  • 17. Gọi 1 1 1z x y i= + và 2 2 2z x y i= + với 1 2 1 2, , ,x x y y R∈ Khi đó ta có : ( ) ( ) 2 2 2 2 2 2 1 1 1 1 1 1 1 2 2 2 2 2 2 2 2 2 2 2 2 2 2 2 2 2 2 2 1 2 1 21 1 2 2 1 2 1 21 2 1 2 1 2 2 4 4 4 5 25 25 25 2 2 1316 2 24 16 z x y x y x y z x y x y x y x x y yx y x y x x y yz z x x y y    = += += +=     = ⇔ + = ⇔ + = ⇔ + =        + =+ + + = + +− = − + − =   ( ) ( ) 2 2 2 2 2 2 1 2 1 2 1 2 1 1 2 2 1 2 1 22 2 4 25 13 42z z z x x y y x y x y x x y y= + = + + + = + + + + + = + + = 2. Tổ một có 3 học sinh nam và 4 học sinh nữ. Tổ hai có 5 học sinh nam và 2 học sinh nữ. Chọn ngẫu nhiên mỗi tổ một học sinh đi làm nhiệm vụ. Tính xác suất để chọn được hai học sinh gồm một nam và một nữ. Bài giải: Gọi Ω là không gian mẫu của phép thử chọn ngẫu nhiên mỗi tổ một học sinh Khi đó ta có số phần tử của không gian mẫu là : 1 1 7 7 49C CΩ= × = Gọi A là biến cố hai học sinh được chọn là một học sinh nam và một học sinh nữ. Khi đó ta có số phần tử của không gian biến cố A được tính như sau: TH1: Chọn 1 học sinh nam từ tổ một và 1 học sinh nữ từ tổ hai ⇒có 1 1 3 2 6C C× =cách chọn TH1: Chọn 1 học sinh nữ từ tổ một và 1 học sinh nam từ tổ hai ⇒có 1 1 4 5 20C C× = cách chọn Theo quy tắc cộng ta có: 6 20 26AΩ = + = Vậy xác suất cần tìm là : 26 ( ) 49 A P A Ω = = Ω Câu 5 (1,0 điểm).Cho hình chóp SABC, có cạnh SA vuông góc với đáy và AB = a, AC = 2a,  0 120BAC = . Mặt phẳng (SBC) tạo với đáy một góc 600 . Tính thể tích của khối chóp SABC và khoảng cách giữa hai đường thẳng SB và AC theo a . Bài giải: Dựng AM ⊥ BC ( với M BC∈ ) ⇒BC ⊥ (SAM) (vì SA BC⊥ ) ( ) ( ) ( ),( ) ,SBC ABC MA MS SMA= = (Vì  0 90SAM = ) Từ  0 60gt SMA⇒ = Theo định lý cosin cho tam giác ABC ta có: C S A 0 120 0 60
  • 18. 2 2 2 . .cos 7BC AB AC AB BC BAC a= + − = 1 1 . .sin . 2 2 ABCS AB AC BAC AM BC= = . .sin 21 7 AB AC BAC a AM BC ⇔= = Xét SAM∆ ta có:   7 tan tan 7 SA a SMA SA AM SMA AM = ⇔ = =  3 1 1 1 21 . . . .sin 3 3 2 42 SABC ABC a V SA S SA AB AC BAC= = = (đvdt) Dựng hình bình hành ACBD như hình vẽ . Khi đó ta có : AC // BC ⇒ AC// (SBC) 3 ( , ) ( ,( ) (1)SABD SBD V d AC SB d A SBD S ⇒ = = Ta có: 3 21 42 SABD SABC a V V= = Theo Pitago ta có: 2 2 5 14 7 a SD SA AD= + = , 2 2 2 14 7 a SB SA AB= + = Theo định lý cosin cho SBD∆ ta có:  ( ) 2 2 2 3 cos 2. . 4 SB SD BD BSD SB SD + − = =  2 7 sin 1 cos 4 BSD BSD⇒ =− =  ( ) 2 1 5 7 . .sin 2 14 SBD a S SB SD BSD= = 3 2 21 3 342(1) ( , ) 55 7 14 a a d AC SB a ⇔ = = (đvđd) Câu 6 (1,0 điểm).Trong không gian với hệ trục tọa độ Oxy z, cho hai mặt phẳng (P): 2x + z – 3 = 0 và (Q): x + 2y + 3z – 4 = 0. Viết phương trình mặt phẳng vuông góc với hai mặt phẳng (P) và (Q), đồng thời hợp với các mặt phẳng tọa độ thành khối tứ diện có thể tích bằng 4 15 . Bài giải: ( )( ) : 2 3 0 2; 0;1pP x z n+ − = ⇒ =  là VTPT của mặt phẳng (P) ( )( ) : 2 3 4 0 1; 2; 3QP x y z n+ + − = ⇒ =  là VTPT của mặt phẳng (Q)
  • 19. Ta có: ( ) ( ) ( ) ( ) R P R Q ⊥ ⇒ ⊥ mp (R) có VTPT là ( ), 2 5 4R p Qn n n = = − − +     ( ) : 2 5 4 0R x y y m⇒ + − + = ( ) ; 0; 0 2 2 ( ) 0; ; 0 5 5 ( ) 0; 0; 4 4 mm A R Ox A OA mm B R Oy B OB mm C R Oz B OB    = ∩ ⇒ − ⇒ =        = ∩ ⇒ − ⇒ =        = ∩ ⇒ ⇒ =     3 34 1 4 . . 4 4 4 15 6 15 OABCV OAOB OC m m m= ⇔ = ⇔ = ⇔ = ⇔ = ± Vậy có hai mặt phẳng cần tìm là: ( ) : 2 5 4 4 0R x y y+ − + = và ( ) : 2 5 4 4 0R x y y+ − − = Câu 7 (1,0 điểm).Trong mặt phẳng với hệ trục tọa độ Oxy, cho hình bình hành ABCD có phương trình đường chéo AC: 5x + y + 4 = 0. Gọi 23 15 ; 7 7 H   −    là trực tâm của tam giác ABC và 2 ;4 3 G   −    là trọng tâm tam giác ACD . Tìm tọa độ các đỉnh của hình bình hành A, B, C, D. Bài giải: BH đi qua H và vuông góc AC : 5 14 0BH x y⇒ − + = ( )5 14;B BH B m m∈ ⇒ − Gọi I là giao điểm của AC và DB ( ; 5 4)I AC I a a⇒ ∈ ⇒ − − G là trọng tâm tam giác ACD 4BG IG⇒ =   ( ) ( ) 2 2 3 3 7 5 14 4 ; 3 3 2 2 2 4 4 4 5 4 2 4 ; 2 m a a I m a m B      − − + = − − =− ⇒ −     ⇔ ⇔      − = + + = ⇒ −  I là trung điểm của BD ( ) 2 2 1 1; 5 2 2 5 B D I D I B B D I D I B x x x x x x D y y y y y y + = = − =  ⇒ ⇔ ⇒  + = = − =  Ta có ( ); 5 4A AC A a a∈ ⇒ − − Lại có I là trung điểm của AC ( )3 ;11 5C a a⇒ − − + ( )4 ; 6 5AB a a= − − +  và 7 2 35 62 ; 7 7 a a CH − − −  =      Ta có H là trực tâm của tam giác ABC . 0 ,AB CH a A C⇒ = ⇒ ⇒   Câu 8 (1,0 điểm). Giải hệ phương trình: ( )( ) 2 3 32 3 1 3 3 ( 1) (1) 2 4 2( 2) (2) y x y x y x x y x y  − − + − = −   − + − = − A B C D . 2 ;4 3 G   −    23 15 ; 7 7 H   −    5x + y + 4 = 0 I
  • 20. Bình luận:Dễ thấy phương trình (1) chỉ chứa hai nhóm biến là : x và 1y − điều này khiến ta nghĩ ngay đến phương trình thuần nhất (đồng bậc) hoặc cấu trúc hàm số ( ) ( )1f x f y= − hoặc phương trình tích. Từ đó để thực hiện được ý tưởng này ta nên đặt hai ẩn phụ để quy phương trình (1) về dạng đa thức trong sáng hơn, sau đó ta mới xem thử nó cụ thể là thuộc dạng nào. Bài giải: ĐK: 2 1 0 x y x  ≥ ≥  ≥ Dễ thấy y = 1 không thỏa hệ phương trình. Nên ta chỉ xét 1y > . Đặt : 1 0 0 u y v x  = − >  = ≥ ( )2 2 2 4 3 4 2 2 3 4 (1) 3 3 0u v u v u v v u v u v u⇔ − − − = ⇔ + + − = Chia hai vế cho 4 u và đặt 0 v t u = ≥ ta có ( ) ( )4 2 3 2 3 2 1 1 1 1 3 0 ( 1) 2 3 0 2 3 0 ( , 0)` t u v y x y x t t t t t t t t t t VN vi t  = ⇔ = ⇔ − = ⇔ = + ⇔ + + − = ⇔ − + + + = ⇔  + + += > Thay 1y x= + vào phương trình (2) ta có: 32 3 1 2 4 2( 1) (*)x x x x− − + − = − Bình luận:pt (*) chứa 2 loại căn và 3 điều này khiến ta nghĩ đến các hướng phân tích sau: Hướng 1: sử dụng hàm số để chứng minh nghiệm duy nhất (phải đoán được 1 nghiệm), Hướng 2: phân tích về phương trình tích bằng nhân liên hợp (phải đoán được nghiệm hoặc đoán được nhóm phần tử chung), Hướng 3: hoặc quy về phương trình thuần nhất bậc 3. Đặt 3 t = sau đó bình phương hai vế. Hướng 4: Đặt hai ẩn phụ để quy về phương trình bậc 3 theo hai biến hoặc quy về hệ phương trình hai biến không căn thức. Nhưng pt(*) không đoan nghiệm được và trong 3 là biểu thức bậc 3 theo biến x , nên ta chỉ có thể đi theo hướng nhân liên hợp để xây dựng được nhóm chung là biểu thức bậc 2.Vấn đề bây giờ là làm sao có thể đoán được nhóm chung này, ta thử xem : 3 3 3 3 2 4 ( 1) 3( 1)x x x x− − − = − − . Từ đây ta có lời giải: 32 3 (*) 1 2 4 ( 1) 0x x x x ⇔ − − + − − − =   2 2 2 3 33 3 2 3( 1) 1 2 0 4 ( 2) 4 ( 2) x x x x x x x x − − ⇔ − − + = − + − − + − 2 2 2 3 33 3 2 6 1 1 1 0 4 ( 2) 4 ( 2) x x x x x x x x  − −  ⇔ − − + =  − + − − + −  2 2 2 2 3 3 1 0 6 1 1 0 ( ) 2 3( 2) 4 2 4 x x x x VN x x x  − − =  − −⇔ + = − −  − + +   
  • 21. ⇔ 1 5 2 x − = (loại) hoặc 1 5 2 x + = 3 5 2 y + ⇒ = Vậy hệ phương trình có nghiệm duy nhất: ( ) 1 5 3 5 ; ; 2 2 x y  + + =      Kinh nghiệm: Khi dùng máy tính để bấm nghiệm pt(*) ta nên kiểm tra xem nó có giống nghiệm của một trong các biểu thức nằm dưới dấu căn không. Bài tập tương tự: Giải hệ phương trình : ( ) ( ) 2 3 2 2 2 2 2 12 2 2 2 4 (1) 4 3 5 4 8 (2) x x y y x y y y x y x y  + − = − −   + + − = + + HD: ( ) ( )24 2 2 (2) 2 3 8 5y t y t t⇔ − = − + − . Sử dụng BBT của hàm 2 ( ) 3 8 5f t t t= − + − ta chứng minh được ( ) 0f t ≤ dấu bằng xảy ra khi t = 2. Từ đó: (2) 2x y⇔ = Thay x = 2y vào (1) : 2 2 3y y y y− − = − + (*) Để (*) có nghiệm thì : 2 1 2 0 2 y y y y ≤ − − − ≥ ⇔  ≥ kết hợp với ĐK 0 3y≤ ≤ ta có 2 3y≤ ≤ Kỷ năng: Với bài này ta không được may mắn như bài trên . Vì thể ta sẽ đi tìm nhóm chung bằng kỷ thuật them bớt như sau: ( ) 2 2 2 2 2 2 (*) 3 2 ( ) 3 ( ) 0 (2 1) 3 (2 1) 3 2 0 ( ) 3 ( ) y y m n y m y y n y y m y m y n y n y y m n y m y y n y ⇔ − − − − + + − − + + − = + + + − + − + ⇔ − − − − + + = + + − + + Khi đó ba nhóm trên có phần tử chung thì hệ sau phải có nghiệm: 2 2 2 1 3 2 1 3 2 3 2 1 2 m n m m m n n n m n + =−  − =− = − ⇔  − =− − − =−  =− − − 2 2 2 2 3 1 3 1 (*) 3 1 ( 2) 3 ( 1) 0 3 1 0 2 3 1 y y y y y y y y y y y y y y y y − + − + ⇔ − + + − − − + − − = ⇔ − + + + = − + − − + ( ) [ ]2 21 1 1 1 3 1 1 0 3 1 0 1 0, 2 , 3` 2 3 1 2 3 1 y y y y vi y y y y y y y y y     ⇔ − + + + = ⇔ − + = + + > ∀ ∈      − + − − + − + − − +    Câu 9 (1,0 điểm).Tìm tất cả các số thực m sao cho hệ phương trình sau có 4 nghiệm thực phân biệt.
  • 22. ( ) 2 2 2 2.3 .2 7.2 (1) ( 4) 2 3 5 8 32 (2) x y x y x y x y m x y y x y − − + −  + − =  + + + = + + Bài giải: Đặt: t x y= − 2 3 (1) 2.3 .2 7.2 2 4 7 2 t t t t t t+   ⇔ + = ⇔ + =    Xét hàm số : 3 ( ) 2 4 2 t f t t   = +    ta có: 3 3 '( ) 2 ln 4 0 , 2 2 t f t t R     = + > ∀ ∈        Suy ra hàm số: 3 ( ) 2 4 2 t f t t   = +    liên tục và đơn điệu tang trên R. Mà (1) ( ) (1)f t f⇔ = nên t = 1 là nghiệm duy nhất của phương trình(1). 1 1 1t x y y x= ⇔ − = ⇔ = − Thay 1y x= − vào phương trình (2) ta có: ( ) 2 2 4 2 5 8 24 (3)m x x x x+ + = + + Dễ thấy x = -4 không phải là nghiệm của phương trình m R∀ ∈ ( ) ( ) 2222 2 2 4 2 (3) 4 2 4 4 2 4 42 x x m x x x x m xx + + ⇔ + + = + + + ⇔ = + ++ Đăt: 2 4 2 x u x + = + 3 2 2 4 ' 0 2 2 x u x x − = = ⇔ = + , 2 4 lim lim 1 2x x x u x→−∞ →−∞ + = = − + và 2 4 lim lim 1 2x x x u x→+∞ →+∞ + = = + BBT: Từ BBT ta thấy ( 1, 6x R t ∀ ∈ ⇒ ∈ −  4 (3) ( )m t g t t ⇔ = + = 2 2 2 24 4 '( ) 1 0 2 ( ) tt g t t loait t =− =− = = ⇔  = −  BBT: Từ BBT ta có: Hệ phương trình có nghiệm x 'u u −∞ +∞2 0+ - -1 1 6 t '( )g t 1− 62 0- + 10 6 0 - +∞
  • 23. ⇔ Phương trình (3) có nghiệm x ⇔ Phương trình ( )g t m= có nghiệm ( 1, 6t ∈ −  4 5 m m ≥ ⇔  < − Vậy hệ phương trình có nghiệm khi 4m ≥ hoặc 5m < − ĐỀ SỐ 3 Câu 1( 2,0 điểm):Cho hàm số ( )2 3 2 5 6 6 6y m m x mx x=− + + + − Với m là tham số 1. Khảo sát sự biến thiên và vẽ đồ thị của hàm số trên khi m = 1. 2. Tìm m để hàm số trên đơn điệu. Khi đó hàm số trên đồng biến hay nghịch biến. Câu 2(1,0 điểm). 1. Giải phương trình 1 2cos sin 2 6 6 2 x x π π    − − − =        2. Giải phương trình: ( ) ( ) ( )2 2 2 3 4 16log 1 .log 1 log 1x x x x x x− − + − = − − Câu 3 (1,0 điểm). 1. Tính tích phân: 5 4 40 cos5 .cos 1 cos x x I dx x π + = ∫ 2. Tính tích phân : 3 2 1 ( 1)ln 2 1 2 ln e x x x I dx x x + + + = +∫ Câu 4 (1,0 điểm). 1. Tìm tập hợp điểm biểu diển của số phức 1w z i= + − biết rằng số phức ( ) 2 2 i z iz z+ − − là một số thuần ảo. 2. Có 40 tấm thẻ được đánh số từ 1 đến 40. Chọn ngẫu nhiên 10 tấm . Tính xác suất để có 5 tấm thẻ mang số lẻ, 5 tấm thẻ mang số chẵn trong đó có đúng một tâm thẻ mang số
  • 24. chia hết cho 6. Câu 5 (1,0 điểm). Cho hình chóp SABCD có đáy là hình chữ nhật tâm O, AB = a, AD = 2a; M là điểm thuộc cạnh AB sao cho MA = 2MB, tam giác SOM cân tại đỉnh S và nằm trong mặt phẳng vuông góc với (ABCD). Biết mặt bên (SBC) hợp với mặt phẳng đáy (ABCD) một góc bằng 600 . Tính theo a thể tích khối chóp SAMOD và tính khoảng cách từ M đến mặt phẳng (SCD). Câu 6 (1,0 điểm).Trong không gian với hệ trục Oxyz, cho đường thẳng 1 6 : 1 1 1 x y z d − + = = Viết phương trình mặt phẳng chứa đường thẳng d và tạo với mặt phẳng (Oyz) một góc bằng 450 . Câu 7 (1,0 điểm).Trong mặt phẳng với hệ trục tọa độ Oxy, cho hình vuông ABCD có I là giao điểm hai đường chéo, G là trọng tâm của tam giác ABI và điểm E(7; -2) thuộc đường chéo BD sao cho GA = GE. Tìm tọa độ các đỉnh của hình vuông A, B, C, D biết đường thẳng GA có phương trình 3x – y -13 = 0 và điểm A có hoành độ nhỏ hơn 4. Câu 8 (1,0 điểm). Giải Bất phương trình sau: ( )2 2 2 3 3 8 3 2 1 0 1 3 4 5 x x x x x x + + + − + ≤ − − + Câu 9 (1,0 điểm).Tìm giá trị lớn nhất, nhỏ nhất của hàm số sau : 5 4 1 5 4 2 1 6 x x y x x − − + = − + + + --- Hết --- GIẢI ĐỀ SỐ 3 Câu 1( 2,0 điểm):Cho hàm số ( )2 3 2 5 6 6 6y m m x mx x=− + + + − Với m là tham số Tìm m để hàm số trên đơn điệu. Khi đó hàm số trên đồng biến hay nghịch biến. Bình luận :Để hàm số 3 2 ( )y f x ax bx cx d= = + + + đơn điệu( một chiều biến thiên) thì có hai khả năng : hoặc là hàm số đó quy về bậc một (tức a = b = 0) hoặc '( )f x không đổi dấu trên TXĐ . Bài giải : TH1 : ( )2 5 0 0 5m m m m− + =⇔ =∨ =− Với m = 0 ta có hàm số trở thành : 6 6y x= − (vì a = 6 > 0)đơn điệu tăng trên R ⇒ m = 0 thỏa Với m = -5 ta có hàm số trở thành : 2 30 6 6y x x= + − có đồ thị là một parabol nên không đơn điệu⇒ m = -5 không thỏa đề toán TH2 : ( )2 0 5 0 (*) 5 m m m m ≠ − + ≠ ⇔  ≠ − Khi đó : ( )2 2 ' '( ) 3 5 12 6y f x m m x mx= =− + + + là một tam thức bậc hai, nên ta có : Hàm số đơn điệu khi '( )f x không đổi dấu trên TXĐ 2 54 90 0m m⇔ ∆= + ≤ 5 0 3 m⇔ − ≤ ≤ Kết hợp điều kiện (*) ta có : 5 0 3 m− ≤ < Vậy hàm số đơn điệu khi 5 0 3 m− < ≤ (vì m = 0 thỏa theo TH1) m ( )2 3 5a m m=− + -5 0 0 + 5 3 − - -+
  • 25. Bảng xét dấu của hệ số a của '( )f x : Từ BXD trên ta thấy : ( )2 2 3 5 0 5 5 , , 0 '( ) 0, , , 0 3 354 90 0 a m m m f x x R m m m  =− + >     ∀ ∈ − ⇔ ≥ ∀ ∈ ∀ ∈ −      ∆= + ≤ ⇒hàm số ( )f x đơn điệu tăng khi 5 , 0 3 m   ∈ −   Vậy 5 , 0 3 m   ∈ −   thì hàm số đởn điệu tăng. Câu 2(1,0 điểm). 1. Giải phương trình 1 2cos sin 2 6 6 2 x x π π    − − − =        Bình luận:Với phương trình lương giác có chứa cung hằng số dạng ( ) ( ) ( )1 1 2 2, ,..., n na x a x a xα α α+ + + thì ta sẽ có 4 định hướng sau : Hướng 1: Sử dụng công thức công (thướng thì lời giải sẽ hơi cồng kềnh) Hướng 2: Sử dụng cống thưc hạ bậc rồi dùng công thức cộng (thường chỉ dùng cho dạng 2 cos 4 ax π  ±    hoặc 2 sin 4 ax π  ±    ) Hướng 3: Sử dụng công thức tổng thành tích hoặc tích thành tổng (thường thì phương trình có dạng tích của hàmlương giác chứa cung hằng số hoặc chứa các số: 1 3 ; 2 2 ± ± đi với cung ; 3 6 π π ) Hướng 4: Đặt ẩn phụ ( )i it a x α= + với { }1 2, , ...,i na Min a a a= Bài giải :Ta có : 1 sin 2 6 π = 2cos sin 2 sin 2cos 2sin cos 6 6 6 6 6 pt x x x x x π π π π π        ⇔ −= − + ⇔ −= −                ( ) ( ) sin 1 2 2cos 1 sin 0 26 cos 0 6 3 x x k x x k Z x x k π π π π π π  = ⇔ = +  ⇔ − − =⇔ ∈      − = ⇔ = +    Bài tập tương tự: 2 1 sin3sin4 6 sin =++      − xxx π HD: 1 sin 2 6 π = , sin3 sin 2sin 2 cos sin 4 sin 2cos 2 sin 2 6 6 6 x x x x x x x π π π + =       − − = −              +=      − 2 3 10 sin 2 1 210 3 sin xx ππ HD: Đặt 3 10 2 x t π = − xxxx 2cos 6 sin 6 sin.cos4 =      −      + ππ HD: 1 1 1 sin sin cos2 cos cos2 6 6 2 3 4 2 x x x x π π π      + − =− − =−           
  • 26.       −= + +− 24 cos8 cos )sin1(3 tantan3 2 2 3 x x x xx π HD: 2 1 cos 2 8cos 8 4 4sin 4 2 2 x x x π π   + −     − = = +    2. Giải phương trình: ( ) ( ) ( )2 2 2 3 4 16log 1 .log 1 log 1x x x x x x− − + − = − − Bài giải :ĐK : 1x ≥ ( ) ( ) ( )2 2 2 3 4 16 3log 1 .log 1 log 3.log 1 0pt x x x x x x⇔ − − + − − − − = ( ) ( ) 2 2 2 3 4 4 log 1 log 1 log 3 0x x x x ⇔ − − + − − =    ( ) ( ) 2 2 3 2 4 4 log 1 0 (1) log 1 log 3 (2) x x x x  − − =  ⇔  + − =  2 2 2 2 1 0 (1) 1 1 1 1 1 1 2 1 x x x x x x x x x − ≥ ⇔ − − = ⇔ − = − ⇔ ⇔ = − = − + ( )2 2 4 4 4 2 2 3 01 2 3 (2) log 1 log 3 log 3 1 3 2 31 2 3 3 x x x x x x x x x  − ≥ ⇔ + − = = ⇔ − = − ⇔ ⇔ = − = − + Vậy phương trình có hai nghiệm: 1x = và 2 3 3 x = Câu 3 (1,0 điểm). 1. Tính tích phân: 5 4 40 cos5 .cos 1 cos x x I dx x π + = ∫ Bình luận: Với tích phân lương giác không chứa căn thức và mẫu thức thì ta có thể dùng công thức hạ bâcvà công thức tích thành tồng rồi sử dụng công thức: ( ) ( ) 1 sin cosax dx ax c a =− +∫ và ( ) ( ) 1 cos sinax dx ax c a = +∫ Bài giải : 5 4 4 4 1 24 40 0 0 cos5 .cos 1 1 cos5 .cos cos cos x x I dx x xdx dx I I x x π π π + = = + =+∫ ∫ ∫ ( )4 1 0 1 1 1 1 1 cos6 cos4 sin 6 sin 4 4 2 2 6 4 12 0 I x x dx x x π π   = + = + =−    ∫ ( ) 3 24 4 4 2 4 2 20 0 0 1 1 1 tan 4 . tan 1 (tan ) tan 4 cos cos cos 3 3 0 x I dx dx x d x x x x x π π π π   = = = + = + =    ∫ ∫ ∫
  • 27. Vậy 1 2 1 4 5 12 3 4 I I I= + =− + = 2. Tính tích phân : 3 2 1 ( 1)ln 2 1 2 ln e x x x I dx x x + + + = +∫ Bình luận:Với tích phân có căn hoặc mẫu số chứa nhóm có dạng: ln ln , , sin , cos ,x x x x xe x x x x x thì ta thử đạo hàm nhóm đó và so sánh với những nhóm còn lại để chọn ra phương án giải là đặt ẩn phụ hay tich phân từng phần (nếu cần thì ta cũng có thể tách tích phâncho phù hợp).cụ thể như ví dụ trên ta sẽ xét: ( )2 ln ' 1 lnx x x+ =+ điều này khiến ta nghĩ đến việc tách tử số thành : 3 2 2 ( 1)ln 2 1 (2 ln ) (1 ln )x x x x x x x+ + += + + + Bài giải : 2 2 1 1 1 (2 ln ) (1 ln ) 1 ln 2 ln 2 ln e e ex x x x x I dx x dx dx x x x x + + + + = = + + +∫ ∫ ∫ ( ) 3 1 1 2 ln 13 2 ln eex d x x x x =+ + +∫ 3 3 1 1 ln 2 ln ln(2 ) ln 2 13 3 3 3 ee e x x e= − + + = − + + − ĐS: 3 1 ln(2 ) ln 2 3 3 e I e= − + + − Câu 4 (1,0 điểm). 1. Tìm tập hợp điểm biểu diển của số phức 1w z i= + − biết rằng số phức ( ) 2 2 i z iz z+ − − là một số thuần ảo. Bài giải : Gọi , ,w x yi x y R= + ∀ ∈ Ta có: ( ) ( ) ( ) ( ) ( ) ( ) 2 2 2 1 1 1 1 1 1 1 1 z x y i w z i z w i x y i z x y  = − − + = + − ⇔ = − + = − + + ⇒  = − + + ( ) ( ) ( )( ) ( ) ( ) ( ) ( ) 2 2 2 ' 2 2 1 1 1 1 1 1z i z iz z i x y i i x y i x y = + − − = + − − + − − + + − − + +     ( ) ( )2 2 4 3 2 1x y x y x y i= − − + − − + − − + 'z là số thuần ảo ( ) 2 22 2 2 4 3 04 3 0 2 1 0 5 11 8 0 * x y x yx y x y x y y y  + − + + =− − + − − =  ⇔ ⇔  − − + ≠ − + ≠  ĐK: ( )* đúng y R∀ ∈
  • 28. Vậy tập hợp điểm biểu diển của w là một đường tròn tâm 1 2; 2 I   −    và bán kính 5 2 R = 2. Có 40 tấm thẻ được đánh số từ 1 đến 40. Chọn ngẫu nhiên 10 tấm . Tính xác suất để có 5 tấm thẻ mang số lẻ, 5 tấm thẻ mang số chẵn trong đó có đúng một tâm thẻ mang số chia hết cho 6. Bài giải : Trong 40 tấm thẻ có 20 tấm mang số lẽ, 6 tấm mang số chia hết cho 6 và 14 tấm mang số chẵn không chia hết cho 6 Gọi Ω là không gian mẫu của phép thử chọn 10 tấm thẻ từ 40 tấm thẻ ⇒số phần tử của không gian mẫu là : 10 40CΩ = Gọi A là biến cố trong 10 tấm thẻ được chọn có 5 tấm thẻ mang số lẻ, 5 tấm thẻ mang số chẵn trong đó có đúng một tâm thẻ mang số chia hết cho 6 Khi đó số phần tử của không gian biến cố A được xác định như sau: Bước 1: chọn 5 tấm thẻ mang số lẻ từ 20 tấm thẻ ⇒ có 5 20C cách chọn Bước 2: chọn 1 tấm thẻ mang số chia hết cho 6 từ 6 tấm thẻ ⇒ có 1 6C cách chọn Bước 3: chọn 5 tấm thẻ mang số chẵn không chia hết cho 6 từ 14 tấm thẻ ⇒có 4 14C cách chọn Theo quy tắc nhân ta có: 5 1 4 20 6 14A C C CΩ = Vậy xác suất của biến cố A là 5 1 4 20 6 14 10 40 ( ) ...A C C C P A C Ω = = = Ω Câu 5 (1,0 điểm). Cho hình chóp SABCD có đáy là hình chữ nhật tâm O, AB = a, AD = 2a; M là điểm thuộc cạnh AB sao cho MA = 2MB, tam giác SOM cân tại đỉnh S và nằm trong mặt phẳng vuông góc với (ABCD). Biết mặt bên (SBC) hợp với mặt phẳng đáy (ABCD) một góc bằng 600 . Tính theo a thể tích khối chóp SAMOD và tính khoảng cách từ M đến mặt phẳng (SCD). Bài giải : Gọi H là trung điểm OM SH OM⇒ ⊥ (vì SMN∆ cân tại S) Mặt khác ( )SOM và ( )ABCD vuông góc nhau theo gao tuyến OM ( )SH ABCD⇒ ⊥ . Dựng HK ⊥ BC (với K∈ BC) Khi đó ta có : ( )SHK BC⊥ ( ) ( ),( )SBC ABCD SKH⇒ =,  0 60gt SKH⇒ = Gọi E là trung điểm BC . Khi đó BMOE là hình thang có HK là đường trùng bình : 5 2 12 BM EO a HK + ⇒= = Xét tam giác SHK vuông tại H ta có : S A B D C H O M K E F
  • 29.  05 5 3 tan tan 60 12 12 SH a a SKH SH HK = ⇔ = = ( ) 2 1 1 1 1 5 5 . . 0, . . . 2 2 2 2 3 2 12 6 AMOD ABD OMB AB AD a S S S AB AD MB d AB AB AD AB AD= − = − = − = = 2 3 1 1 5 3 5 25 3 . 3 3 12 6 216 SAMOD AMOD a a a V SH S= = = (đvtt) Ta có MA // (SCD) ( ) ( ) 3 ,( ) ,( ) SACD SCD V d M SCD d A SCD S ⇒ = = 3 1 1 5 3 . . 3 2 36 SACD a V SH AD AB= = Dựng HF ⊥ CD (với F ∈ CD) . Khi đó ta có : 3 3 4 2 a HF KC AD= = = Theo Pi ta go cho tam giác SHF ta có : 2 2 2 2 5 3 3 399 12 2 12 a a a SF SH HF     = + = + =        2 1 399 . 2 24 SCD a S SF CD= = ( ) ( ) 3 10 10 133 ,( ) ,( ) 133133 SACD SCD V a a d M SCD d A SCD S = = = = ĐS : 3 5 3 36 SACD a V = (đvtt) và ( ) 10 133 ,( ) 133 a d M SCD = (đvđd) Câu 6 (1,0 điểm).Trong không gian với hệ trục Oxyz, cho đường thẳng 1 6 : 1 1 1 x y z d − + = = Viết phương trình mặt phẳng chứa đường thẳng d và tạo với mặt phẳng (Oyz) một góc bằng 450 . Bình luận :Với bài toán lập phương trình mặt phẳng có liên qua đến góc hoặc khoảng cách thì thông thường ta giải bằng cách gọi phương trình tổng quát ( ): 0P ax by cz d+ + + =( ĐK : 2 2 2 0a b c+ + > ) Sau đó sử dụng 2 dữ kiện bậc một (đi qua điểm hoặc VTPT vuông góc vectơ cho trước) để chuyển (P) theo hai tham số. cuối cùng sử dụng công thức góc hoặc khoảng cách để lập phương trình bậc hai theo hai tham số đó rồi chọn giá trị cho một tham số để tìm tham số còn lại. Bài giải : Ta có tọa độ (1;0; 6)A − và (0; 1; 12)B − − đồng thời thỏa phương trình d A d⇒ ∈ , B d∈ (Oyz): 0x = có VTPT là (1;0;0)Oyzn =  Gọi (P): 0ax by cz d+ + + =( ĐK : 2 2 2 0a b c+ + > )có VTPT là ( ; ; )Pn a b c=  Ta có : ( ) ( ): ( 6 ) 6 0( ) 6 0 6 ( ) ( ) 12 0 6 ; 6 ;P P ax a c y cz c aA P a c d d c a d P B P b c d b a c n a a c c + − − + + − =∈ − + = = −    ⊂ ⇒ ⇔ ⇔ ⇒    ∈ − − + = =− − = − −     ( ) ( ) ( ) 22 2 . cos ( ),( ) cos , 6 P Oyz P Oyz P Oyz n n a P Oyz n n n n a a c c = = = + − − +      
  • 30. 2 2 2 2 2 0 1 2 2 12 37 12 22 12 37 37 c a gt a a ac c a ca ac c = ⇔ = ⇔ = + + ⇔  = −+ +  Với 0 ( ): 1 0c P x y= ⇒ − − = Với 12 ( ):37 35 12 109 0 37 a c P x y z=− ⇒ + − − = Vậy phươg ntrình mặt phẳng cần tìm là 37 35 12 109 0x y z+ − − = Câu 7 (1,0 điểm).Trong mặt phẳng với hệ trục tọa độ Oxy, cho hình vuông ABCD có I là giao điểm hai đường chéo, G là trọng tâm của tam giác ABI và điểm E(7; -2) thuộc đường chéo BD sao cho GA = GE. Tìm tọa độ các đỉnh của hình vuông A, B, C, D biết đường thẳng GA có phương trình 3x – y -13 = 0 và điểm A có hoành độ nhỏ hơn 4. Bình luận :Với bài toán hình vuông ABCD trong tọa độ phẳng ta có thể giải bằng cách xét một hình đồng dạng với nó là A’B’C’D’ với các đỉnh ta đều xác định cụ thể . Ta đi xác định các tính chất về góc và tỷ số độ dài trong hình A’B’C’D’. Vì hai hình đồng dạng nhau thì tính chất về góc và tỷ số độ dài không thay đổi nên các tính chất đó ta có thể áp đặt vào hình ABCD của đề toán . Bài giải : Xét hình vuông A’B’C’D’ cạnh bằng 2, khi đó tọa độ được xác định như hình vẽ: Ta có : ( )' ' ': 2 0 ' ; 2E B D x y E m m∈ + − = ⇒ − ( ) 2 2 2 21 1 ' ' ' ' 1 1 3 3 G E G A m m     = ⇔ − + − = +        2 0 ( ) 6 8 0 4 3 m loai m m m = ⇔ − = ⇔  =   4 2 ' ; 3 3 E   ⇒     1 ' ' ; 1 3 G A   = − −     và 1 ' ' 1; 3 G E   = −     Ta có : ' '. ' ' 0 ' ' ' 'G A G E G A G E=⇒ ⊥   và 2 2 2 2 4 4 3 3' ' 4 6 8 . ' ' 3 5 55 5 6 6 E B E F     +        = = =     +        Vì A’B’C’D’ và BACD đồng dạng nhau nên tính chất về góc và tỷ số độ dài của hai hình sẽ không đổi (1) ' ' 8 (2) ' ' 5 AG GE EB E B EF E F ⊥  ⇒  = = Từ (1) kết hợp với giã thuyết ta có : GE qua E và vuông góc GA : 3 1 0EG x y⇒ + − = G là giao điểm của AG và EG ( )4; 1G⇒ − ( ); 3 13A AG A m m∈ ⇒ − (Với m < 4). A’(0; 0) B’(0; 2) D’(2; 0) C’(2; 2) I’(1; 1)G( 1 3 ; 1) x y E’ F’( 1 3 ; 2 2 )
  • 31. Ta có: ( ) ( ) 2 2 3 4 3 12 10 4 1 (3; 4) 5 ( ) m AG GE m m m A m loai = = ⇔ − + − = ⇔ − = ⇔ ⇒ − =  Gọi F(a ; b) là trung điểm BI ta có: 4 3 2( 4) 9 1 2 ; 1 ( 4) 2( 1) 2 2 a AG GF F b − = −   =⇔ ⇒  − − − = +     ( ) 9 5( 7) 8 7 32 (2) 5 8 (3;2) 21 5 2 8 2 2 B B B B x x EB EF B y y    − = −  =   ⇔ = ⇔ ⇔ ⇒  =   + = +      F là trung điểm BI (6; 1)I⇒ − I là trung điểm AC ( )9; 2C⇒ . I là trung điểm BD ( )9; 4D⇒ − Vậy các điểm cần tìm là: (3; 4)A − , (3;2)B , ( )9; 2C và ( )9; 4D − Bình luận: • Trong bài này ta có thể chứng minh AG GE⊥ bằng cách khác như sau: Dễ thấy IG là trung trực của AB ⇒ GA = GB ⇒ G là tâm đường tròn ngoại tiếp tam giác ABE   0 2 90AGE ABE⇒ = = .Nhưng khi đó để tìm điểm B ta cũng phải sử dụng talet tính tỷ số độ dài EB EF (tương đối phức tạp) • Phương pháp trên có thể sử dụng cho bất cứ hình nào miễn sao hình đó đặt vào hệ trục ta có thể xác định được tọa độ các đỉnh. Cụ thể như: hình vuông, hình chữ nhật có tỷ số độ dài, hình thoi có tỷ số hai đường chéo, tam giác vuông có tỷ số hai cạnh góc vuông, tam giác đều…. Câu 8 (1,0 điểm). Giải Bất phương trình sau: ( )2 2 2 3 3 8 3 2 1 0 1 3 4 5 x x x x x x + + + − + ≤ − − + Bài giải : Ta có : 2 2 211 11 3 4 5 3 4 5 1 1 3 4 5 0 4 3 3 x x x x x x a ∆ − + ≥ − = ⇔ − + ≥ > ⇔ − − + < ( )2 2 ( ) 3 3 8 3 2 1 0 (*)bpt f x x x x x⇔ = + + + − + ≥ Bình luận :Với BPT có dạng: ( ) ( ) ( ) 0f x g x h x+ ≥ thì thông thường ta nghĩ đến các hương biến đổi sau: Hướng 1 : Sử dụng đồng nhất thức : ( ) 222 2 3 3 8 3 2 1x x m x n x+ += − + + để chuyển về thuần nhất Hướng 2 :Chuyển về dạng phương trình bậc hai theo nhóm biến 2 2 1x + . Hướng 3 : Thêm bới để nhân liên hợp rồi chuyển về dạng tích Hướng 4 :Phân tích về dạng ( ) ( )( ) ( )f u x f v x= sau đó xét hàm dặt trưng ( )f t Bài này ta có thể đi theo hướng 2: Ta sẽ tìm lượng thêm bớt để đưa về phương trình bậc 2 như sau : ( ) 2 2 2 2 ( ) 2 1 8 3 2 1 (2 3) 3 0f x m x x x m x x m= + + − + − − + + −= ( ) ( ) ( )2 2 2 2 2 8 3 4 (2 3) 3 8 12 64 2(2 24) 4 12 9x m m x x m m m x m x m m∆ = − − − + + − = − + + − + + − + Xét 0∆ = có nghiệm kép (để ∆ có thể đưa về dạng hằng đẳng thức)
  • 32. ( )( )2 2 2 ' (2 24) 8 12 64 4 12 9 0m m m m m∆∆= + − − + + − + = bấm máy tính ta có nghiệm nguyên 3m = Xét 2 2 2 2 ( ) 0 3 2 1 (8 3) 2 1 3 0f x x x x x x= ⇔ + + − + − + = ( ) 22 2 (8 3) 4.3( 3 ) 10 3x x x x∆= − − − + = − ( ) ( ) 2 2 (8 3) 10 3 2 1 2 1 6 ( ) 0 (8 3) 10 3 2 1 6 3 x x x x f x x x x x − − − − + = =− + = ⇔ − − + − += =  ( )2 2 (*) 2 1 2 1 2 1 0 3 x x x x   ⇔ + + − + − ≥    Dễ thấy : 2 2 2 2 271 2 1 1 2 1 0 , 36 36 36 6 6 6 x x x x x x x x x R+ = + + > = ≥ ⇔ + − > ∀ ∈ 2 2 2 2 1 2 0 2 1 0 (*) 2 1 2 1 0 2 1 1 2 0 1 2 0 2 4 0 x x x x x x x x x x  − <  + ≥ ⇔ + + − ≥ ⇔ + ≥ − ⇔ ⇔ ≥ − ≥  − ≤ Vậy bất phương trình có tập nghiệm là : [ )0 ,S= + ∞ Câu 9 (1,0 điểm).Tìm giá trị lớn nhất, nhỏ nhất của hàm số sau : 5 4 1 5 4 2 1 6 x x y x x − − + = − + + + Bài giải : Nhận thấy : ( ) ( ) 2 2 5 4 2 1 9x x− + + = Đăt : 5 4 3sin 0 2 1 3cos 0 x u x u  − = ≥  += ≥ ĐK : 0 , 2 u π  ∈   (vì sin 0u ≥ ,cos 0u ≥ ) 2sin cos 2sin 2cos 4 u u y u u − = + + . Đặt : 2 2 tan sin 2 1 u t t u t = ⇒ = + và 2 2 1 cos 1 t u t − = + 2 22 2 2 2 2 2 2 1 2 4 11 1( ) 2 1 6 4 2 2 2 4 1 1 t t t tt ty f t t t t t t t − − + −+ += = = − + + − + + + , [ ]0 , 0 , t 0 , 1 2 2 4 u u π π    ∀ ∈ ⇒ ∈ ⇒ ∈       ( ) 2 22 20 16 12 4 31 '( ) 0 56 4 2 t t f t t t t − + + ± = = ⇔ = + +
  • 33. BBT : Từ BBT ta có : [ ] [ ] [ ] [ ] 0,1 0,1 1 in ( ) 0 2 1 ( ) 1 3 M f t khi t Max f t khi t  =− =   = =  Vậy ( ) 1 2 Min y = − khi 5 5 4 3sin 0 0 4 x x− = = ⇔ = ( ) 1 3 Max y = khi 2 1 3cos 0 1 2 x x π + = = ⇔ =− ĐỀ SỐ 4 Câu 1 (2,0 điểm). Cho hàm số : 3 2 3 2y x mx= − + (với m là tham số) 1. Khảo sát sự biến thiên và vẽ đồ thị hàm số trên khi m = 1. 2. Tìm m để đường thẳng đi qua hai điểm cực trị của đồ thị hàm số trên tạo với các trục tọa độ thành một tam giác có diện tích bằng 4 Câu 2 (1,0 điểm). 1. Giải phương trình : sin 4 2 cos3 4sin cosx x x x+= + + 2. Giải hệ phương trình : ( )( ) ( )2 2 2 2 2 2 11 17 2 2 6 log 1 1 1 log y x y x x x y x x y y  + + + + − = +   − + + −= +   Câu 3 (1,0 điểm). 1. Tính tích phân : 2 2 3 1 (1 )ln e x x x I dx x + + = ∫ 2. Tính thể tích khối tròn xoay tạo thành khi quay hình phẳng được giới hạn bởi các đường 1 , 0, 0 1 4 3 y y x x = = = + − và 1x = xung quanh trục hoành. Câu 4 (1,0 điểm). 1. Cho số phức z thỏa mãn 2 3z z i+ = + . Tính tổng: 2014 2015 2016 T z z z= + + 2. Gọi E là tập hợp các số tự nhiên có ba chữ số sao cho số đứng sau lớn hơn số đứng trước. Lấy ngẫu nhiên từ tập E ra một số , Tính xác suất để số được chọn là một số chẵn. Câu 5 (1,0 điểm). Cho hình lăng trụ tam giác ABCA’B’C’ có AB = a, BC = 2a,  0 30ACB = , hình 4 31 5 − 4 31 5 + 0 0 0 1 + + ++- - t '( )f t ( )f t 1 2 − 1 3
  • 34. chiếu vuông góc của A’ lên mặt phẳng (ABC) trùng với trọng tâm G của tam giác ABC và góc giữa AA’ tạo với mặt phẳng (ABC) bằng 0 60 . Tính theo a thể tích của khối đa diện BCC’B’A’ và khoảng cách giữa hai đường thẳng B’C’ và A’C. Câu 6( 1,0 điểm): Trong không gian với hệ trục tọa độ Oxyz, cho các điểm A(1; 1; -1), B(1; 1; 2), C(-1; 2; -2) và mặt phẳng (P) có phương trình x – 2y + 2z +1 = 0. Viết phương trình mặt phẳng đi qua A , vuông góc mặt phẳng (P) và cắt đoạn thẳng BC tại điểm I sao cho IB = 2IC Câu 7 (1,0 điểm). Trong mặt phẳng với hệ trục tọa độ Oxy, cho tam giác ABC có trực tâm H(5; 5) và đường thẳng BC : x + y – 8 = 0. Đường tròn ngoại tiếp tam giác ABC đi qua điểm ( )1; 5D . Tìm tọa độ các đỉnh của tam giác ABC, biết rằng độ dài cạnh BC bằng 4 2 và hoành độ của điểm B lớn hơn 3. Câu 8 (1,0 điểm).Giải hệ phương trình: ( )( ) ( )3 2 2 2 3 1 4 1 0 1 2 5 4 1 4 x y y x y x x x x x y x x  − + + + + + + =    + + += + + +  Câu 9 (1,0 điểm). Tìm giá trị nhỏ nhất của biểu thức : 2 2 2 2 2 20 9 42 2 1 4 2 1 3 (3 4 ) 5 y y P x y x x y x x x y − + = + − + + + + + + − + --- Hết --- GIẢI ĐỀ SỐ 4 Câu 1 (2,0 điểm). Cho hàm số : 3 2 3 2y x mx= − + (với m là tham số) Tìm m để đường thẳng đi qua hai điểm cực trị của đồ thị hàm số trên tạo với các trục tọa độ thành một tam giác có diện tích bằng 4. Bài giải: Ta có: ( )2 3 0 2 ' 3 6 0 3 2 0 2 2 4 x y y x mx x x m x m y m = ⇒ = = − =⇔ − =⇔  = ⇒ = − Hàm số có hai cực trị ⇔ phương trình ' 0y = phải có hai nghiệm phân biệt 2 0 0m m⇔ ≠ ⇔ ≠ (*) Khi đó hai điểm cực trị của đồ thị là : ( )0; 2A và ( )3 2 ; 2 4B m m− ( )3 2 ; 4AB m m= −  là VTCP của AB 2 3 0 2 : : 2 2 0 2 4 x y AB AB m x y m m − − ⇒ = ⇔ + −= − (vì 0m ≠ ) Gọi 2 2 1 1 ; 0M AB Ox M OM m m   = ∩ ⇒ ⇒ =    Ta có: ( )0 ; 2 2A AB Oy OA= ∩ ⇒ = 2 1 1 1 4 . 4 4 2 2 AMOgt S OAOM m m ⇒ = ⇔ = ⇔ = ⇔ =± thỏa điều kiện(*)
  • 35. Vậy 1 2 m = ± thỏa yêu cầu bài toán. Câu 2 (1,0 điểm). 1. Giải phương trình : sin 4 2 cos3 4sin cosx x x x+= + + Bình luận: Với phương trình lượng giác chứa hai số hạng cùng hệ số, cùng loại hàm, cùng tính chẵn hoặc lẽ của cung (hai số hạng được gạch dưới) thì ta sử dụng công thức tổng thành tích . Sau đó Tìm mọi cách để biến đổi các số hạng còn lại để xuất hiện nhóm chung với biểu thức tích ở trên .cuối cùng ta chuyển phương trình về dạng tích. Bài giải: ( ) ( ) ( ) 4sin cos cos2 2cos2 cos 4sin 2 0 cos cos2 2sin 1 2sin 1 0 2sin 1 0 (1) 2sin 1 (cos cos2 1) 0 cos cos2 1 0 (2) pt x x x x x x x x x x x x x x x x ⇔ − − + = ⇔ − − − = − = ⇔ − − = ⇔  − = 1 5 (1) sin 2 2 2 6 6 x x k x k π π π π⇔ = ⇔ = + ∨ = + ( )( )3 2 (2) 2cos cos 1 0 cos 1 2cos 2cos 1 0 cos 1 2x x x x x x k π⇔ − − = ⇔ − + + = ⇔ = ⇔ = Vậy phương trình có ba họ nghiệm 5 2 2 2 6 6 x k x k x k π π π π π= + ∨ = + ∨ = 2. Giải hệ phương trình : ( )( ) ( )2 2 2 2 2 2 11 17 2 2 6 (1) log 1 1 1 log (2) y x y x x x y x x y y  + + + + − = +   − + + −= +   Bài giải: Điều kiện : ( )( )2 2 1 1 0 1 00 2 11 17 02 11 17 0 2 2 02 2 0 x y x yx y y xy x y xy x  − + >  >   > > ⇔  + + ≥+ + ≥    + − ≥+ − ≥  (*) ( ) ( )2 2 2 2 2 2 2(2) log 1 log 1 log log 1x x x y y y⇔ − − + −= − + + ( ) 2 2 2 22 ( 1) log 1 log (3) ( 1) 1 1 x y x y x y  − ⇔ + −= +  − + +  Xét hàm số: ( )2( ) log 0 , 1 t f t t t t = + ∀ + ∞ + Ta có : ( ) 1 '( ) 1 0 0 , ( 1)ln 2 f t t t t = + > ∀ ∈ + ∞ ⇒ + ( )f t đơn điệu tăng trên ( )0 , + ∞
  • 36. Do đó 2 2 (2) ( ) ( 1) 1f y f x y x⇔ = − ⇔ = − Thay 2 1y x= − vào phương trình (1) ta có: 2 2 2 11 15 2 3 6 (3)x x x x x+ + + + − = + Dễ thấy x = - 3 không phải là nghiệm của phương trình Xét 3x ≠ − ta có: ( )( ) ( )( )2 2 (3) 3 2 11 15 2 3 3 6x x x x x x x⇔ + + + + + − = + + ( )( ) 2 2 2 2 2 2 3 2 11 15 2 3 2 11 15 2 3 0x x x x x x x x x ⇔ + + + + + − − + + − + − =    2 2 2 2 2 11 15 2 3 0 3( ) 2 11 15 2 3 3 (4) x x x x x loai x x x x x  + + + + − = ⇔ =− ⇔  + + − + − = +  Lây (3) trừ (4) vế theo vế ta có phương trình hệ quả : 2 2 7 3 2 2 3 3 4 8 21 0 2 2 x x x x x x+ − =⇔ + − =⇔ =− ∨ = Thay 7 2 x = − và 3 2 x = vào (3) đều thỏa mãn . Với 7 45 2 4 x y=− ⇒ = Với 3 5 2 4 x y= ⇒ = Vậy hệ phương trình có hai nghiệm: ( ) 7 45 ; ; 4 4 x y   = −    và ( ) 3 5 ; ; 4 4 x y   =     Bình luận:  Ở cấu trúc pt(2) ta thấy vừa chứa biến trong lôga vừa chứa biến ở dạng đa thức (tức là có hai dạng hàm khác nhau cùng xuất hiện trong một phương trình). Vì thế để giải được (2) thì nhất định ta phải dùng phương pháp hàm số hoặc phương pháp đánh giá (không sử dụng biến đổi đại số thông thường được). Lại thấy (2) có hai biến có thể cô lập ở hai vế khác nhau được nên ta nghĩ ngay đến dạng toán sử dụng tính đơn điệu với cấu trúc ( ) ( )f u f v= (với u, v là các biểu thức theo x, y)  Ở pt(3) ta nhận thấy : 2 2 2 2 2 2 11 15 2 3 9 18 ( 3)( 6)x x x x x x x x+ + − + − = + + = + + nên ta nghĩ ngay đến phép nhân liên hợp. Ngoài ra pt(3) còn có thể giải cách khác: ( )2 2 2 11 15 ( ) 2 3 6 0x x x m x x m   + + − + + + − − − =     xét ĐNT: 2 2 2 2 (11 2 ) 15 2 39 12 ,x m x m x x m m x R+ − + − = + − + − ∀ ∈ ⇒ 9 2 m = sau đó ta nhân liên hợp
  • 37. Hoặc : Đặt ( ) 2 2 2 2 2 11 15 3 ( 6) 2 3 u x x u v x x v x x  = + + ⇒ − = + + = + − khi đó 6 (3) 3 u v x u v x + = + ⇒  − = + Câu 3 (1,0 điểm). 1. Tính tích phân : 2 2 3 1 (1 )ln e x x x I dx x + + = ∫ Bài giải: 2 2 1 2 33 3 1 1 1 1 ln ln ln ln 1 e e e e x x x x x x I dx dx dx dx I I I x x x x + + = = + + = + +∫ ∫ ∫ ∫ Giải 1I : 3 2 1 ln 1 1 2 u x du dx x dv dx v x x  = ⇒ =   = ⇒ =−  1 2 3 2 2 2 2 2 1 1 1 1 1 1 1 1 1 1 1 3 ln 1 12 2 2 4 2 4 4 4 4 e e e I x dx x x e x e e e =− + =− − =− − + =−∫ 2 2 1 ln 1 ln (ln ) 12 2 e ex I x d x= = =∫ 3 ln 1 1 e I x= = Vậy 2 7 3 4 4 I e = − 2. Tính thể tích khối tròn xoay tạo thành khi quay hình phẳng được giới hạn bởi các đường 1 , 0, 0 1 4 3 y y x x = = = + − và 1x = xung quanh trục hoành. Bài giải: Thể tích của khối tròn xoay theo đề được tính theo công thức : 21 0 1 1 4 3 V dx x π   =   + −  ∫ Đặt: 2 2 1 4 3 4 3 ( 1) 3 2( 1) ( 1) 3 t x x t dx t dt dx t dt= + − ⇔ − = − ⇒ − = − ⇔ = − − Đổi cận: 1 2x t= ⇒ = 0 3x t= ⇒ = 2 3 2 2 3 2 32 1 2 1 1 2 1 2 1 ln ln3 ln 2 23 3 3 3 6 t V dt dt t t t t t π π π π−       =− = − = + = − −            ∫ ∫ (đvtt) Câu 4 (1,0 điểm).
  • 38. 1. Cho số phức z thỏa mãn 2 3z z i+ = + . Tính tổng: 2014 2015 2016 T z z z= + + Bài giải: Gọi ( ),z x yi x y R=+ ∈ ( ) 1 2 3 2 3 3 3 1 1 x z z i x yi x yi i x yi i z i y = + = + ⇔ + + − = + ⇔ − = + ⇔ ⇒ = − = − Ta có : ( ) 3 2 24 1 1 1 i i i i  = − =− ⇔  =− = ( ) ( ) ( ) 10071007 25122014 2 1007 1007 1007 4 3 1007 1 2 2 2z z i i i i i = =− =− =− =   ( ) ( )2014 2 1007 1007 1008 1 2 1 1 2 3.2 2T z z z i i i i= + += + − −= + 2. Gọi E là tập hợp các số tự nhiên có ba chữ số sao cho số đứng sau lớn hơn số đứng trước. Lấy ngẫu nhiên từ tập E ra một số , Tính xác suất để số được chọn là một số chẵn. Bài giải:Gọi 1 2 3x x x x= với 1 2 3x x x< < Từ đó ta thấy trong số x không có chữ số 0 và cứ mỗi cách chọn ra một bộ 3 chữ số khác nhau từ tập hợp X = {1, 2, 3, 4, 5, 6, 7, 8, 9} thì ta chỉ thiết lập được duy nhất một số x Số phần tử của tập E là số cách chọn 3 chữ số từ tập X ⇒ số phần tử của tập E là: 3 9 84E C= = Gọi Y là tập con của E và chỉ chứa các số chẵn : Để tính số phần tử của tập Y ta chia thành 3 trường hợp sau (vì 3 2x ≠ ) TH1: 3 4x ≡ khi đó 1 2,x x chỉ được chọn từ {1,2,3} ⇒ có 2 3C số x có chữ số cuối là 4. TH2: 3 6x ≡ khi đó 1 2,x x chỉ được chọn từ {1,2,3,4,5} ⇒ có 2 5C số x có chữ số cuối là 6. TH3: 3 8x ≡ khi đó 1 2,x x chỉ được chọn từ {1,2,3,4,5,6,7} ⇒ có 2 7C số x có chữ số cuối là 8. Theo quy tắc cộng ta có số phần tử của tập X là: 2 2 2 3 5 7 34X C C C= + + = Gọi Ω là không gian mẫu của phép thử lấy một số từ tập E ⇒ số phần tử củaΩ là 84Ω = Gọi A là biến cố lấy ra được số chẵn (tức là lấy một số từ Y)⇒số phần tử của AΩ là 34AΩ = Vậy xác suất của biến cố A là 34 17 ( ) 84 42 A P A Ω = = = Ω Câu 5 (1,0 điểm). Cho hình lăng trụ tam giác ABCA’B’C’ có AB = a, BC = 2a,  0 30ACB = , hình chiếu vuông góc của A’ lên mặt phẳng (ABC) trùng với trọng tâm G của tam giác ABC và góc giữa AA’ tạo với mặt phẳng (ABC) bằng 0 60 . Tính theo a thể tích của khối đa diện BCC’B’A’ và khoảng cách giữa hai đường thẳng B’C’ và A’C.
  • 39. Bài giải: Gọi M là trung điểm BC.Đặt : AC x= Ta có AG là hình chiếu của AA’ lên (ABC) ⇒ ( ) ( ) ',( ) ', 'AA ABC AA AG A AG= =  0 ' 60gt A AG⇒ = Theo định lý cosin trong ABC∆ ta có:  ( ) ( ) 2 2 2 22 2 0 2 2 2 . .cos 2 2 . 2 .cos30 2 3 3 0 3 3 AB CA CB CACB ACB a x a x a x ax a x a AC a = + − ⇔ = + − ⇔ − + = ⇔ = ⇒ = Ta có: 2 2 2 2 2 2 3 4AB AC a a a BC ABC+ = + = = ⇒ ∆ vuông tại A. 2 2 2 3 3 2 3 BC a AG AM⇒ = = = Xét 'AA G∆ ta có:  ' 2 3 tan ' ' tan ' 3 A G a A AG A G AG A AG AG = ⇔ = = Thể tích khối lăng trụ ABCA’B’C’ là 31 1 2 3 ' . ' . . . 3 2 2 3 ABC a V A G S A G AB AC a a V a= = = ⇔= (đvtt) Ta có B’C’ // BC ⇒ B’C’ // (A’BC) ( ) ( ) ' ' ' 3 ' ', ' ',( ' ) B A BC A BC V d B C A C d B A BC S ⇒ = = Lại có: 3 ' ' ' ' ' ' ' ' ' ' 1 1 1 3 3 3 3 B A BC A ABC CA B C B A BC B A BC a V V V V V V V V V= + + = + + ⇔ = = Theo Pita go ta có: 2 22 7 3 2 3 AC a GB AB   = + =    2 22 13 3 2 3 AB a GC AC   = + =    2 2 19 ' ' 3 a A B GA GB= + = 2 2 5 ' ' 3 a A C GA GC= + = Theo định lý cosin:    2 2 2 2' ' 7 51 ' sin ' 1 cos ' 2 '. 10 10 CA CB A B CosBCA BCA BCA CA CB + − = =⇒ =− =  2 ' 1 51 . 'sin ' 2 6 A BC a S CB CA BCA= = ( ) 3 ' ' 2 ' 3 6 3 51 ' ', ' 1751 B A BC A BC V a a d B C A C S a = = = (đvđd) A A’ C B B’ C’ 0 30 0 60 a 2a G M
  • 40. Câu 6( 1,0 điểm): Trong không gian với hệ trục tọa độ Oxyz, cho các điểm A(1; 1; -1), B(1; 1; 2), C(-1; 2; -2) và mặt phẳng (P) có phương trình x – 2y + 2z +1 = 0. Viết phương trình mặt phẳng đi qua A , vuông góc mặt phẳng (P) và cắt đoạn thẳng BC tại điểm I sao cho IB = 2IC Bài giải : Gọi (Q) là mặt phẳng cần tìm Ta có : điểm I thuộc đoạn thẳng BC sao cho IB = 2IC 2 2IB IC IC BC IC⇒ =− ⇔ + =−      [ ] ( ) [ ] 1 ( 1) 3 ( 1) 1 5 2 4 2 1 3 1 2 3 2 ; ; ; ; 3 3 3 3 3 3 2 ( 2) 3 ( 2) I I I x CB CI y I IA z  − −= − −      ⇔ = ⇔ − = − ⇒ − − ⇒ = − −         − −= − −    (P) :x – 2y + 2z +1 = 0 ( )P⇒ có VTPT là ( )1; 2;1Pn= −  Mặt phẳng (Q) qua A, I và vuông góc với (P) 4 5 6 , ; ; 3 3 3 Pn n AB   ⇒= =         là VTPT của(Q) Do đó mặt phẳng (Q) qua A và có VTPT là n  nên: ( ): 4 5 6 3 0Q x y z+ + − = Vậy mặt phẳng cần tìm có phương trình là :4 5 6 3 0x y z+ + − = Câu 7 (1,0 điểm). Trong mặt phẳng với hệ trục tọa độ Oxy, cho tam giác ABC có trực tâm H(5; 5) và đường thẳng BC có phương trình x + y – 8 = 0. Đường tròn ngoại tiếp tam giác ABC đi qua điểm ( )1; 5D . Tìm tọa độ các đỉnh của tam giác ABC, biết rằng độ dài cạnh BC bằng 4 2 và hoành độ của điểm B lớn hơn 3. Bài giải : Gọi đường tròn ngoại tiếp ∆ABClà (C) có tâm I (a; b) và bán kính R Gọi E là giao điểm khác A của AH và (C) Ta có :  EBC EAC= (cùng chắn cung EC ) mà  HBC EAC= (cùng phụ với góc ACB ) BC⇒ là đường phân giác của HBE ⇒K, H đối xứng nhau qua BC. AH qua H và vuông góc BC : 0AH x y⇒ − = Gọi K là giao điểm của AH và BC ( )(4;4) 3;3K E⇒ ⇒ . A B C H(5; 5) I D(1; 5) E K 8 0x y+ − = 4 2BC =
  • 41. Ta có: ( ) ( ) 2 22 2 3 ( 3) 1 ( 5) 2 ( ; 2)IE ID a b a b b a I a a= ⇔ − + − = − + − ⇔ = + ⇒ + Theo Pitago ta có: [ ] 2 22 2 ( , ) 2 BC IE R d I BC   = = +     ( ) 22 2 2 2 2 (4; 6)2 8 4 2 3 ( 1) 4 2 101 1 Ia a a a a R IE    + + −   ⇔ − + − = + ⇔ = ⇒      = =+    ( ) 2 2 ( ) : 4 ( 6) 10C x y⇒ − + − = A là giao điểm khác E của AH và (C)⇒A(7; 7) B, C là hai giao điểm của BC và (C) ⇒tọa độ B,C là nghiệm của hệ: 2 8 5 1 3 710 21 0 x y x x y yy y = − = =   ⇔ ∨   = =− + =   mà hoành độ điểm B lớn hơn 3 nên: (5;3)B và (1;7)C Vậy tam giác ABC có ba đỉnh là A(7; 7) , (5;3)B và (1;7)C Bình luận: • Với những bài toán có yếu tố đường thẳng cắt đường tròn thì nhất định ta phải sử dụng công thức bitago: [ ] 2 22 ( , ) 2 BC R d I BC   = +     để viết phương trình đường thẳng hoặc đường tròn • Với bài toán cho trực tâm, đường tròn ngoại tiếp tam giác + Nếu đề cho phương trình một cạnh thì ta phải đối xứng trục tâm qua cạnh đó để có thêm điểm thuộc đường tròn ngoại tiếp . + Nếu đề cho trung điểmM của cạnh BC thì ta dựng dường kính AA’ sau đó chứng minh BHCA’ là hình bình hành , suy ra M là trung điểm của HA’ Câu 8 (1,0 điểm).Giải hệ phương trình: ( )( ) ( )3 2 2 2 3 1 4 1 0 (1) 1 2 5 4 1 4 (2) x y y x y x x x x x y x x  − + + + + + + =    + + += + + +  Bài giải : Điều kiện: 2 3 0 4 0 1 0 y x y x x + ≥  + + ≥  ≤ − ∨ ≥ (*) ( ) ( )(1) 1 2 3 4 2 3 1 0x y y x y y x y⇔ − + + + + + − + + − + =
  • 42. ( ) 1 1 2 3 1 0 1 0 1 4 2 3 x y y x y y x x y y   ⇔ − + + + + = ⇔ − + = ⇔ = +  + + + +  Vì 1 2 3 1 0 4 2 3 y x y y + + + > + + + + với mọi ,x y thỏa ĐK (*) Thay 1y x= + vào (2) ta có: ( )3 2 2 1 2 5 4 1 2 5x x x x x x x + + += + + ( ) ( )2 2 2 21 1 2 5 4 2 5 2 5 5 4 0 (3)x x x x t x t x x x x ⇔ + + + = + + ⇔ − + + + + = với 2 1 t x x = + Xem (3) là phương trình bật hai theo biến t và xem x như là một tham số ta có: ( ) ( )2 2 2 5 4 5 4 9t x x x∆= + − + + = 2 5 3 1 (4) 2 (3) 2 5 3 4 (5) 2 x t x x t x + − = = + ⇔  + + = = +  2 2 2 2 1 1 0 11 (4) 1 1 1 3 2 1 2 1 0 2 2 x x y x x x x x x x x yx x x ≥ − =− ⇒ = ≥ − ⇔ + = + ⇔ ⇔ ⇔  + = + + = ⇒ =+ − =  2 2 2 2 3 2 3 2 4 111 4 4(5) 4 1 8 16 8 16 1 0 3 2 3 2 1 4 4 x x yx x x x x x x x x x x y  ≥ − =− − ⇒ =− ≥ − ⇔ + = + ⇔ ⇔ ⇔  + = + + + − = =− + ⇒ =  Vậy hệ phương trình có 4 nghiệm: 1 0 x y = −  = , 1 2 3 2 x y  =   =  , 3 2 1 4 3 2 4 x y  =− −    = − và 3 2 1 4 3 2 4 x y  =− +    = Bình luận: • Để có thể thêm bớt được hợp lý trong cách giải của phương trình (1) thì trước tiên ta phải dự đoán được nhóm chung sẽ xuất hiên sau khi nhân liên hợp là gi. Ta sẽ tìm nó bằng cách sau: Xét 1x = thì ( )( )(1) 1 2 3 1 5 1 0 2y y y y⇔ − + + + + + = ⇒ =
  • 43. Xét 2x = thì ( )( )(1) 2 2 3 1 6 1 0 3y y y y⇒ − + + + + + = ⇒ = Từ đó nhóm chung ax by c+ + có hai nghiệm ( )1, 2 và ( )2, 3 2 0 2 3 0 a b c a c a b c b c + += =  ⇒ ⇔  + + = =−  vậy nhóm chung là 1cx cy c x y− + = − + • Ngoài ra phương trình (1) cũng có thể giải bằng phép đặt ẩn phụ như sau: 2 2 2 3 0 1 4 0 u y x y v u v x y = + ≥ ⇒ − = − − = + + ≥ . ( )( ) ( )( ) ( )2 2 2 2 (1) 1 1 1 0 1 0v u u v v u u v u⇔ − − + + + = ⇔ − + + − = ( ) ( )( )1 1 0 4 2 3 1v u u v u v u x y y y x⇔ − + + + = ⇔ = ⇔ + + = + ⇔ = +   • Ở phương trình (3) ta thấy có nhóm 2 1 x x + là nổi bật nhất . Rỏ ràng nhóm này ta không thể phân tích được (sai lầm phổ biến: 3 3 2 1 1 1x x x x x x + + += = khi đó điều kiện bị thay đổi ) Vì thể nên ta phải nghĩ đến việc biến đổi các số hạng còn lại để có được cấu trúc đồng dạng với 2 1 x x + sau đó đặt ẩn phụ .Nếu may mắn thì ta sẽ chuyển tất cả qua ẩn phụ được còn không ta phải đối diện với phương trình hai biến x và t khi đó chỉ còn một hướng để đi là phải chuyển được về phương trình tich. Câu 9 (1,0 điểm). Tìm giá trị nhỏ nhất của biểu thức : 2 2 2 2 2 20 9 42 2 1 4 2 1 3 (3 4 ) 5 y y P x y x x y x x x y − + = + − + + + + + + − + Bài giải : ( ) ( ) ( ) ( ) 2 2 2 22 9 42 1 1 2 3 2 5 5 P x y x y x y y= − + + − − + + − − + Xét các vectơ: ( ) ( ) ( )1; , 1; 2 2; 3u x y v x y u v y= − = − − ⇒ + = −     Ta có: ( ) ( ) ( ) 2 2 22 2 1 1 2 4 9u v u v x y x y y+ ≥ + ⇔ − + + − − + ≥ +     Khi đó ta có: 2 9 42 4 9 5 5 P y y≥ + − + (vì ( ) 2 3 2 0x y− ≥ )
  • 44. Xét hàm sô: 2 9 42 ( ) 4 9 5 5 f y y y= + − + Ta có: ( )2 2 2 9 5 4 99 9 '( ) 54 9 5 4 9 y yy f y y y − + = −= + + 1 '( ) 0 2 f y y= ⇔ = ( lại có '(0) 0f < và '(1) 0f > ) BBT: Từ BBT ta thấy: ( ) 10Min y = khi 1 2 y = Suy ra 10MinP = khi 1 3 x = và 1 2 y = --- Hết --- ĐỀ SỐ 5 Câu 1 (2,0 điểm). Cho hàm số : 3 23 6 10 2 y x x x= − − + có đồ thị (C) 1. Khảo sát sự biến thiên và vẽ đồ thị (C) của hàm số trên 2. Sử dụng đồ thị (C) của hàm số trên để biện luận theo m số nghiệm của phương trình: ( )( ) 10 2 10 2 2 1 x m x x x + − − + = Câu 2 (1,0 điểm). 1. Giải phương trình: 1 2tan cot 2 3 sin 2 x x x + = + 2. Giải bất phương trình: 2 3 2 3 2 log ( 1) log ( 1) 0 3 4 x x x x + − + > − − Câu 3 (1,0 điểm). Tính tích phân: ln2 0 1 1x I dx e = − ∫ Câu 4(1,0 điểm): 1. Cho số phức z thỏa mãn điều kiện: ( )1 2 2(1 2 )z i z i+ − = − . Tìm phần thực và phần ảo của số phức 2 3w z z= − 2. Cho tập E = {1; 2; 3; 4; 5}. Viết ngẫu nhiên lên bảng hai số tự nhiên , mỗi số gồm ba chữ số đôi một khác nhau thuộc tập E . Tính xác suất để trong hai số đó có đúng một số chẵn. y '( )f y ( )f y 1 2 0- + 10
  • 45. Câu 5(1,0 điểm): Cho hình chóp SABCD có đáy ABCD là hình chữ nhật có AB =a và SA vuông góc đáy . đường thẳng SC tạo với mặt phẳng (ABCD) một góc 300 . Gọi M là hình chiếu của A lên đường thẳng SB. Biết 3 3 a AM = . Tính theo a thể tích của khối chóp SABCD và khoảng cách từ điểm M đến mặt phẳng (SCD). Câu 6(1,0 điểm):Trong không gian với hệ trục Oxyz, cho mặt phẳng (P): 2x + 2y + z – 5 = 0 và đường thẳng 2 2 3 : 1 1 2 x y z d − − − = = . Gọi ∆ là đường thẳng vừa cắt vừa vuông góc với d. Viết phương trình đường thẳng ∆ . Biết khoảng cách từ ∆ đến mặt phẳng (P) bằng 10. Câu 7(1,0 điểm): Trong mặt phẳng với hệ trục tọa độ Oxy, cho hình thang ABCD với cạnh đáy AB và CD = 2AB. Gọi H là chân đường vuông góc hạ từ D xuống AC và M là trung điểm của HC. Biết tọa độ đỉnh B(5; 6), phương trình đường thẳng DH: 2x – y = 0, phương trình đường thẳng DM: x – 3y + 5 = 0, tìm tọa độ các đỉnh của hình thang ABCD. Câu 8(1,0 điểm): Giải hệ phương trình : 2 2 4 ( 5) 1 4 2 2 4 ( 4) 2 1 x y x y x y y x x x  + − −= − +  − + = − Câu 9(1,0 điểm): Cho x, y, z là các số thực dương thỏa mãn xyz + x + z = y . Tìm giá trị lớn nhất của biểu thức ( )2 2 2 2 2 2 2 4 3 1 1 1 1 1 z z P x y z z z = − − + + + + + + --- Hết --- GIẢI ĐỀ 5 Câu 1 (2,0 điểm). Cho hàm số : 3 23 6 10 2 y x x x= − − + có đồ thị (C) 1. Khảo sát sự biến thiên và vẽ đồ thị (C) của hàm số trên Bài giải: • Tập xác định : D = R • Sự biến thiên: 2 ' 3 3 6y x x= − − ; ' 0 1y x= ⇔ =− hoặc 2x = Chiều biến thiên: Hàm số đồng biến trên các khoảng :( ); 1−∞ − và ( )2 ; + ∞ Hàm số ngịch biến trên khoảng :( )1; 2−
  • 46. Cực trị : Hàm số đạt cực đại tại x = -1 với giá trị cực đại là 27 2 CDy = Hàm số đạt cực tiểu tại x = 2 với giá trị cực tiểu là 0CTy = Giới hạn tại vô cực : lim x y →−∞ = −∞ ; lim x y →+∞ = +∞ Bảng biến thiên: • Đồ thị : '' 6 3y x= − ; 1 '' 0 2 y x= ⇔ = . Đồ thị có điểm uống là 1 27 ; 2 4 I       Đồ thị có hai điểm chung với Ox là 5 ; 0 2   −    và ( )2; 0 và có điểm chung với Oy là ( )0;10 2. Sử dụng đồ thị (C) của hàm số trên để biện luận theo m số nghiệm của phương trình: ( )( ) 10 2 10 2 2 1 (*) x m x x x + − − + = Bài giải: Điều kiện: 0x ≠ Khi đó ta có: ( ) 2 10 2 10 * 2 3 2 x m x x x + − ⇔ − − = 3 2 2 3 12 10 2x x x m⇔ − − + = 3 23 6 10 5 (1) 2 x x x m⇔ − − + = + −∞ +∞-1 2 0 0+ - + −∞ +∞27 2 0 x 'y y 0-15 2 − 2 .. . . y x 27 2
  • 47. Suy ra : (1) là phương trình hoành độ giao điểm của đồ thị (C) và đường thẳng d: y = m + 5 (luôn song song với trục Ox). Nên số nghiệm của (1) là số giao điểm của đường thẳng d và đồ thị (C). Từ đồ thị (C) ta có: • Với 5m < − (tức là: 5 0m + < ) thì d và (C) có một điểm chung duy nhất nên (1) có nghiệm duy nhất • Với m = - 5 (tức là : m + 5 = 0) thì d và (C) có hai điểm chung nên (1) có hai nghiệm • Với 17 0 2 m< < (tức là: 27 0 5 2 m< + < ) thì d và (C) có 3 điểm chung nên (1) có ba nghiệm. • Với 17 2 m = ( tức là: 27 5 2 m + = ) thì d và (C) có 2 điểm chung nên (1) có hai nghiệm. • Với 17 2 m > ( tức là: 27 5 2 m + > ) thì d và (C) có 1 điểm chung duy nhất, nên (1) có một nghiệmduy nhất. Mặt khác (1) có nghiệm x = 0 ⇔ m + 5 = 10 ⇔ m = 5 Kết luận: Khi 5 17 2 m m < −   >  thì phương trình (*) có một nghiệm duy nhất Khi 5 17 2 5 m m m = −   =   = thì phương trình (*) có hai nghiệm phân biệt. Khi 17 5 2 5 m m  − < <   ≠ thif phương trình (*) có ba nghiệm phân biệt Câu 2 (1,0 điểm). 1. Giải phương trình: 1 2tan cot 2 3 sin 2 x x x + = + Bài giải: Điều kiện: sin 2 0 ( ) (*) 2 x x k k Z π ≠ ⇔ ≠ ∈ 0-15 2 − 2 .. . . y x 27 2 10
  • 48. Ta có: sin cos2 cos2 cos sin 2 sin cos 1 tan cot 2 cos sin 2 cos sin 2 cos sin 2 sin 2 x x x x x x x x x x x x x x x x + + = + = = = 1 tan tan cot 2 3 sin 2 pt x x x x ⇔ + + = + 1 1 tan 3 sin 2 sin 2 x x x ⇔ + = + tan 3 3 x x k π π⇔ = ⇔ = + ( thỏa điều kiện (*)) Vây phương trình có một họ nghiệm duy nhất là: 3 x k π π= + với k Z∈ 2. Giải bất phương trình: 2 3 2 3 2 log ( 1) log ( 1) 0 (1) 3 4 x x x x + − + > − − Bài giải: Điều kiện: 2 1 0 1 (*) 43 4 0 x x xx x + > > −  ⇔  ≠− − ≠  Khi đó ta có: ( ) ( )( ) 2 32 3 2 log ( 1) 2 3log 22log ( 1) 3log ( 1) (1) 0 0 3 4 1 4 xx x x x x x + −+ − + ⇔ > ⇔ > − − + − ( ) 2log ( 1) 0 4 x x + ⇔ > − Vì 32 3log 2 0 1 0x − >  + > 2log ( 1) 0 4 0 x x + > ⇔  − > hoặc 2log ( 1) 0 4 0 x x + <  − < 0 4 x x > ⇔  > hoặc 0 4 x x <  < 0 4 x x < ⇔  > Kết hợp với điều kiện (*) ta có 1 0 4 x x − < <  > Vậy bất phương trình có tập nghiệm là ( ) ( )1, 0 4 ,S = − ∪ + ∞ Câu 3 (1,0 điểm). Tính tích phân: ln2 0 1 1x I dx e = − ∫ Bài giải:
  • 49. ln 2 0 1 1 x x x I e dx e e = − ∫ Đặt: 2 1 1 2x x x u e u e udu e dx= − ⇔ + = ⇔ = Đổi cận: ln 2 1x u= ⇒ = 0 0x u= ⇒ = ( ) 1 1 22 0 0 1 1 2 2 11 I udu du uu u = = ++∫ ∫ Đặt: tanu t= với , 2 2 t π π  ∈ −    ( )2 2 1 tan 1 cos du dt t dt t ⇒ = = + Đổi cận 1 4 u t π = ⇒ = 0 0u t= ⇒ = ( ) 4 4 2 2 0 0 1 2 tan 1 2 2 4 tan 1 2 0 I t dt dt t t π π π π = + = = = +∫ ∫ ĐS: 2 I π = Câu 4(1,0 điểm): 1. Cho số phức z thỏa mãn điều kiện: ( )1 2 2(1 2 )z i z i+ − = − . Tìm phần thực và phần ảo của số phức 2 3w z z= − Bài giải: Gọi ( , )z x yi x y R=+ ∈ ( ) ( ) ( )( )1 2 2(1 2 ) 1 2 2(1 2 )z i z i x yi i x yi i+ − = − ⇔ + + − − = − ( ) ( ) ( )2 2 2 2 4x y x i i⇔ − + − = + − 2 2 2 1 2 2 4 2 x y y z i x x −= =  ⇔ ⇔ ⇔ = +  − =− =  Khi đó: ( ) 22 3 2 3(2 ) 3w z z i i i= − = + − + =− + Vậy w có phần thực là 3− và phần ảo là 1
  • 50. 2. Cho tập E = {1; 2; 3; 4; 5}. Viết ngẫu nhiên lên bảng hai số tự nhiên , mỗi số gồm ba chữ số đôi một khác nhau thuộc tập E . Tính xác suất để trong hai số đó có đúng một số chẵn. Bài giải: Gọi X là tập hợp tất cả các số có 3 chữ số khác nhau lấy từ tập E ⇒số phần tử của X là 3 5 60X A= = Ta thấy trong 60 số của tập X bao gồm 2 42 24A× = số chẵn và 60 24 36− = số lẽ Gọi Ω là không gian mẫu của phép thử theo đề (tức là phép thử chọn hai số từ tập X) Suy ra số phần tử của không gian mẫu là: 2 60 1770CΩ= = Gọi A là biến cố trong hai số lấy ra có đúng một số chẵn Suy ra số phần tử của không gian biến cố A là 1 1 24 36 864A C CΩ = × = Vậy Xác suất của biến cố A là: 864 144 ( ) 1770 295 A P A Ω = = = Ω Câu 5(1,0 điểm): Cho hình chóp SABCD có đáy ABCD là hình chữ nhật có AB = a và SA vuông góc đáy . đường thẳng SC tạo với mặt phẳng (ABCD) một góc 300 . Gọi M là hình chiếu của A lên đường thẳng SB. Biết 3 3 a AM = . Tính theo a thể tích của khối chóp SABCD và khoảng cách từ điểm M đến mặt phẳng (SCD). Bài giải: Ta có ( )SA ABCD⊥ ⇒ AC là hình chiếu cuae SC lên mp(ABCD) ( ) ( ) ,( ) ,SC ABCD SC AC SCA⇒ = = (vì  0 90SAC = )  0 30gt SCA⇒ = Xét SAB∆ ta có: 2 2 2 2 1 1 1 2 2 a SA AS AH AB a = − = ⇒ = Xét SAC∆ ta có:  6 .cot 2 a AC SA SCA= = Theo Pitago cho ABC∆ ta có: 2 2 2 a BC AC AB= − = 3 1 1 . . . 3 3 6 SABCD ABCD a V SA S SA AB AD= = = S A B C D M K 0 30 3 3 a a
  • 51. Xét SAB∆ ta có: 2 2 2 2 2 2 1 . 3 SM SA SA SM SB SA SB SB SA AB = ⇔ = = = + Mặt khác BM cắt mp(SCD) tại S nên theo talet ta có: ( ) ( ) ( ) ( ) ( ) ,( ) 1 1 1 ,( ) ,( ) ,( ) (1) ,( ) 3 3 3 d M SCD MS d M SCD d B SCD d A SCD d B SCD BS ==⇔ = = (vì AB // (SCD)) Ta có: ( ) ( ) ( ) ( ) ( ) SCD SADCD SA CD SAD CD AD SCD SAD SD ⊥⊥  ⇒ ⊥ ⇒  ⊥ ∩ =  Dựng AK SD⊥ (với K SD∈ ) ( )AK SCD⇒ ⊥ ⇒ K là hình chiếu vuông góc của A lên mp(SCD) ( ),( ) (2)d A SCD AK⇒ = Xét SAD∆ ta có: 2 2 2 2 1 1 1 4 2 a AK AK AD AS a = + = ⇔ = (3) Từ (1), (2) và (3) ta có: ( ),( ) 6 a d M SCD = Vậy thể tích khối chóp SABCD là 3 6 SABCD a V = (đvtt) và ( ),( ) 6 a d M SCD = (đvđd) Câu 6(1,0 điểm):Trong không gian với hệ trục Oxyz, cho mặt phẳng (P): 2x + 2y + z – 5 = 0 và đường thẳng 2 2 3 : 1 1 2 x y z d − − − = = . Gọi ∆ là đường thẳng vừa cắt vừa vuông góc với d. Viết phương trình đường thẳng ∆ . Biết khoảng cách từ ∆ đến mặt phẳng (P) bằng 10. Bài giải: Mặt phẳng (P): 2x + 2y + z – 5 = 0 ⇒ (P) có VTPT là ( )2; 2;1Pn =  Đường thẳng 2 : 2 3 2 x t d y t z t = +  = +  = + ⇒ d có VTCP là ( )1;1; 2dn =  ( ) ( ) / / , 3; 3; 0p d P gt u n u d ∆ ∆  ⇒ ⇒ = = −  ⊥ ∆    là VTCP của ∆ Gọi M là giao điểm của d và ∆ ( )2 ; 2 ; 3 2M t t t⇒ + + + ∈∆ 2 2 2 4 (6 ; 6 ;11)4 2 4 2 3 2 5 ( ,( )) 10 10 1 5 6 ( 4 ; 4 ; 9)2 2 1 t Mt t t d M P t t M = ⇒+ + + + + −  = ⇔ = ⇔ + = ⇔  =− ⇒ − − −+ + 
  • 52. Vậy có hai đường thẳng ∆ cần tìm là 6 : 6 11 x t y t z = +  ∆ =−  = và 4 : 4 9 x t y t z =− +  ∆ =− −  = − Câu 7(1,0 điểm): Trong mặt phẳng với hệ trục tọa độ Oxy, cho hình thang ABCD với cạnh đáy AB và CD = 2AB. Gọi H là chân đường vuông góc hạ từ D xuống AC và M là trung điểm của HC. Biết tọa độ đỉnh B(5; 6), phương trình đường thẳng DH: 2x – y = 0, phương trình đường thẳng DM: x – 3y + 5 = 0, tìm tọa độ các đỉnh của hình thang ABCD. Bài giải: Gọi I là giao điểm của AC và BD D DH DM= ∩ ⇒tọa độ D là nghiệm của hệ phương trình: 2 0 3 5 0 x y x y − =  − + = ( )1; 2D⇒ Theo talet ta có: ( ) 1 5 3( 5) 11 14 2 3 ; 2 6 3 6 3 3 I I xID DC BD BI I yIB AB − = −   = =⇔ = ⇔ ⇒  −= −     Đường thẳng AC đi qua I và vuông góc DH ⇒ AC qua 11 14 ; 3 3 I       và có VTPT là ( )1; 2n =  : 2 13 0AC x y⇒ + − = Ta có: M AC DM= ∩ ⇒tọa độ D là nghiệm của hệ phương trình: 2 13 0 3 5 0 x y x y + − =  − + = 29 18 ; 5 5 M   ⇒     H AC DH= ∩ ⇒tọa độ H là nghiệm của hệ phương trình: 2 13 0 2 0 x y x y + − =  − = 13 26 ; 5 5 H   ⇒     M là trung điểm HC ( ) 2 7 7; 2 2 2 C M H C M H x x x C y y y = − = ⇔ ⇒ = − = ( ) ( ) ( ) 7 1 2 5 2 2; 6 2 2 2 6 A A x gt DC AB A y −= − ⇒ = ⇔ ⇒ − = −   Vậy các đỉnh cần tìm là : ( )2; 6A , ( )7; 2C và ( )1; 2D Bình luận : Với những bài toán có giả thuyết liên quan đến tỷ số độ dài thì ta nghĩ ngay đến kỹ thuật tìm điểm A B(5; 6) D C 2 0x y− = 3 5 0x y− + = M H I
  • 53. bằngđẳng thức vectơ và viết phương trình đường liên quan đến góc. Câu 8(1,0 điểm): Giải hệ phương trình : 2 2 4 ( 5) 1 4 2 2 4 ( 4) 2 1 x y x y x y y x x x  + − −= − +  − + = − Bài giải: Điều kiện: 1 0 x y ≥  ≥ 2 2 4 20 1 4 2 2 (1) 4 16 2 1 (2) x xy y y x y hpt xy y x x  + − −= − + ⇔  − + = − Lấy (1) trừ (2) vế theo vế ta có: 2 2 4 1 4 2 2 2 1x y x y x y x− − − = − + − − 2 2 2 1 1 4 2 2 4 (3)x x y y y⇔ + − −= + + Đặt : 2 2 1 0 1 22 0 u x x u y vv y  = − ≥ = +  ⇒  == ≥   ( ) 22 4 2 4 2 4 2 (3) 1 2 1 2 2 2 2 2 2 (4)u u v v v u u u v v v⇔ + + − = + + ⇔ + + = + + Xét hàm số: 4 2 ( ) 2 2f t t t t= + + với [ )0 ,t∀ ∈ + ∞ Ta có: 3 '( ) 4 4 2 0f t t t= + + > , [ )0 ,t∀ ∈ + ∞ ( )f t⇒ liên tục và đơn điệu tăng trên [ )0 , + ∞ Do đó: (4) ( ) ( ) 1 2 2 1f u f v u v x y y x⇔ = ⇔ = ⇔ − = ⇔ = − Thay 2 1y x= − vào (2) ta có: 2 2 ( 1) 8( 1) 2 1 2 9 8 2 1 (5)x x x x x x x x− − − + = − ⇔ − + = − ( ) 2 2 2 17 5 1 1 17 45 2 9 8 2 1 0 2 10 0 12 2 2 2 1 2 x x x x x x x x x x x − +     ⇔ − + − − + − − − = ⇔ − + + =         − + −    2 2 5 2 2 3 2 2 17 1 17 2 45 2 0 5 0 14 4 5 2 2 3 2 22 1 2 2 4 x y x x x x x x x y   + + = ⇒=    ⇔ − + + = ⇔ − + = ⇔       − −− + −  = ⇒=     
  • 54. Vậy hệ phương trình có hai nghiệm: 5 2 2 2 3 2 2 4 x y  + =   + = và 5 2 2 2 3 2 2 4 x y  − =   − = Bình luận: • Với những hệ có cấu trúc của hai phương trình lệch nhau thì thông thường ta sẽ biến đổi (nhân liên hợp , đặt ẩn phụ để quy về dạng tích hoặc thuần nhất, hàm số…)một phương trình rồi thế vào phương trình còn lại. • Với những hệ có cấu trúc của hai phương trình tương đồng nhau thì thông thường ta phải sử lý đồng thời cả hai phương trình cùng một lúc như sau: +) Cộng hoặc trừ hai phương trình lại để có thể quy chúng về dạng tích hoặc cấu trúc hàm số. +) Biến đổi hệ theo 2 nhóm biến nào đó, sau đó đặt ẩn phụ để quy hệ về dạng đơn giản hơn • Đối với pt(5) để tìm ra nhóm 1 2 x   −    thì ta cần xét phép thử sau: ( ) ( ) ( ) 2 2 2 2 2 (2 4) 4 2 9 8 2 1 0 2 10 8 2 1 x m m x x x m x m x x x m x m x + − + + − + − + + + − − = ⇔ − + − + + − − Để có được nhóm chung thì : 2 1 2 4 4 1 2 10 8 2 m m m m − + = = ⇒ =− − − lượng thêm bớt là 1 2 x   −    • Ta có thể giải (5) theo cách khác như sau: ( ) 22 2 (5) 2 8 8 2 1 2( 2) 1 1x x x x x x ⇔ − + = + − ⇔ − = − +  Câu 9(1,0 điểm): Cho x, y, z là các số thực dương thỏa mãn xyz + x + z = y . Tìm giá trị lớn nhất của biểu thức ( )2 2 2 2 2 2 2 4 3 1 1 1 1 1 z z P x y z z z = − − + + + + + + Bài giải: Ta có : ( )1 1 x z xyz x z y y xz xz + + + = ⇔ = ≠ − Ta có: ( ) ( )( ) ( )( ) 2 2 22 2 2 2 2 2 2 2 2 (1 ) 22 12 2 2 2 2 (1) 1 1 1 1 1 1 1 1 1 1 z z x xxz x y x x x z x zx z xz  − +−  − = − = − = + + + + + + + ++  +  − 
  • 55. Theo BĐT bu-nhi-a-cop-xki ta có: ( ) ( ) ( )22 2 2 2 2 2 2 (1 ) 2 .(1 ) 1. 2 1 (1 ) 2 1 1 (2)z x x z x x z x x z x− + ≤ − + ≤ + − + = + + Dấu “=” xảy ra khi 2 2 2 (1 ) 2 0 1 21 2 0 1 z x x x xzx x z  − + ≥  ⇔ − = − = >  (vì x > 0 và z > 0) Từ (1) và (2) ta có: ( ) ( )( ) 2 2 2 2 2 2 2 2 1 12 2 2 1 1 1 1 1 z z x z x y x z z + + − ≤ = + + + + + Khi đó : ( ) ( )2 2 2 2 2 2 2 2 4 3 3 2 1 1 1 1 1 1 1 z z z z z P z z z z z z z ≤ − + = − + + + + + + + Đặt : ( )2 0 ,1 1 z t z = ∈ + (vì : 0z > và 2 2 2 1 1 1 z z z z z + > = ⇔ < + ) 2 2 2 2 2 2 1 1 1 1 1 1 1 z t t z z z = =− ⇔ =− + + + Khi đó ta có: ( )2 3 3 1 2 ( ) 3P t t t P f t t t≤ − − ⇔ ≤ =− + Xét hàm số: 3 ( ) 3f t t t=− + với ( )0 ,1t ∈ Ta có: 2 '( ) 9 1f t t=− + , 1 '( ) 0 3 f t x= ⇔ = hoặc 1 3 x = − Bảng biến thiên: 0 0 0 1 + -+- - t 1 3 1 3 − 0 -2 8 9 '( )f t ( )f t
  • 56. Từ Bảng biến thiên ta có: ( ) 8 ( ) , 0 ,1 9 f t t≤ ∈ Dấu bằng xảy ra khi 2 1 1 2 3 3 41 z t z z = ⇔ = ⇔ = + Vậy giá trị lớn nhất của 8 9 P = khi 1 2 2 2 x z y  = =   = --- Hết---